Калькулятор онлайн решение неравенств: Решение неравенств с модулем онлайн · Как пользоваться Контрольная Работа РУ

2 + 3x + 1) > 0

Подробнее про Неравенство.

Указанные выше примеры содержат также:

  • модуль или абсолютное значение: absolute(x) или |x|
  • квадратные корни sqrt(x),
    кубические корни cbrt(x)
  • тригонометрические функции:
    синус sin(x), косинус cos(x), тангенс tan(x), котангенс ctan(x)
  • показательные функции и экспоненты exp(x)
  • обратные тригонометрические функции:
    арксинус asin(x), арккосинус acos(x), арктангенс atan(x), арккотангенс acot(x)
  • натуральные логарифмы ln(x),
    десятичные логарифмы log(x)
  • гиперболические функции:
    гиперболический синус sh(x), гиперболический косинус ch(x), гиперболический тангенс и котангенс tanh(x), ctanh(x)
  • обратные гиперболические функции:
    гиперболический арксинус asinh(x), гиперболический арккосинус acosh(x), гиперболический арктангенс atanh(x), гиперболический арккотангенс acoth(x)
  • другие тригонометрические и гиперболические функции:
    секанс sec(x), косеканс csc(x), арксеканс asec(x), арккосеканс acsc(x), гиперболический секанс sech(x), гиперболический косеканс csch(x), гиперболический арксеканс asech(x), гиперболический арккосеканс acsch(x)
  • функции округления:
    в меньшую сторону floor(x), в большую сторону ceiling(x)
  • знак числа:
    sign(x)
  • для теории вероятности:
    функция ошибок erf(x) (интеграл вероятности), функция Лапласа laplace(x)
  • Факториал от x:
    x! или factorial(x)
  • Гамма-функция gamma(x)
  • Функция Ламберта LambertW(x)
  • Тригонометрические интегралы: Si(x), Ci(x), Shi(x), Chi(x)
Правила ввода

Можно делать следующие операции

2*x
— умножение
3/x
— деление
x^2
— возведение в квадрат
x^3
— возведение в куб
x^5
— возведение в степень
x + 7
— сложение
x — 6
— вычитание
Действительные числа
вводить в виде 7. 5, не 7,5
Постоянные
pi
— число Пи
e
— основание натурального логарифма
i
— комплексное число
oo
— символ бесконечности

7 Лучшее решение калькулятора неравенства Чтобы выбрать

лучшее решение калькулятора неравенства

Онлайн-калькуляторы для решения неравенства разработаны группами высококвалифицированных математиков и программистов, чтобы быть невероятно полезными и удобными для пользователя, но при этом способными давать точные результаты. конечному пользователю.

Основное программное обеспечение, которое управляет вычислением неравенства , обычно основано на алгоритме решения неравенства, который был разработан в течение многих лет академических исследований, но эта информация никогда не была доступна широкой публике до сих пор.

Этот уникальный алгоритм решает неравенства с однозначными коэффициентами за считанные секунды, а это значит, что вам не придется часами заниматься сложной математикой, чтобы найти ответы, необходимые для домашних заданий или классных проектов.

Удобный калькулятор неравенства может стать отличным инструментом для учащихся на уроках математики и естественных наук.

Однако найти хороший может быть непросто. К счастью, есть несколько замечательных, доступных для бесплатной загрузки или доступных онлайн в Интернете.

В этом посте рассказывается о лучшем онлайн-калькуляторе для поиска пошаговых решений неравенства, поэтому вы можете найти лучший калькулятор решения неравенства для себя.

В нем также будет представлен краткий обзор того, что такое неравенства и как их можно решить с помощью калькулятора неравенства.

1. Mathpapa

Это приложение имеет понятный интерфейс с удобными кнопками и опциями для учащихся.

Он также предлагает интуитивно понятное пошаговое решение для решения неравенств, которое идеально подходит для тех, кому сложно решить эти проблемы самостоятельно.

Он предоставляет вам пошаговых решения неравенств, просто введя вопрос в калькулятор и нажав «решить».

Предположим, вы все еще не понимаете вопрос или вам трудно понять концепцию неравенства.

На этот случай в Mathpapa есть множество доступных видеороликов, объясняющих концепцию и помогающих понять основы неравенств.

2. Калькулятор линейного неравенства

Это приложение для расчета линейного неравенства отображает решения для различных линейных неравенств.

Это приложение позволяет выполнять сложные расчеты для широкого спектра неравенств с глубоким пониманием пошаговых решений.

Решения включают квадратичный метод, закрытые интервалы и факторы. Это приложение совместимо с Android, iOS и настольными компьютерами.

Приложение действительно быстрое и простое в использовании, занимает мало места на вашем телефоне и работает в полноэкранном режиме.

3. Научный калькулятор NCalc

Научный калькулятор NCalc — это новый способ изучать и практиковать математические уравнения.

Этот калькулятор использует новейшие технологии искусственного интеллекта для автоматического решения неравенств и математических уравнений, которые вы вводите, без какой-либо ручной работы.

Удобный интерфейс и простой подход упрощают работу, поэтому вам не нужно тратить время на запутанные, сложные формулы и расчеты.

4. Quickmath

Решить калькулятор неравенства Веб-сайты отлично подходят, когда вам нужно решить проблему неравенства. Эти сайты проведут вас шаг за шагом, рассказывая, что делать и как вы можете это сделать.

Quickmath — самый популярный веб-сайт калькулятора неравенства. Вы заинтересованы в получении быстрого математического решения для данного неравенства? quickmath — ваш всеобъемлющий ресурс для решения неравенств.

Вы получите результаты в течение нескольких секунд и сможете проверить путь к вашему решению.

quickmath позволяет решить неравенства всего одним щелчком мыши, вы можете нарисовать числовую линию, введя неравенство, а также нарисовать график построенных неравенств за считанные секунды.

Вы можете выбрать толщину линии, линии сетки, переменные, а также объединение и пересечение областей при рисовании графика. Интерфейс очень удобный и бесплатный .

5. Mathway

Калькулятор неравенства — еще один популярный веб-сайт с калькулятором неравенства, который имеет аналогичный интерфейс. Он вычисляет неравенство между двумя выражениями и отображает результат в простой для понимания таблице.

Этот калькулятор является важным инструментом, который позволяет быстро получить ответы с заданным решением. Результат может быть отображен в форме неравенства и записи интервала.

Если у вас есть проблемы с пониманием решения, вы можете упростить его, добавив больше шагов, чтобы сделать решение подробным и понятным.

Mathway также позволяет загружать изображение неравенства или вопроса с телефона или рабочего стола, что делает его еще более доступным и простым для получения прямых решений.

Для этого калькулятора также доступно мобильное приложение, которое можно найти как в магазине приложений, так и в магазине приложений Apple.

6. Symbolab

Symbolab — один из самых популярных сайтов-калькуляторов неравенств, позволяющий выполнять различные вычисления с высокой точностью и скоростью. Это удобный, простой в использовании калькулятор для решения любых неравенств.

Вы можете использовать Symbolab, чтобы помочь с домашними и школьными заданиями или выполнять вычисления. Все, что вам нужно сделать, это ввести уравнение или неравенство в Symbolab, и он предоставит вам пошаговые решения неравенств.

Если вы хотите представить свой ответ в виде числовой прямой или нарисовать график из неравенства, вам поможет Symbolab.

Вы можете получить пошаговое решение, представление на числовой прямой и графическое представление, просто введя неравенство в калькулятор.

7. Калькулятор Wolfram Alpha

Wolfram Alpha — это веб-сайт, созданный для помощи студентам и профессионалам в понимании математики.

Одной из самых ценных функций Wolfram Alpha является решатель, который может помочь вам найти ответы на проблемы, которые на первый взгляд кажутся сложными.

Но как с его помощью решать неравенства? Есть два способа использования Wolfram Alpha для решения неравенств.

Первый и наиболее распространенный способ — это когда ваше неравенство содержит одну переменную рядом со знаком = (так называемый знак равенства).

Один из этих типов уравнений требует числового ответа; решить их так же просто, как подключить свое численное решение. Например, если x>3, то -4

Используя решатель Wolfram alpha, мы получаем числовой ответ 6, потому что x=6 удовлетворяет обоим условиям.

Как пользоваться калькулятором неравенства

Калькулятор неравенства — это тип решения математических уравнений, который может помочь вам найти ответ на неравенство.

Калькулятор решения неравенства предоставляет вам более быстрый и эффективный способ проверки вашей работы. Этот калькулятор предназначен для решения одного шага за раз, что позволяет вам легко убедиться, что ваш ответ правильный.

Чтобы использовать калькулятор неравенства, выполните следующие простые действия:

  • Введите ваши математические выражения в поля ввода калькулятора. Убедитесь, что все значения заключены в круглые и квадратные скобки. Кроме того, убедитесь, что все дроби или показатели степени указаны в соответствующих местах.
  • Нажмите кнопку Решить после того, как вы закончите вводить утверждение в поле ввода.
  • Просмотрите свои результаты, нажав ‘ Показать раствор’ .

Лучший способ решения линейных уравнений состоит в выполнении следующих простых шагов:

  • Определите, для какого уравнения решается, и убедитесь, что оно находится по одну сторону от знака равенства.
  • Используйте операцию, противоположную тому, что вы решаете, чтобы получить другую часть уравнения; используйте сложение, если вы решаете для вычитания; используйте умножение, если вы решаете деление; и наоборот.

Решение квадратных уравнений с одной переменной

Квадратное уравнение является основным алгебраическим уравнением, которое обычно имеет вид ax2 + bx + c = 0. Это уравнение имеет два решения, когда x = +/- квадратный корень из b2-4ac (т. коэффициент х2).

Чтобы найти решения на калькуляторе, возьмите квадратный корень из b2-4ac и разделите его на 2a. Это будет одно решение для x, если вы снова разделите его на 2a, это будет другое.

Конец примечания

Существует множество причин, по которым решение калькулятора неравенства может быть полезным, особенно если учесть, насколько сложными могут быть некоторые неравенства. Эти калькуляторы упрощают поиск решений ваших вопросов о неравенстве.

Разобраться, как решить неравенство, может быть довольно сложно. Это может показаться достаточно простым, но на самом деле это не так просто, как кажется. Это потому, что для решения этих проблем не всегда используется одна формула.

Некоторые методы будут эффективны для конкретных проблем, в то время как во многих других случаях может потребоваться другой подход.

Поиск правильного уравнения в каждом конкретном случае может занять много времени и усилий, поэтому так много людей обращаются к для решения калькулятора неравенства .

Калькулятор неравенства позволяет им быстро и легко найти ответ, поэтому им не придется прилагать все усилия самостоятельно; просто введите свой вопрос в калькулятор, чтобы сразу же начать получать ответы!

Просто введите свои неравенства и уравнения и нажмите «Решить», чтобы увидеть простые инструкции, объясняющие, как решать неравенства.

Все решения являются пошаговыми, а это значит, что вы больше никогда не застрянете на проблеме — независимо от того, какой тип неравенства или выражения вам представлен.

сообщить об этом объявлении

Рабочий лист графика и подстановки ответ ключ все вещи алгебра 0005

Графики и операции замещения — YouTube

www. youtube.com › смотреть

13.01.2021 · Как найти значение x+y | Хорошая задача по алгебре | Математическая олимпиада. MathZeroByZero …
Время: 36:27
Прислан: 13.01.2021

Графики и подстановки — YouTube

www.youtube.com › смотреть

09.11.2017 · KutaSoftware: Алгебра 1-Системы уравнений Словесные задачи Часть 1. MaeMap. MaeMap …
Dauer: 16:38
Прислан: 09.11.2017

Джина Уилсон все вещи алгебра 2015 блок ответов 5 9HP7HY

shxgkkcdi.koenenandco.nl 90 005

Результаты 1 — 24 из 422 · … Все вопросы по алгебре 2013, ответы на вопросы, Графики и подстановки… Множество бесплатных математических листов по адресу: Все вопросы по алгебре®. Учебная программа …

Подстановка и графические учебные ресурсы — TPT

www.teacherspayteachers.com › Обзор › Search:su…

Результаты 1–19 из 19+ · Этот системный рабочий лист является партнерской деятельностью. Один партнер решает систему с помощью алгебры. Другой использует метод графика. Оба …

All Things Algebra Answer Key Unit 5

ptgtfpebj.fizjoprofit.pl

Результаты 1–24 из 36 ключ, Графики против замены Джина …

Джина Уилсон все алгебра системы уравнений приложения …

ddygrlhhm.aven-habitat.fr

Результаты 1 — 24 из 53000+ Уравнения и неравенства Unit Bundle …

gina wilson алгебра всего, системы уравнений, приложения …

pgwedfxfx.rems-murr-pokal.de

Результаты 1–24 из 53000+ · Gina Wilson All Things Algebra Раздел 5. Рабочие листы с ключевыми ответами представляют собой графики и замену. Ответьте на все вопросы, …

Диаграмма против подстановки Ключ ответа

fiwhlqhnz.polskiecentrumcaravaningowe.pl

График против подстановки Ключ ответа Все Алгебра — Во многих областях чтение графиков может быть полезным навыком. Рабочий лист графика и замены …

Ähnliche Fragen

Как вы решаете задачу замены и построения графика?

Как шаг за шагом решать уравнения подстановки?

Когда вы решаете систему уравнений с помощью графика, каков ответ?

[PDF] Kuta Software — Infinite Algebra 1

www.

A 1 a 2 a n: Решение. Задание 19, Вариант 1

HIV 1, 2 Ag/Ab Combo (определение антител к ВИЧ типов 1 и 2 и антигена p24)

Исследование специфических антител и антигена p24 вируса иммунодефицита человека.

Синонимы русские

Антитела к ВИЧ 1, 2, антитела к вирусу иммунодефицита человека, ВИЧ-1 p24, ВИЧ-1 антиген, p24-антиген, антитела IgM и IgG суммарно.

Синонимы английские

Anti-HIV, HIV antibodies, human immunodeficiency virus antibodies, HIV-1 p24, HIV-1 Ag, p24-antigen, IgG, IgM total antibody test.

Метод исследования

Электрохемилюминесцентный иммуноанализ (ECLIA).

Какой биоматериал можно использовать для исследования?

Венозную кровь.

Как правильно подготовиться к исследованию?

  • Исследование на наличие ВИЧ-инфекции может проводиться анонимно и конфиденциально. При конфиденциальном обследовании в обязательном порядке необходимо предъявление паспорта.
  • Не курить в течение 30 минут до исследования.

Общая информация об исследовании

ВИЧ (вирус иммунодефицита человека) – вирус семейства ретровирусов, который поражает клетки иммунной системы человека (CD4, Т-хелперы). Вызывает СПИД.

ВИЧ-1 – наиболее распространенный тип вируса, чаще всего встречающийся в России, США, Европе, Японии и Австралии (как правило, субтип В).

ВИЧ-2 – редкий тип, распространен в Западной Африке.

Для диагностики вируса иммунодефицита человека используется комбинированная тест-система четвертого поколения, способная определять ВИЧ-инфекцию уже через 2 недели после попадания вируса в кровь, тогда как тест-системы первого поколения делают это только через 6-12 недель с момента инфицирования.

Преимуществом этого комбинированного ВИЧ-анализа является выявление, благодаря использованию антител к ВИЧ-1 p24 в качестве реагентов, специфического антигена p24 (белка вирусного капсида), который может быть обнаружен данным тестом уже через 1-4 недели с момента инфицирования, т.  е. еще до сероконверсии, что значительно сокращает «период окна».

Кроме того, такой анализ на ВИЧ выявляет в крови антитела к ВИЧ-1 и ВИЧ-2 (с использованием реакции антиген-антитело), которые вырабатываются в достаточном количестве для определения тест-системой через 2-8 недель с момента заражения.

После сероконверсии антитела начинают связываться с антигеном p24, в результате чего тест на антитела к ВИЧ будет положительным, а тест на p24 – отрицательным. Однако спустя некоторое время в крови будут определяться и антитела, и антиген одновременно. На терминальной стадии СПИД-тест на антитела к ВИЧ может давать отрицательный результат, так как нарушается механизм выработки антител.

Стадии ВИЧ-инфекции

  1. Инкубационный период, или «период серонегативного окна»,  – время с момента заражения до выработки в крови защитных антител к вирусу, когда тесты на антитела к ВИЧ отрицательны, но человек уже может передавать вирус другим людям. Продолжительность данного периода – от 2 недель до 6 месяцев.
  2. Период острой ВИЧ инфекции наступает в среднем через 2-4 недели с момента инфицирования и длится примерно 2-3 недели. На данном этапе у некоторых людей могут развиться неспецифические симптомы, схожие с симптомами гриппа, что связано с активной репликацией вируса.
  3. Латентная стадия протекает бессимптомно, но в течение нее происходит постепенное снижение иммунитета и увеличение количества вируса в крови.
  4. СПИД (синдром приобретенного иммунодефицита) является конечной стадией развития ВИЧ-инфекции, которая характеризуется сильным угнетением иммунной системы, а также сопутствующими заболеваниями, энцефалопатией или онкологическими заболеваниями.

Несмотря на то что ВИЧ-инфекция неизлечима, сегодня существует высокоактивная антиретровирусная терапия (АРВТ), которая может значительно продлить жизнь ВИЧ-инфицированного и улучшить ее качество.

Данный тест имеет особенно высокую диагностическую ценность, если инфицирование ВИЧ произошло незадолго до момента тестирования (за 2-4 недели).

Для чего используется исследование?

Анализ используется для ранней диагностики ВИЧ, что позволяет предотвратить дальнейшую передачу вируса другим людям, а также своевременно начать антиретровирусную терапию и лечение заболеваний, способствующих прогрессированию ВИЧ-инфекции.

Когда назначается исследование?

  • При устойчивой симптоматике (на протяжении 2-3 недель) неясной этиологии: субфебрильной температуре, поносе, ночной потливости, резкой потере веса, увеличении лимфатических узлов.
  • При рецидивирующей герпетической инфекции, вирусных гепатитах, пневмонии, туберкулезе, токсоплазмозе.
  • Если пациент страдает заболеваниями, передающимися половым путем (сифилисом, хламидиозом, гонореей, генитальным герпесом, бактериальным вагинозом).
  • Если у пациента был незащищенный вагинальный, анальный или оральный секс с несколькими половыми партнерами, новым партнером или партнером, в ВИЧ-статусе которого пациент не уверен.
  • Когда пациент проходил процедуру переливания донорской крови (хотя случаи инфицирования таким путем практически исключены, т. к. кровь тщательно тестируется на наличие вирусных частиц и подвергается специальной термообработке).
  • Если пациент употреблял наркотики инъекционным путем, используя нестерильные инструменты.
  • При беременности / планировании беременности (прием азидотимидина во время беременности, кесарево сечение во избежание передачи вируса ребенку в момент прохождения через родовые пути и отказ от кормления грудью снижают риск передачи ВИЧ от матери к ребенку с 30 % до 1 %).
  • Случайный укол шприцом или другим предметом (например, медицинским инструментом), содержащим в себе инфицированную кровь (в таких случаях вероятность инфицирования крайне низкая).

Что означают результаты?

Референсные значения

Результат: отрицательный.

Причины отрицательного результата:

  • отсутствие ВИЧ-инфекции,
  • период серонегативного окна (ни антиген, ни антитела еще не выработались в достаточном количестве, необходимом для определения тест-системой).

Причины положительного результата:

  • инфицирование ВИЧ.

Важные замечания

  • Диагностика антител к ВИЧ у детей грудного возраста, рождённых от ВИЧ-инфицированных матерей, затруднена, так как младенец получает антитела от матери через плацентарную кровь. Как правило, тест на антитела к ВИЧ у таких детей становится отрицательным не позднее 18 месяцев, если ребенок не инфицирован ВИЧ.
  • С помощью данного ВИЧ-теста невозможно определить, как давно произошло инфицирование, или стадию ВИЧ (например, СПИД).
  • ВИЧ содержится практически во всех жидкостях организма, но только в крови, сперме и вагинальном секрете концентрация вируса достаточна для инфицирования. Кроме того, вирус нестабилен и способен жить лишь в жидких средах человеческого организма, поэтому ВИЧ-инфекция не передается через поцелуй, укусы насекомых и при бытовых контактах (например, при использовании общего туалета, через слюну, воду и пищевые продукты).
  • Данный анализ на ВИЧ хотя и сокращает «период окна», но все же способен определить наличие антигена/антител не ранее чем через 1-3 недели с момента возможного инфицирования.
  • Если событие, грозящее заражением ВИЧ-инфекцией, произошло менее чем за 1-3 недели до тестирования, рекомендуется повторить тест.
  • Инкубационный период может составлять до полугода, однако всё индивидуально и зависит от вирусной и бактериальной нагрузки и иммунного ответа организма.
  • Анализы первого и третьего поколений могли давать ложноположительный результат, если в крови обследуемого присутствовали антитела к вирусу Эпштейна –  Барр, ревматоидному фактору, главному комплексу гистосовместимости HLA или антитела после введения вакцины против ВИЧ. Однако вероятность ложноположительного результата при комбинированном тесте практически исключена.
  • В случае положительного результата тестирования проводится подтверждающий анализ с использованием метода иммуноблотинга (тест на антитела к ряду специфических белков вируса).

Кто назначает исследование?

Врач общей практики, терапевт, инфекционист, дерматовенеролог.

В чем разница между USB 3.1 Gen 1, Gen 2 и USB 3.2?

Search Kingston.com

Чтобы начать, нажмите принять ниже, чтобы открыть панель управления файлами cookie. Затем нажмите кнопку Персонализация, чтобы включить функцию чата, а затем Сохранить.

Версия вашего веб-браузера устарела. Обновите браузер для повышения удобства работы с этим веб-сайтом. https://browser-update.org/update-browser.html

апр 2019

  • USB Flash Drives
  • Мобильный образ жизни
  • Персональное хранилище
  • USB-C
Блог Главная

Понимание стандартов USB и их спецификаций может быть непростой задачей, особенно с учетом множества их обновлений. Мы ответим на вопрос о том, в чем состоит различие между USB 3.1 Gen 1 и Gen 2, и поговорим о том, почему Gen 2 лучше Gen 1, а также приведем другую полезную информацию, которая позволит вам понять все, что нужно знать о стандартах USB.

USB 3.0, выпущенный более десяти лет назад в 2008 году, стал третьей основной версией стандарта USB. Он обеспечил существенное улучшение по сравнению с USB 2.0, который впервые появился в 2000 году и обеспечивал скорость передачи данных всего 480 Мбит/с. С тех пор мы перешли от USB 3.0, который теперь известен как USB 3.1 Gen 1. Следовательно, USB 3.0 — это то же самое, что USB 3.1 Gen 1.

USB-IF, организация, сопровождающая спецификации USB (универсальная последовательная шина) и обеспечивающая соответствие требованиям, сделала это для того, чтобы разработчикам и производителям было проще получать одну и ту же важную информацию для обеспечения надлежащей разработки продуктов и обратной совместимости. Она отвечает за правила именования USB-кабелей и устройств.

Сравнение USB 3.1 Gen 1 и USB 3.1 Gen 2

Различие между USB 3.1 Gen 1 и USB 3.1 Gen 2 состоит исключительно в скорости. USB 3.1 Gen 1 поддерживает скорости до 5 Гбит/с, а USB 3.1 Gen 2 — до 10 Гбит/с. USB-IF намеревалась использовать различные названия для обозначения USB 3. 1 Gen 1 и USB 3.1 Gen 2, что было бы лучше для целей маркетинга. Предполагалось называть USB 3.1 Gen 1 и Gen 2 «SuperSpeed USB» и «SuperSpeed USB+» соответственно, но эти названия так и не прижились в отрасли. Часто, чтобы различить эти два стандарта USB, OEM-производители добавляют скорости 5 Гбит/с или 10 Гбит/с в свои таблицы спецификаций. Другие же просто называют их «USB 3.1 Gen 1» или «USB 3.1 Gen 2».

Появление USB 3.2

С тех пор технология USB продвинулась еще дальше, и в 2017 году появился стандарт USB 3.2. Есть четыре различных варианта USB 3.2 со своими собственными названиями. Вот эти четыре варианта USB 3.2:

USB 3.2
Gen 1×1

USB 3.2
Gen 1×2

USB 3.2
Gen 2×1

USB 3.2
Gen 2×2

Скорость передачи
данных
5Gbps10Gbps10Gbps20Gbps
Предыдущее
название

USB 3. 1 Gen 1
и USB 3.0

USB 3.1 Gen 2
Варианты
интерфейса

USB-A, USB-C,
microUSB

Только USB-C

USB-A, USB-C,
microUSB

Только USB-C


С появлением стандарта USB 3.2 преобладающие в отрасли соединения USB-A стали постепенно заменяться на USB-C. Поскольку USB-C поддерживает более высокие скорости передачи данных и позволяет быстрее заряжать другие периферийные устройства, он, естественно, стал основным USB-разъемом при использовании USB 3.2 Gen 2.

Thunderbolt — торговая марка аппаратного интерфейса, разработанного Intel® в сотрудничестве с Apple®, который соединяет периферийные устройства с компьютером. Thunderbolt 1 и 2 используют тот же разъем, что и Mini DisplayPort (MDP), тогда как для Thunderbolt 3 используется USB-C.

USB4 — следующий шаг

Следующим шагом в развитии технологии USB станет стандарт USB4. Он обеспечит скорость передачи данных 40 Гбит/с, совместимость с Thunderbolt 3 и будет использовать только разъем USB-C. Стандарт USB4 направлен на увеличение пропускной способности и ориентирован на конвергенцию экосистемы разъемов USB-C и минимизацию путаницы для конечных пользователей.

Вот ряд ключевых особенностей USB4:

  • Функции отображения, передачи данных и загрузки/сохранения с помощью одного разъема USB-C.
  • Универсальная совместимость с существующими продуктами USB и Thunderbolt.
  • Возможности определения портов для обеспечения прогнозируемых и последовательных возможностей пользователя.
  • Повышение гибкости хоста в плане настройки полосы пропускания, управления питанием и других параметров, связанных с производительностью системы.

Стандарт USB прошел долгий путь с момента своего первого появления и будет продолжать развиваться в будущем. Когда речь заходит о USB 3.1 Gen 1 и Gen 2, единственным различием между ними является скорость, и они обратно совместимы с версиями USB 3. 0 и USB 2.0. В будущем, с появлением новых поколений стандартов USB и внедрением соединений USB-C, улучшения будут еще более существенными.

#KingstonIsWithYou

4:59

В чем разница между USB 3.1 Gen 1, Gen 2 и USB 3.2?

Есть различные типы USB. Это сбивает с толку! Мы раскроем тайну названий, чтобы вы точно знали, что получаете и соответствует ли это вашим конкретным потребностям.

Сортировать по Название — от A до Z

No products were found matching your selection

Фильтр Filters Applied

Сортировать по Название — от A до Z

No products were found matching your selection

3-8 9 Оценить квадратный корень из 12 10 Оценить квадратный корень из 20 11 Оценить квадратный корень из 50 94 18 Оценить квадратный корень из 45 19 Оценить квадратный корень из 32 20 Оценить квадратный корень из 18 9{n+1}-1}{a-1}$ по индукции

спросил

Изменено 10 лет, 5 месяцев назад

Просмотрено 45 тысяч раз

$\begingroup$

Как с помощью математической индукции показать, что для $ n \ge 0$ и $a \ne 1$ справедливо следующее утверждение? 9{n+1}-1}{a-1}$$

Я понимаю принцип математической индукции, но понятия не имею, как его применить здесь.

1 к 10 как понять: Соотношение 1 к 10 – это сколько?

Каково соотношение 1 к 10? – Обзоры Вики

Таким образом, как часть 3/30 можно упростить до 1/10, соотношение 3:30 (или 4:40, 5:50, 6:60 и т. д.) можно упростить до 1:10.

Отсюда, что такое 25% как отношение? Следуя той же логике, 25% 1:3 а не 1: 4, иначе 50% будут равны 1: 2 (но 1 из 2 — это пропорция, а не соотношение).

Каково соотношение 2 к 3? 2 + 3 = 5. Наше соотношение 2:3 содержит всего 5 частей.

Дополнительно Каково соотношение 5 к 4? Типичные соотношения сторон и размеры экранов и видео

Имя и фамилия Соотношение сторон Высота (пиксель)
576p 5:4 576
720p 16:9 720
1080p 16:9 1080
2160p (4K UHD) 16:9 2160

Каково соотношение 5 к 3? Соотношение 5 к 3 — простейшая форма соотношение 250 к 150, и все три отношения эквивалентны.

Какой процент составляет соотношение 2 к 1?

Простое преобразование процентов и соотношений.

Соотношение Процент -%
2.5 — 1 29%
2 — 1 33%
1.5 — 1 40%
1 — 1 50%

Каково соотношение 2 5? Соотношение 2 к 5 или 2:5 или 2/5. Все они описывают соотношение в различных формах дробей. Следовательно, отношение может быть выражено дробями или десятичными дробями. 2:5 в десятичных дробях равно 0.4.

Что такое соотношение 50%? 2: 4 можно записать как 2/4 = 0.5; Умножаем 0.5 на 100, получаем 0.5 × 100 = 50, поэтому соотношение 2: 4 составляет 50%.

Какое соотношение между 2 и 5?

Соотношение 2 к 5 или 2:5 или 2/5. Все они описывают соотношение в различных формах дробей. Следовательно, отношение может быть выражено дробями или десятичными дробями. 2:5 в десятичных дробях равно 0.4.

Также Как вы решаете проблемы с соотношением? Как решить задачу на соотношение

  1. Сложите доли коэффициента, чтобы найти общее количество долей.
  2. Разделите общую сумму на общее количество акций.
  3. Умножьте на необходимое количество акций.

Как рассчитать соотношение мужчин и женщин?

Определение: ПОЛОВОЕ СООТНОШЕНИЕ при РОЖДЕНИИ – это число живорождений мужского пола, проживающих в стране (для определенного географического региона, например, страны, штата или округа, за определенный период времени, обычно календарный год), деленное на число живорождений женского пола (для одного и того же география и период времени) и умножается на 100 или 1,000.

Каково соотношение 2 1? Например, если количество мальчиков и девочек на хоккейном матче составляет 2:1, мы знаем следующую информацию: мальчиков больше, чем девочек. Там 2 мальчика на каждую девушку. Количество мальчиков в два раза больше, чем девочек. , что равносильно утверждению, что девочек вдвое меньше, чем мальчиков.

Что такое соотношение 1 к 1?

Соотношение 1: 1 составляет 1 часть или 1 единица определенного количества. например, две бутылки воды содержат по 2 литра воды в каждой, соотношение будет 22=11=1:1.

Какое соотношение 1 7?

So 2: 14 и 3: 21 два соотношения, равные 1:7. Студентов также просят определить, равны ли два заданных отношения, сначала записав каждое отношение в виде дроби, а затем записав каждую дробь в наименьшем выражении.

Какое соотношение 4 к 1? Обратите внимание, что отношение 12 к 3 эквивалентно отношению 4 к 1, то есть 12:3 = 4: 1.

Какое соотношение 1 и 2?

Преобразовать отношение в дробь

Соотношение 1: 2 читается как «от 1 до 2». Это означает, что из трех целых одна часть стоит 3, а другая — 1.

Как рассчитать соотношение 2 1? Соотношение 2 к 1 это простейшая форма соотношения 4 к 2. И соотношения эквивалентны, потому что отношения между каждой парой чисел одинаковы. Например, если у нас есть отношение 250 к 150, мы можем упростить его, разделив оба числа на 10, а затем на 5, чтобы получить 5 до 3: 250: 150 25: 15 5: 3.

Какое соотношение 3 к 12?

Обратите внимание, что отношение 12 к 3 считается эквивалентным отношению 4 — 1, то есть 12:3 = 4:1.

Каково соотношение 10 и 6? Пример: упростите соотношение 6:10.

Упрощенное соотношение 3: 5.

Как вы вычисляете проблемы с соотношением?

Чтобы рассчитать соотношение суммы, мы разделите сумму на общее количество частей в соотношении, а затем умножьте этот ответ на исходное соотношение.

Как превратить 3 1 в проценты? «Если отношение «ДА» к «НЕТ» составляет 3:1, каковы проценты?» в этом случае ответ будет 75% «ДА» на 25% ‘НЕТ’.

Что такое соотношение 40%?

Процент — особый вид дроби

100% = 100 / 100 =1
50% = 50 / 100 = 5 / 10
25% = 25 / 100 = 5 / 20
40% = 40/100 = 4 / 10
5% = 5 / 100 = 1 / 20

Сравнение десятичных дробей — как правильно? правила и примеры

Поможем понять и полюбить математику

Начать учиться

Как столяр подбирает инструмент для разных задач: пила, отвертка, клей. Так и мы можем выбирать, как решать задачки. В этой статьем узнаем про способы сравнения десятичных дробей.

Понятие десятичной дроби

Прежде чем мы расскажем, как сравнивать десятичные дроби, вспомним основные определения, виды дробей и разницу между ними.

Дробь — это число в математика, в которой a и b — числа или выражения. По сути, это всего лишь одна из форм, в которое можно представить число. Есть два формата записи:

  • обыкновенный вид — 1/2 или a/b,
  • десятичный вид — 0,5.

В обыкновенной дроби над чертой принято писать делимое, которое становится числителем, а под чертой всегда находится делитель, который называют знаменателем. Черта между числителем и знаменателем означает деление.


В десятичной дроби знаменатель всегда равен 10, 100, 1000, 10000 и т. д. По сути, десятичная дробь — это то, что получается, если разделить числитель на знаменатель. Ее записывают в строчку через запятую, чтобы отделить целую часть от дробной. Вот так:

  • 0,1
  • 2,53
  • 9,932

Конечная десятичная дробь — это когда количество цифр после запятой точно определено.

Бесконечная десятичная дробь — это когда после запятой количество цифр бесконечно. Для удобства математики договорились округлять эти цифры до 1-3 после запятой. 

Узнай, какие профессии будущего тебе подойдут

Пройди тест — и мы покажем, кем ты можешь стать, а ещё пришлём подробный гайд, как реализовать себя уже сейчас

Свойства десятичных дробей

Главное свойство десятичной дроби звучит так: если к десятичной дроби справа приписать один или несколько нулей — ее величина не изменится. Это значит, что если в вашей дроби куча нулей — их можно просто отбросить. Например:

  • 0,600 = 0,6
  • 21,10200000 = 21,102

Обыкновенная и десятичная дробь — давние друзья. Вот, как они связаны:

  • Целая часть десятичной дроби равна целой части смешанной дроби. Если числитель меньше знаменателя, то целая часть равна нулю.
  • Дробная часть десятичной дроби содержит те же цифры, что и числитель этой же дроби в обыкновенном виде, если знаменатель обыкновенной дроби равен 10, 100,1000 и т. д.
  • Количество цифр после запятой зависит от количества нулей в знаменателе обыкновенной дроби, если знаменатель обыкновенной дроби равен 10, 100,1000 и т. д. То есть 1 цифра — делитель 10, 4 цифры — делитель 10000.

Курсы по математике в онлайн-школе Skysmart помогут подтянуть оценки, подготовиться к контрольным, ВПР и экзаменам.

Правило сравнения десятичных дробей

Чтобы сравнить две десятичные дроби, сначала нужно сравнить их целые части. Если целые части равны, продолжаем искать первый несовпадающий разряд. Большей будет та дробь, у которой соответствующий разряд больше.

Вот так с первой строчки раскрыли тему сравнения десятичных дробей 😜 Но это еще не все — едем дальше.

Алгоритм сравнения десятичных дробей

  1. Убедиться, что у обеих десятичных дробей одинаковое количество знаков (цифр) справа от запятой. Если нет, то дописать (убрать) нужное количество нулей в одной из десятичных дробей.
  2. Сравнить десятичные дроби слева направо. Целую часть с целой, десятые с десятыми, сотые с сотыми и т. д.
  3. Когда одна из частей десятичной дроби окажется больше, чем другая, эту дробь можно назвать большей.

Применим правило на практике. Сравним десятичные дроби: 15,7 и 15,719.

Как решаем:

  • Допишем в первой десятичной дроби нужное количество нулей, чтобы уравнять количество знаков справа от запятой: 15,700 и 15,719.
  • Сравним десятичные дроби слева направо.

Целую часть с целой частью: 15 = 15. Целые части равны.

Десятые с десятыми: 7 = 7. Десятые также равны.

Сотые с сотыми: 0 < 1. Так как сотые второй десятичной дроби больше, значит и сама дробь больше: 15,700 < 15,719.

Ответ: 15,7 < 15,719.

Еще один способ сравнения десятичных дробей:

Чтобы сравнить две десятичные дроби, нужно уравнять количество знаков после запятой (приписать к одной из них справа нули), затем отбросить запятую, и сравнить два натуральных числа.

Сравним 3,656 и 3,48.

Как решаем:

  • Уравниваем количество знаков справа после запятой: 3,656 и 3,480.
  • Отбросим запятые: 3656 и 3480.
  • Сравним полученные числа: 3656 > 3480.

Ответ: 3,656 > 3,48.

Запоминаем!

Меньшая десятичная дробь лежит на координатном луче левее большей, а большая — правее меньшей.

Например, 0,3 < 0,4 < 0,5, поэтому точка A (0,3) лежит левее точки B (0,4), а точка C (0,5) лежит правее точки B (0,4).


Шпаргалки для родителей по математике

Все формулы по математике под рукой

Лидия Казанцева

Автор Skysmart

К предыдущей статье

Умножение многочлена на многочлен

К следующей статье

Свойства умножения и деления

Получите план обучения, который поможет понять и полюбить математику

На вводном уроке с методистом

  1. Выявим пробелы в знаниях и дадим советы по обучению

  2. Расскажем, как проходят занятия

  3. Подберём курс

Как учить числа 1–10 в детском саду — KindergartenWorks

Восприятие чисел и обучение написанию и использованию чисел 0–10 — довольно сложная задача в первые девять недель детского сада.

Вот набор материалов, которые помогают мне учить дошкольников формировать числительные и учить их считать от одного до десяти.

Потому что, будем честными.

Много киндеров приходит на счет до десяти умений — а потом еще и — лотов нет. Я обнаружил, что очень часто поступающие ученики могут правильно сосчитать только до четырех или пяти предметов.

Вот то, что я создал для своего класса, и вы тоже можете это использовать.

Научите их с помощью истории

Точно так же, как я узнал, что обучение процедурам с помощью истории очень эффективно, я узнал, что обучение числам с помощью истории также эффективно.

Но мне не нравились никакие существующие книги, потому что они не учили студентов буквально «вот как считать» и «вот как сделать 4». А это как раз то, что мне было нужно.

Итак, я создал книгу чисел монстров, а также превратил ее в аудиокнигу.

Моим детям нравится , и это помогает мне многому научить за короткий промежуток времени.

В нем рассказывается о процессе счета 1:1 до четырех.

Он извлекает выгоду из того факта, что большинство учащихся могут сосчитать столько же предметов, но подталкивает их к пониманию того, что в счете используется шаблон, который они могут применять к каждому последующему числу.

Как учитель, вы просто используете повторяющиеся фразы из книги «Когда вы касаетесь одного, вы называете число. Когда вы касаетесь другого, вы называете следующее число», когда вы работаете с детьми, которым нужна помощь в счете и касании объекты один в один.

После этого повторяются простые для запоминания песнопения для формирования чисел, чтобы попрактиковаться в правильном формировании чисел.

Таким образом, вы можете предложить ученикам попрактиковаться в этом вместе с вами во время чтения – они быстро учатся петь вместе с вами и могут даже рисовать пальцами в воздухе цифры, когда вы читаете вслух.

Конечно, чтобы максимизировать число обучаемых с младшими учениками, нужно делать это более одного раза.

Вы можете распечатать эту историю и читать ее столько раз, сколько захотите.

Чтобы сделать его более увлекательным и интерактивным, давайте поговорим о том, как задействовать все тело ученика, чтобы научиться писать числа еще быстрее.

Научите их с помощью видео

Видео поможет научиться писать числа и сделать его более интерактивным. Итак, используйте это бесплатное видео Monster Numbers для детского сада в своих интересах!

Нам нравится показывать его на большом экране, чтобы практиковаться в составлении чисел руками, локтями, ногами и головами. Так весело!

Также отлично работает, если разбить его на части, чтобы сосредоточиться всего на паре цифр!

Наслаждайтесь этим бесплатным видео о том, как писать цифры — это около 10 минут, так что это сам по себе мини-урок, но он должен помочь детям двигаться, пока они учатся.

Теперь, когда вы заложили хорошую основу, читая и перечитывая «Числа монстров» и показывая видео несколько раз, пришло время дать студентам возможность попрактиковаться в малых группах.

Обучайте их в небольших группах

Пусть ваши ученики создадут свою собственную маленькую книжку с числами монстров и «ловят» монстров, заполняя страницы.

Мне нравится использовать это в качестве урока с моими математическими группами.

Мини-книжка начинается с цифры 5, так как именно здесь большинству киндеров нужно работать над счетом 1:1, когда они поступают в детский сад.

Вместе напишите несколько страниц, используя песнопения, которые вы выучили из книги и видео, и дайте учащимся немного попрактиковаться с карандашом и бумагой.

Отправьте книжки домой, чтобы родители могли видеть, над чем работают ученики.

Используйте числовые заклинания от руки в любое время, когда сможете

Теперь, когда ваши ученики могут практически процитировать все видео, пришло время использовать эти легко запоминающиеся заклинания в любое время, когда ваши ученики работают над написанием чисел или нуждаются в помощи. правильно писать цифры.

Всякий раз, когда учащиеся пытаются писать самостоятельно или вы пишете числа перед всем классом (например, в течение календарного времени) — используйте песнопения!

Подскажите учащимся несколько слов и дайте им произнести их вслух.

Отправьте набор песнопений домой родителям, чтобы они могли использовать их и дома!

Будем надеяться, что они заработают место в домашнем холодильнике на несколько недель. {wink}

Научите, что такое десятичная рамка

Помогите своим ученикам быстро развить чувство числа, помогая им видеть числа организованным образом.

Внедрение десятичной рамки — использование десятичной рамки было для меня критически важным для обучения так много математических навыков. Но позвольте мне облегчить вам задачу. Ваши дети уже подвергались воздействию одного из них!

«Хранитель монстров» из книги — десятикадровый.

Это отличная картинка для киндеров, и вы можете распечатать столько манипуляций с монстрами, сколько захотите.

Используйте 10 рамок всей группой или с небольшими группами по мере необходимости — они отлично подходят для изучения счета, прикосновения к объектам 1:1 и знакомства с расположением 10 рамок.

Научите их, как можно разместить любые фишки в десятичной рамке, чтобы предметы оставались организованными и облегчали счет.

В течение учебного года учащиеся узнают, что 5 сверху и 2 снизу равно 7 без необходимости считать каждый предмет. Это мощное средство , когда дело доходит до обучения ваших учеников субитизации больших чисел.

А пока научите их, что такое десятичная рамка, и почаще используйте ее для счета.

Связанный: Как выполнить манипуляции с десятью рамками

Использовать некоторые самостоятельные действия

Вы научили считать 1:1 и составлять числа всей группой, в малых группах, и теперь вам нужно дать учащимся много практики выполнения это самостоятельно с помощью самостоятельной деятельности.

Существует множество распечаток, простых игр и центральных заданий, которые помогут вам в этом.

Я сделал центр деятельности, которая не требует объяснения, как это сделать. Таким образом, я мог бы использовать его в начале учебного года, когда киндеры еще не очень самостоятельные работники.

Поскольку большинство учеников приходят в детский сад с общим представлением о том, как собрать пазл, я просто сделал десять фреймов числовыми пазлами, чтобы собрать все вместе.

Есть два разных вида головоломок, которые можно различать.

Все головоломки с числами из 3 частей имеют одинаковую форму, что означает, что они должны считать, если они хотят собрать головоломку правильно.

Пересмотрите то, что сработало

Если вы обучали учащихся счету и составлению чисел:

  • вся группа
  • в малых группах
  • с самостоятельными действиями

и повторением чисел каждый раз, когда вы может — ваши ученики будут на пути к использованию чисел, а не только к их изучению, в детском саду!

Так как рамки монстра номер десять более дружелюбны, их легко использовать снова, когда придет время ввести новый навык.

Как только они узнают числа до 10, пора углубиться в числа от 0 до 5.

Особое примечание: в детском саду очень важно сделать 5, так что не пропускайте, не игнорируйте и не приукрашивайте его!

И тогда вы готовы сосредоточиться на более глубоком копании в числах 5-10.

С тех пор, как я немного ненавижу рабочих листов, я сделал десять каркасных рабочих ковриков. Они отлично подходят для того, чтобы дать учащимся целевое число, которое нужно сделать разными способами.

Учащиеся практикуют различные способы составления чисел 5-10 При желании вы можете использовать рабочие коврики вместе с листами для записей.

Их было весело делать, и мне нравится идея использовать даже бинго-мазки для изображения предметов в десяти кадрах. В этом случае эти маляры создадут фантастических монстров.

Получить номера монстров

Если вам нравится идея использования этой печатной книги и любых ресурсов для печати, вы можете приобрести набор номеров монстров здесь:

Перейти к покупке

Вы получите:

— 35-страничную книгу «Числа монстров» для печати
— Аудиозапись (mp3) этой книги, прочитанной мной вслух
— Изображение этикетки компакт-диска
— 3 разных набора головоломки с числами (сопоставление чисел с десятью кадрами 1-10)
— Рукописный плакат 0-10 пения / родительский ресурс
— 11-страничная мини-книга «Числа моего монстра»
— 5 рабочих ковриков и 5 соответствующих листов для записи
— Манипулятивные изображения монстров и Десять кадров на тему монстров (Monster-Keepers)

Бонус: учите их с пластилином

Я люблю учить тактильно. Пластилин — основной материал в моем классе детского сада.

Итак, я позаботился о том, чтобы создать независимое занятие, в котором используется пластилин, чтобы практиковаться в составлении чисел и счете до десяти.

Эти бонусные   числовые формы из пластилина и рабочие коврики из десяти рамок по-прежнему изображают хранителя монстров и знакомых монстров, но учащиеся переходят к использованию простой десятирамочной рамки для подсчета своих шариков-монстров из пластилина.

Вы можете просто заламинировать или вложить страницы в пластиковый конверт, и все готово!

Перейти к покупке

Подведем итоги

Учим числа от 1 до 10 в детском саду, используя: всю группу, малую группу и самостоятельную деятельность.

Обязательно используйте книги, видеоролики, песнопения и заставляйте учащихся перемещать и манипулировать предметами или пластилином всякий раз, когда это возможно, чтобы сделать это на практике.

Таким образом, вы точно увеличите время, затрачиваемое на изучение чисел до десяти.

Я надеюсь, что видео- и печатные ресурсы Monster Numbers помогут быстро внедрить его в вашем классе детского сада.

Если вам нравится то, что я делаю здесь, на KindergartenWorks, то обязательно подпишитесь сегодня. Я с нетерпением жду возможности делиться с вами идеями еженедельно.

Больше математики

  • Создание 5 идей и игр для беглости
  • Изучение чисел для подростков в детском саду
  • Обучение: как рисовать рамку из десяти
  • 2 упражнения и 5 видео для обучения двухмерным фигурам

Учить японский — Урок 3: Цифры (1–10)

    1. Бесплатные уроки
    2. Учить японский

Рациональные числа пример: Рациональные числа: что это такое, свойства и примеры

Рациональные числа, зачем нужны рациональные числа, в чем их особенность

Какие числа называются рациональными

Рациональные числа — это числа, которые можно записать в виде отношения mn, где m — целое число, a n — натуральное число.

 

Чтобы понимать. что такое рационального числа, необходимо знать, что такое натуральные числа, целые числа, дроби (правильные \(\frac{2}{3}\), неправильные \(\frac{3}{2}\), бесконечные периодические 0,(4) и смешанные \(4 \frac{2}{3}\),).

 

Натуральные числа представляют собой числа, которые мы используем при счете (1, 2, 3…). Является ли натуральное число рациональным? Да. Рациональные числа включают в себя все натуральные числа (1, 2, 3, 4, и так далее), так как их можно записать в виде дроби, где числитель — это само число, а знаменатель равен 1. Например, число 3 можно записать как 3/1, что является дробью и, следовательно, рациональным числом.

 

 

В свою очередь целые числа – это натуральные числа, а также противоположные им и ноль (1, 2, 0, -1, -2…).

 

Дробью называется число, записанное в виде отношения mn (\(\frac{2}{3}\)), где m – целое, а n – натуральное число. Понятие дроби берет свое начало еще с древних времен, когда людям в торговых делах и бытовых нуждах требовалось определить часть какого-либо целого.

История рациональных чисел

История рациональных чисел началась с возникновением финансово-кредитных отношений между людьми. Чтобы в численном виде представить задолженность человека, нужно было записывать числа, противоположные натуральным. Так появились отрицательные числа (-3, -167). А для того, чтобы записывать часть целого (например, возвращение долга частями), стали использовать дроби. Именно поэтому математикам необходимо было как-то объединить такие характерные числа, дать им общее название. Так появились рациональные числа (от латинского ratio – «отношение»).

 

Для усвоения материала приведем пример задания на определение рациональных чисел из ряда чисел.

 

Рациональные числа: пример с ответом

 

Задача. Даны числа: -34, 480, 0,11, \(\frac{1}{2}\), 8. Какие из них рациональные?

 

Решение.

Рассмотрим отдельно каждое из заданных чисел:

  • Число -34 – целое число;
  • Число 480 – натуральное число;
  • Число 0,11 =  \(\frac{11}{100} \) – десятичная дробь;
  • Число  \(\frac{1}{2}\) — правильная дробь;
  • Число 8 – натуральное число.

Каждое из заданных чисел можно представить в виде дроби с целым и натуральным знаменателем. Значит, все 5 чисел являются рациональными числами.

Ответ: все числа являются рациональными.

 

Чтобы хорошо понимать специфику рациональных чисел, нужно усвоить два правила:

  • Сумма рациональных чисел является рациональным числом.
  • Произведение рациональных чисел является рациональным числом.

Теория в математике очень важна. Умение отличать натуральные числа от целых, рациональные от иррациональных поможет Вам не запутаться и не ошибиться в практике. Удачи!

 

Часто задаваемые вопросы

Рациональные числа — что это такое (примеры)

Обновлено 20 июля 2021 Просмотров: 178 177 Автор: Дмитрий Петров

Здравствуйте, уважаемые читатели блога KtoNaNovenkogo.ru. Сегодня мы вновь поговорим о математических терминах.

И на этот раз расскажем все о РАЦИОНАЛЬНЫХ ЧИСЛАХ. Они обязательно входят в школьную программу, и дети начинают изучать их в 6 классе.

Само слово «рациональный» знакомо многим. И под ним подразумевается нечто «логичное» и «правильное». На деле так и есть.


Рациональные числа — это …

Термин имеет латинские корни, и в переводе «ratio» означает «число», «расчет», «разум», «рассуждение» и «нумерация». Но есть и другие переводы – «дробь» и «деление».

РАЦИОНАЛЬНОЕ ЧИСЛО – любое число, которое можно показать в виде дроби a/b. Здесь а – целое число, а b – натуральное.

Стоит напомнить, что:

  1. Целые числа – это все возможные числа, как отрицательные, так и положительные. И к ним же относится ноль. Главное условие – они не должны быть дробными. То есть -15, 0 и +256 можно назвать целыми числами, а 2,5 или -3,78 – нет.
  2. Натуральные числа – это числа, которые используются при счете, то есть они имеют «натуральное происхождение». Это ряд из 1, 2, 3, 4, 5 и так далее до бесконечности. А вот ноль и отрицательные числа, как и дробные – к натуральным не относятся.

И если применить эти определения, то мы можем сказать, что:

РАЦИОНАЛЬНОЕ ЧИСЛО – это вообще все возможные числа, кроме бесконечных непериодических десятичных дробей. Среди них натуральные и целые числа, обыкновенные и конечные десятичные дроби, а также бесконечные периодические дроби.

История изучения рациональных чисел

Точно неизвестно, когда люди начали изучать дроби. Есть мнение, что много тысяч лет назад. И началось все с банального дележа. Например, кому-то нужно было разделить добычу, но на равные части это не получалось сделать. Зато получалось сколько-то целых, и сколько-то в довесок.

Скорее всего, дроби изучали и в Древнем Египте, и в Древней Греции. Тогдашние математики далеко продвинулись в науке. И трудно предположить, что эта тема осталась ими не изучена. Хотя, к сожалению, ни в одних трудах так и не было найдено конкретных указаний на рациональные числа.

А вот официально считается, что понятие десятичной дроби появилось в Европе в 1585 году. Этот математический термин в своих трудах увековечил голландский инженер и математик Симон Стевин.

До занятия наукой, он был обыкновенным купцом. И скорее всего, именно в торговых делах часто сталкивался с дробными числами. Что потом и описал в своей книге «Десятая».

В ней Стевин не только объяснял полезность десятичных дробей, но и всячески пропагандировал их использование. Например, в системе мер для точного определения величины чего-либо.

Разновидности рациональных чисел

Мы уже написали, что под понятия рациональные числа подпадают практически все возможные варианты. Теперь рассмотрим более подробно существующие варианты:

  1. Натуральные числа. Любое число с 1 и до бесконечности можно представить в виде дроби. Достаточно вспомнить простое математическое правило. Если поделить число на единицу, то получится то же самое число. Например, 5 = 5/1, 27 = 27/1, 136 = 136/1 и так далее.
  2. Целые числа. Точно такая же логика, как в случае с натуральными числами, действует и тут. Отрицательные числа также можно представить в виде дроби с делением на единицу. И точно также будет в отношении нуля. Например, -356 = -356/1, -3 = -3/1, 0 = 0/1 и так далее.
  3. Обыкновенные дроби. Это напрямую говорится в определении рациональных чисел. Например, 6/11, 2/5, -3/10 и так далее.
  4. Бесконечные периодические дроби. Это числа, у которых после запятой бесконечное множество знаков и их последовательность повторяется. Самые простые примеры 1/3, 5/6 и так далее.
  5. Конечные десятичные дроби. Это числа, которые можно записать двумя разными вариантами, и у которых вполне конкретное количество знаков после запятой. Самый простой пример – половина. Ее можно обозначить дробью 0,5 или дробью ½.

Все числа, которые входят в понятие рациональных, называются МНОЖЕСТВОМ рациональных чисел. В математике его принято обозначать латинской буквой Q.

А графически это можно изобразить вот так:

Свойства рациональных чисел

Рациональные числа подчиняются всем главным законам математики:

  1. А + В = В + А
  2. А + (В + С) = (А + В) + С
  3. А + 0 = А
  4. А + (-А) = 0
  5. А * В = В * А
  6. А * 1 = А
  7. А * 0 = 0
  8. (А + В) * С = А * С + В * С
  9. (А – В) * С = А * С – В * С

Ради интереса можете попробовать подставить вместо букв любые числа и убедиться, что эти законы верны.

Вместо заключения

Раз есть в математике рациональные числа, значит, должны быть и им противоположные. Так и есть – они называются иррациональными. Это числа, которые нельзя записать в виде обычной дроби.

К таким числам относится математическая константа «пи». Многие знают, что она равна 3,14 и бесконечное количество знаков после запятой, причем их последовательность никогда не повторяется.

Также к иррациональным числам относится много корней. Это касается тех, у кого в результате не получается целого числа. Самый простой пример – корень из 2. Но это уже тема для другой статьи.

Удачи вам! До скорых встреч на страницах блога KtoNaNovenkogo.ru

Эта статья относится к рубрикам:

  • Математика

рациональных чисел — Определение | Примеры

Рациональные числа представлены в виде p/q, где p и q могут быть любыми целыми числами и q ≠ 0. Это означает, что рациональные числа включают натуральные числа, целые числа, целые числа, дроби целых чисел и десятичные дроби ( завершающие десятичные дроби и повторяющиеся десятичные дроби). Давайте узнаем больше о рациональных числах и о том, как идентифицировать рациональные числа и примеров рациональных чисел в этом уроке.

1. Что такое рациональные числа?
2. Типы рациональных чисел
3. Как определить рациональные числа?
4. Рациональные и иррациональные числа
5. Часто задаваемые вопросы о рациональных числах

Что такое рациональные числа?

Слово «рациональный» произошло от слова «рацио». Итак, рациональные числа хорошо связаны с концепцией дробей, которые представляют отношения. Другими словами, если число может быть выражено в виде дроби, где и числитель, и знаменатель являются целыми числами, число является рациональным числом.

Определение рациональных чисел

Рациональное число — это число, имеющее форму p/q, где p и q — целые числа, а q не равно 0.

Набор рациональных чисел

Множество рациональных чисел обозначается Q. Следует отметить, что рациональные числа включают в себя натуральные числа, целые числа, целые числа и десятичные дроби.

Обратите внимание на следующий рисунок, определяющий рациональное число.

Примеры рациональных чисел

Если число может быть представлено в виде дроби, где и числитель, и знаменатель являются целыми числами, это число является рациональным числом. Ниже приведены некоторые примеры рациональных чисел.

  • 56 (что можно записать как 56/1)
  • 0 (другая форма 0/1)
  • 1/2
  • √16, что равно 4
  • -3/4
  • 0,3 или 3/10
  • -0,7 или -7/10
  • 0,141414… или 14/99

Типы рациональных чисел

Различные типа рациональных чисел даны следующим образом.

  • Целые числа, такие как -2, 0, 3 и т. д., являются рациональными числами.
  • Дроби, числители и знаменатели которых являются целыми числами, такими как 3/7, -6/5 и т. д., являются рациональными числами.
  • Завершающие десятичные дроби, такие как 0,35, 0,7116, 0,9768 и т. д., являются рациональными числами.
  • Неконечные десятичные числа с некоторыми повторяющимися шаблонами (после запятой), например 0,333…, 0,141414… и т. д., являются рациональными числами. Они широко известны как непрерывающиеся повторяющиеся десятичные дроби.

Как определить рациональные числа?

Рациональные числа легко идентифицировать с помощью следующих характеристик.

  • Все целые числа, целые числа, натуральные числа и дроби с целыми числами являются рациональными числами.
  • Если десятичная форма числа завершается или повторяется, как в случае 5,6 или 2,141414, мы знаем, что это рациональные числа.
  • Если десятичные дроби кажутся бесконечными или неповторяющимися, то такие числа называются иррациональными. Как и в случае √5, равного 2,2360679.77499789696409173… иррациональное число.
  • Другой способ определить рациональные числа — посмотреть, может ли число быть выражено в форме p/q, где p и q — целые числа, а q не равно 0.

Пример: Является ли 0,923076923076923076923076923076… рациональным числом?

Решение: Данное число имеет набор десятичных знаков 923076, который повторяется и повторяется непрерывно. Таким образом, это рациональное число.

Возьмем другой пример.

Пример: Является ли √2 рациональным числом?

Решение: Если мы запишем десятичное значение √2, мы получим √2 = 1,414213562…, которое является непрерывающимся и неповторяющимся десятичным числом. Следовательно, это не рациональное число. Это иррациональное число.

Рациональные числа в десятичной форме

Рациональные числа также могут быть представлены в десятичной форме. Вы знаете, что 1,1 — рациональное число? Да, потому что 1,1 можно записать как 1,1 = 11/10. Теперь давайте поговорим о некончающихся десятичных дробях, таких как 0,333… Поскольку 0,333… можно записать как 1/3, значит, это рациональное число. Следовательно, неконечные десятичные числа, имеющие повторяющиеся числа после запятой, также являются рациональными числами.

Является ли 0 рациональным числом?

Да, 0 — рациональное число, поскольку его можно записать в виде дроби целых чисел, например 0/1, 0/-2,… и т. д. Другими словами, 0/5 = 0, 0/-2 = 0 , 0/1 = 0 и так далее.

Список рациональных чисел

Из приведенной выше информации становится ясно, что существует бесконечное количество рациональных чисел. Следовательно, невозможно определить весь список рациональных чисел. Однако некоторые рациональные числа могут быть перечислены как 3, 4,57, 3/4, 0, -7 и так далее. Это показывает, что все натуральные числа, целые числа, целые числа, дроби и десятичные числа (завершающие десятичные числа и повторяющиеся десятичные числа) считаются рациональными числами.

Сложение и вычитание рациональных чисел

Для сложения и вычитания рациональных чисел мы используем те же самые правила сложения и вычитания целых чисел. Давайте разберемся в этом с помощью примера.

Пример: Решить 1/2 — (-2/3)

Решение: Решим это, используя следующие шаги:

  • Шаг 1: Поскольку мы упрощаем 1/2 — (-2 /3), мы будем следовать правилу сложения и вычитания чисел, согласно которому факт вычитания может измениться на факт сложения, а знак вычитаемого изменится на противоположный. Это сделает его 1/2 + 2/3
  • Шаг 2: Теперь нам нужно сложить эти дроби 1/2 + 2/3
  • Шаг 3: Используя правила сложения дробей, мы преобразуем данные дроби в подобные дроби, чтобы получить общие знаменатели, чтобы их стало легче складывать. Для этого нам нужно найти НОК знаменателей 2 и 3, который равен 6. Затем мы преобразуем дроби в соответствующие им эквивалентные дроби, что сделает их 3/6 + 4/6. Это даст сумму 7/6, которую можно записать в виде смешанной дроби \(1\dfrac{1}{6}\)

Умножение и деление рациональных чисел

Умножение и деление рациональных чисел можно выполнять так же, как дроби. Чтобы умножить любые два рациональных числа, мы умножаем их числители и знаменатели по отдельности и упрощаем полученную дробь. Давайте разберемся в этом с помощью примера.

Пример: Умножить 3/5 × -2/7

Решение: Решим это, выполнив следующие шаги:

  • Шаг 1: Чтобы умножить 3/5 × (-2)/7, мы сначала умножим числители, а затем умножим знаменатели.
  • Шаг 2: В этом случае, когда мы умножаем числители, это будет 3 × (-2) = -6.
  • Шаг 3: Когда мы умножим знаменатели, получится 5 × 7 = 35. Следовательно, произведение будет -6/35.

Когда нам нужно разделить любые две дроби, мы умножаем первую дробь (которая является делимым) на величину, обратную второй дроби (которая является делителем). Давайте разберемся в этом с помощью примера.

Пример: Разделить 3/5 ÷ 2/7

Решение: Решим это, используя следующие шаги:

  • Шаг 1: Чтобы разделить 3/5 ÷ 2/7, мы сначала напишет обратную величину второй дроби. Это составит 3/5 × 7/2
  • .
  • Шаг 2: Теперь мы умножим числители Это будет 3 × 7 = 21.
  • Шаг 3: Затем умножим знаменатели, будет 5 × 2 = 10. Следовательно, произведение будет 21/10 или \(2\dfrac{1}{10}\)

Рациональные и иррациональные числа

Числа, которые НЕ являются рациональными числами, называются иррациональными числами. Множество иррациональных чисел представлено Q´. Разницу между рациональными и иррациональными числами можно понять из следующего рисунка и таблицы, приведенных ниже.

Рациональные числа Иррациональные числа

Это числа, которые можно представить в виде целых чисел.

Примеры: 1/2, 0,75, -31/5 и т. д.

Это числа, которые нельзя представить в виде целых чисел.

Примеры: √5, π и т. д.

Завершающие десятичные дроби. Они НИКОГДА не завершают десятичные дроби, которые не имеют точного значения.

Они могут быть неконечными десятичными знаками с повторяющимися шаблонами десятичных знаков или повторяющимися десятичными знаками.

Пример: 1.414, 414, 414 … содержит повторяющиеся десятичные дроби, где 414 повторяется.

Они должны быть неконечными десятичными знаками без повторяющихся шаблонов десятичных знаков.

Пример: √5 = 2,236067977499789696409173…. не имеет повторяющихся десятичных знаков

Множество рациональных чисел содержит полностью натуральные числа, все целые числа и все целые числа. Набор иррациональных чисел является отдельным набором и НЕ содержит никаких других наборов чисел.

Советы и рекомендации по рациональным числам

  • Рациональные числа включают дроби и любые числа, которые могут быть представлены дробями.
  • Натуральные числа, целые числа, целые числа, дроби целых чисел и конечные десятичные дроби являются рациональными числами.
  • Неконечные десятичные дроби с повторяющимися шаблонами десятичных дробей, то есть повторяющиеся десятичные дроби, также являются рациональными числами.

☛ Статьи по теме

  • Простые числа
  • Составные номера
  • Четные числа
  • Нечетные числа
  • Реальные числа
  • Натуральные числа
  • Целые числа
  • Иррациональные числа
  • Подсчет чисел
  • Кардинальные числа
  • Четные и нечетные числа

Cuemath — одна из ведущих мировых обучающих платформ по математике, которая предлагает онлайн-уроки по математике в режиме реального времени один на один для классов K-12. Наша миссия — изменить то, как дети изучают математику, чтобы помочь им преуспеть в школе и на конкурсных экзаменах. Наши опытные преподаватели проводят 2 или более живых занятий в неделю в темпе, соответствующем потребностям ребенка в обучении.

 

Примеры рациональных чисел

  1. Пример 1: Определите рациональные числа среди следующих: √4, √3, √5/2, -4/5, π, 1,41421356237309504…..

    Решение:

    Рациональное число, когда упрощенный должен быть либо конечным десятичным числом, либо неконечным десятичным числом с повторяющимся шаблоном десятичных знаков. Следовательно, рациональные числа среди заданных чисел равны √4 (в результате получается 2) и -4/5.

  2. Пример 2: Укажите истинное или ложное значение относительно рациональных чисел.

    а.) Каждое целое является рациональным числом.

    б.) Каждое рациональное число является целым числом.

    c.) Каждое рациональное число является целым числом.

    г.) Каждое действительное число является рациональным числом.

    д.) Каждое целое число является рациональным числом.

    Решение:

    а.) Верно, каждое целое число является рациональным числом.

    б.) Неверно, каждое рациональное число не является целым числом. Это может быть часть целых чисел или десятичная дробь. К рациональным числам относятся натуральные числа, целые числа, целые числа и дроби целых чисел.

    c.) Ложь, каждое рациональное число не является целым числом, потому что рациональные числа включают дроби и конечные десятичные дроби, а целые числа включают только отрицательные числа, положительные числа и 0

    d.) Нет, каждое действительное число не является рациональным числом, потому что действительное числа включают в себя и иррациональные числа.

    д.) Верно, каждое целое число является рациональным числом.

  3. Пример 3:

    Запишите следующее рациональное число в десятичной форме: 1/2

    Решение:

    Рациональное число 1/2 можно преобразовать в десятичное число, разделив числитель на знаменатель. Нам нужно разделить 1 на 2 и мы получим 0,5

перейти к слайдуперейти к слайдуперейти к слайду

Есть вопросы по основным математическим понятиям?

Станьте чемпионом по решению проблем, используя логику, а не правила. Узнайте, почему стоит математика, с нашими сертифицированными экспертами

Забронируйте бесплатный пробный урок

Практические вопросы по рациональным числам

 

перейти к слайдуперейти к слайду

Часто задаваемые вопросы о рациональных числах

Что такое рациональное число в математике?

Любое число в форме p/q, где p и q — целые числа, а q не равно 0, является рациональным числом . Примеры рациональных чисел: 1/2, -3/4, 0,3 или 3/10.

Как определить рациональное число?

Чтобы определить, является ли данное число рациональным или иррациональным, нам нужно преобразовать его в десятичную форму. Если десятичная дробь заканчивается или не заканчивается повторяющимися десятичными знаками, то число является рациональным. В противном случае это нерационально. Например, 1,414, 414, 414… имеет повторяющиеся шаблоны десятичных знаков, где повторяется 414. Значит, это рациональное число. С другой стороны, если мы возьмем пример √5 = 2,236067977499789696409173…., мы видим, что оно не завершается и не имеет повторяющихся шаблонов десятичных знаков. Значит, это иррациональное число. Другой способ определить рациональное число — проверить, является ли оно натуральным числом, целым числом, целым числом или дробью целых чисел. Если это одно из них, то это рациональное число.

Что такое завершающие рациональные числа?

Завершающими рациональными числами являются те десятичные числа, которые заканчиваются после определенного количества знаков после запятой. Например, 1,5, 3,4, 0,25 и т. д. являются конечными числами. Все конечные числа являются рациональными числами, поскольку их легко записать в виде p/q.

В чем разница между рациональными и иррациональными числами?

Рациональное число — это число, десятичная форма которого является конечной или повторяющейся по своей природе. Например, 2,67 и 5,666 … Принимая во внимание, что иррациональные числа — это те числа, десятичная форма которых не заканчивается и не повторяется после определенного количества знаков после запятой. Например, √5 = 2,236067977499789696409173…. не имеет повторяющихся десятичных знаков и не заканчивается, поэтому это иррациональное число.

Что такое иррациональные числа?

Иррациональные числа — это те, которые не могут быть представлены целыми числами в форме p/q. Множество иррациональных чисел обозначается Q´. Несколько примеров иррациональных чисел: √2, √5 и так далее. Их десятичные формы не прекращаются и не повторяются.

Является ли 0 рациональным числом?

Да, 0 — рациональное число, так как мы можем записать его как 0/1, где 0 и 1 — целые числа, а знаменатель не равен 0.

Какое число нужно добавить к числу Пи, чтобы получить рациональное число?

Если мы добавим — π к π, мы получим, — π + π = 0. Эта сумма является рациональным числом. Следовательно, добавляя — π к π, мы получаем рациональное число.

Как складывать рациональные числа?

Для сложения рациональных чисел используются те же правила сложения целых чисел. Давайте разберемся в этом с помощью примера. Если нам нужно добавить 4,53 + 2,31, мы начнем добавлять с правой стороны. Расставив числа в столбцах в соответствии с их разрядностью, начнем с сотого места. Итак, 3 + 1 = 4. Переходя к десятому столбцу, мы добавляем 5 + 3 = 8. Затем мы добавляем цифры в столбце единиц, то есть 4 + 2 = 6. Таким образом, мы получаем сумму как 6,84

Как делить рациональные числа?

Чтобы разделить рациональные числа, мы используем обычные правила деления целых чисел, дробей или десятичных дробей, в зависимости от обстоятельств. Например, разделим дроби 7/5 ÷ 1/5. Чтобы разделить 7/5 ÷ 1/5, сначала напишем обратную величину второй дроби. Это сделает его 7/5 × 5/1. Теперь мы умножим числители. Это будет 7 × 5 = 35. Затем мы умножим знаменатели, это будет 5 × 1 = 5. Следовательно, произведение будет 35/5, что равно 7.

Как умножать рациональные числа?

Рациональные числа можно умножать, используя правила умножения. Например, давайте умножим 43 × 10, мы получим произведение как 430.

Откуда начинаются рациональные числа?

Рациональные числа не начинаются с какого-либо определенного числа. Они включают в себя все натуральные числа, целые числа и целые числа, а также конечные десятичные дроби и дроби, которые могут быть представлены в виде p/q, где q не равно 0.

В чем разница между дробями и рациональными числами?

Существует небольшая разница между дробями и рациональными числами, которая приведена ниже:

  • Любое число, которое может быть представлено в виде p/q, где ‘p’ и ‘q’ — целых числа и q не должны быть равно 0. Например, 1/2, -3/4, 7/9, -100/21 и т.д.
  • Дроби могут быть записаны в виде p/q, но ‘p’ и ‘q’ должны быть целыми числами . Например, 3/4, 1/9, и так далее. Это означает, что значения p и q не могут быть отрицательными.

Все ли целые числа рациональны?

Да, все целые числа являются рациональными числами, поскольку рациональные числа включают в себя все натуральные числа, целые числа, целые числа, конечные десятичные дроби и дроби в форме p/q, где «p» и «q» — это целых числа , а q не должно быть равным 0.

Все ли целые числа рациональны?

Да, все целые числа являются рациональными числами, поскольку рациональные числа включают в себя все натуральные числа, целые числа, целые числа, конечные десятичные дроби и дроби в форме p/q, где ‘p’ и ‘q’ равны целые числа и q не должны быть равны 0.

Скачать БЕСПЛАТНО учебные материалы

Скачать бесплатно рабочие листы для 9 класса

Что такое рациональные числа? Определение, свойства, примеры

Основой овладения любым предметом является знание его лексики. То, как мы комбинируем буквы алфавита, чтобы составить слово, а затем упорядочиваем слова в предложениях, чтобы передать наши мысли, является для нас естественным инстинктом только потому, что мы знаем английский алфавит. Итак, каким должен быть 101 балл, чтобы сделать математику естественным инстинктом? Знание его рудиментарной единицы: чисел. Мы учим считать, используя целые числа, но одним из наиболее важных аспектов математической системы счисления являются рациональные числа, или, проще говоря, числа между числами.

Что такое рациональные числа?

Рациональные числа — это числа, которые можно записать в виде pq, где p и q — целые числа, а q≠0. Отличие рациональных чисел от дробей заключается в том, что у дробей не может быть отрицательных числителя или знаменателя. Следовательно, числитель и знаменатель дроби — целые числа (знаменатель ≠ 0), тогда как в случае рациональных чисел числитель и знаменатель — целые числа.

Родственные игры

Примеры рациональных чисел

  • $\frac{4}{5}$, $\frac{-10}{15}$, $\frac{9}{-17}$, $\frac{-2}{-7}$
  • Ноль является рациональным числом, поскольку его можно записать в виде  $\frac{0}{10}$, $\frac{0}{2}$, $\frac{0}{-15}$, $\ frac{0}{27}$ и т.  д. 

Итак, ноль можно представить в виде дроби с ненулевым знаменателем.

  • Каждое натуральное число является рациональным числом.

1 = $\frac{1}{1}$, 5 = $\frac{5}{1}$, 8 = $\frac{8}{1}$

Итак, каждое натуральное число можно выразить как дробь с 1 в знаменателе.

  • Каждое целое число является рациональным числом.

-7 = $\frac{-7}{1}$, -13 = $\frac{-13}{1}$ 

Итак, каждое целое число можно представить в виде дроби с 1 в знаменателе.

  • Каждая дробь является рациональным числом.

$\frac{2}{3}$, $\frac{18}{7}$, $\frac{19}{11}$

Где числитель и знаменатель являются целыми числами, а знаменатель не равен до нуля.

  • Положительные рациональные числа

Рациональное число положительно, если его числитель и знаменатель имеют одинаковые знаки (либо оба положительные, либо оба отрицательные).

$\frac{-5}{-6}$, $\frac{7}{12}$, $\frac{-19}{-12}$, $\frac{42}{85}$ 

  • Отрицательные рациональные числа

Рациональное число отрицательно, если его числитель и знаменатель имеют противоположные знаки (один из них положительный, а другой отрицательный).

$\frac{-32}{43}$, $\frac{27}{-63}$, $\frac{-12}{14}$, $\frac{9}{-16}$

  • Десятичные числа: 0,45, 0,5446, 0,01864. Поскольку десятичные дроби можно записать в виде дробей со знаменателем 10, 100, 1000… десятичные числа также считаются рациональными числами.
Связанные рабочие листы

Непримеры рационального числа

  • Квадратный корень, кубический корень и т. д. натуральных чисел, если их точное значение не может быть определено.

$\sqrt{2}$, $\sqrt{3}$, $\sqrt[2]{8}$, $\sqrt[3]{81}$, $\sqrt[4]{27}$

  • $\sqrt{3}$, – $\sqrt{5}$, 8 + $\sqrt{10}$, $\sqrt{2}$ + $\sqrt{3}$

Стандартная форма рациональных чисел

Говорят, что рациональное число $\frac{p}{q}$ находится в стандартной форме, если числитель и знаменатель (p и q) не имеют общего делителя (или множитель), кроме 1 и знаменатель « q » положительный.

Как преобразовать рациональное число в стандартную форму:

  • Рассмотрим рациональное число $\frac{18}{–24}$;

Давайте посмотрим, как мы можем преобразовать это рациональное число в его стандартную форму.

Шаг 1: Убедитесь, что знаменатель положительный. Если нет, умножьте числитель и знаменатель на «-1», чтобы изменить знак.

Шаг 2: Найдите все общие делители числителя и знаменателя.
            Здесь числа 18 и 24 имеют общие делители 1, 2, 3, 6.

Шаг 3: Разделите числитель и знаменатель на их наибольший общий делитель. Вот так:

Шаг 3 не требуется! Как видите, $\frac{–3}{4}$  – стандартное рациональное число и стандартная форма $\frac{18}{–24}$.

  • Рассмотрим десятичное число 0,5

Шаг 1: Чтобы преобразовать десятичное число в стандартное рациональное число, мы сначала преобразуем его в рациональное число.

Шаг 2: Теперь выполните те же шаги для преобразования рационального числа в его стандартную форму, как определено выше, и мы получим

Операции над рациональными числами

Сложение рациональных чисел:

  • числа равны:

Сложите числители и оставьте знаменатели прежними.

Например, мы можем сложить два рациональных числа 2/8 и ⅜ следующим образом:

  • Когда знаменатели данных чисел не равны:

Например, чтобы сложить рациональные числа ⅖ и ¾ 

Шаг 1: Найдите НОК знаменателя заданных рациональных чисел. Здесь НОК 4 и 5 равен 20.

Шаг 2: Преобразуйте каждое рациональное число в эквивалентное рациональное число с НОК в качестве нового знаменателя.

Шаг 3: Для этих новых рациональных чисел сложите числители и оставьте прежними знаменатели, и это будет окончательный ответ.

Следовательно, ⅖ + ¾ = 23/20.

Вычитание рациональных чисел:

  • Когда знаменатели данных чисел равны:

Вычтите числители и оставьте знаменатель прежним.

Например:

  • Когда знаменатели данных чисел не равны:

Например, чтобы вычесть ⅓ из ½ 

Шаг 1: Найдите НОК знаменателя заданных рациональных чисел. В этом случае НОК 2 и 3 равен 6.

Шаг 2: Преобразуйте каждое рациональное число в эквивалентное рациональное число с НОК в качестве нового знаменателя.

Шаг 3: Выполните шаги вычитания рациональных чисел с одинаковыми знаменателями.

Следовательно, ½ – ⅓ = ⅙ 

Умножение рациональных чисел:

Это, пожалуй, самая простая операция над рациональными числами.

Чтобы умножить два рациональных числа, просто умножьте числитель и знаменатель первого рационального числа на числитель и знаменатель второго рационального числа соответственно.

Например, чтобы умножить рациональные числа ⅖ и ¾, 

Решаемые примеры 

Пример 1: Выразите $\frac{25}{40}$ в стандартной форме.

Решение:

Наибольший делитель 25 и 40 = 5

Пример 2. Сложите:  $\frac{2}{11}+\frac{7}{11}$ .

Решение:

$\frac{2}{11}+\frac{7}{11}= \frac{2+7}{11}=\frac{9}{11}$ 

Пример 3. Вычтите $\frac{5}{7}$ из $\frac{9}{7}$ .

Решение:

$\frac{9}{7}–\frac{5}{7}= \frac{9–5}{7}=\frac{2}{7}$

Практические задачи

1

Число 398 можно выразить рациональным числом следующим образом:

398/1

0/398

1/398

398 не является рациональным числом.

Правильный ответ: 398/1
Целые числа можно представить в виде рациональных чисел со знаменателем 1.
398 = $\frac{398}{1}$

2

Число 0,7 можно представить в виде рационального числа как:

$\frac{10}{7}$

$\frac{7}{10}$

$\frac{0.7}{10}$

0,7 не является рациональным числом

Правильный ответ составляет: $\frac{7}{10}$
Удалив десятичную точку, мы можем представить $\frac{7}{10}$ как 0,7

3

Стандартная форма числа $\frac{–20}{15}$:

1

$\ frac{2}{5}$

$\frac{-10}{3}$

$\frac{3}{-10}$

Правильный ответ: $\frac{-10}{3} $
Наибольший общий делитель 20 и 15 равен 5; поэтому деление числителя и знаменателя в -20/15 на 5 дает нам -10/3.

График модуль икс: Функция y = |x| — урок. Алгебра, 8 класс.

Главный график отключения горячей воды в Магнитогорске 2023 с адресами

Первые отключения горячей воды пройдут в Магнитогорске со дня на день. Самый актуальный график на 2023 год выложил “Водоканал”. Кто следующий на очереди? Расскажет “МР-инфо”.

Отключения горячей воды связаны с опрессовкой – испытанием и ремонтом тепловых сетей перед следующим отопительным сезоном. Эта процедура в Магнитогорске длится 10 дней, а во многих других городах – две недели.

Испытания проводят поэтапно, район за районом. В этом году первые отключения пройдут на Левом берегу. Горячей воды не будет в квартирах уже 15 мая, а включат её 25 мая. Это коснётся потребителей Центральной котельной от бойлерных по улице Маяковского в домах №№ 11, 19, 19/1, 21а, 23а, 52, 56, 60, 64; по улице Фрунзе в домах №№ 3, 9,15; по улице Ш. Руставели в домах №№ 2, 9; по улице Чкалова в домах №№ 8/3, 14а; по улице Чкалова в домах №№ 8/3, 14а; по улице Ржевского в домах №№ 1, 6, 7; по улице Пушкина в домах №№ 28, 32, 36; по улице Пушкина в доме №8а; по улице Трамвайная в домах №№ 28, 30; по улице Рубинштейна в доме №5; по улице Чайковского в домах №№ 55, 59, 61, 63, 76; по улице Кирова в доме №63; по улице Красноармейская в доме №5.

Отключения пройдут на центральном тепловом пункте в посёлке Самстрой и ЯВ–48/18, а также в различных учреждениях: детская больница (улица Рубинштейна, 4), Народный суд (улица Маяковского, 23), Дворец культуры (проспект Пушкина, 19), МП трест «Водоканал» (улица Маяковского, 15а и улица Полтавская, 6), спортивная школа №6 (проспект Пушкина, 17/1), администрация Орджоникидзевского района (улица Маяковского, 19/3), МП «Маггортранс» (улица Чкалова, 29), ООО «Новая энергия» (улица Елькина, 14), ООО «Урал Модуль» (улица Молодежная, 19), горбольница №1 (улица Чкалова, 44), школа №16 (улица Чкалова, 60), Магнитогорский педагогический колледж (улица Полевая, 2/а), детский сад №166 (улица Фрунзе, 54), школа №17 (переулок Тихвинский, 2), база МП трест «Теплофикация» (улица Казахская, 3а), 2 отряд федеральной противопожарной службы (улица Лермонтова,18/а), школа №31 (улица Рубинштейна, 2), школа-интернат №2 (улица Лесная, 6), детский сад №179 «а», «б» (улица Рубинштейна, 7,9), «Станция скорой медицинской помощи» (улица Шоссейная, 1), Центр коммунального сервиса (улица Шоссейная, 11), ООО «МБ ТРАНС» (улица Шоссейная, 11), Евстегнеев С. И. (улица Шоссейная, 6), ООО «Урал-Авто» (улица Красноармейская, 41/1), ООО «Некеров-Тур» (улица Профсоюзная, 10), ООО «МТПК» (улица Шоссейная, 40), ООО «Делин» (улица Шоссейная, 2), АО ММУ «Промвентиляция» (улица Красноармейская, 41а), отдел полиции №11 (улица Рубинштейна, 6).

С 15 мая по 9 июня останутся без ГСВ потребители бойлерных: проммилиция (улица Электросети, 8), кадровый центр «Персонал» (улица Кирова, 84), РКЦ (улица Кирова, 78), ПАО «ММК» (улица Кирова, 72, 76), ЦТД (проспект Пушкина, 2), проектный отдел (проспект Пушкина, 6 ), МП трест «Водоканал» (улица Электросети, 3), АТП ПАО «ММК» (улица Дежнева, 16).

Почти одновременно с указанными кварталами Левого берега отключения пройдут и в Ленинском районе – с 16 по 26 мая. Останутся без ГВС кварталы 1, 1а, 2, 2а, 3, 3а, 3б, 4а, 4б, 4в, 4г, 5а, 5б, 5в, 6а, 7а, 7б, 7/4, 8, 9, 11, 12, 12а, 13, 14, 14а, 14б, 15а, 15б, 16, 17, 17а, 18, 18а, 19а, 19б, 20а, 20б, 21а, 21б, 22а, 22б, 23, 23а, 27, 28, 29, 30, 36, 39, 50, 51, 52, 53, 53а, 57, 59, 60, 61, 62, 63, 64, 65, 66, 67, 67а, 68, 68а, 69, 80, 82, 83, 93а, 94, а также коттеджи и жилые дома по улице Суворова №№ 31, 54, гостиничный комплекс (улица Гагарина, 32), школа №15 (улица Суворова, 76а), ООО «Лифт» (улица Суворова, 33), МУП «МГС» (улица Суворова, 52), КВД (улица Тимирязева, 31), МП трест «Водоканал» (улица Советская, 30), Медсанчасть (улица Набережная, 18), спорт-клуб «Металлург-Магнитогорск», аквапарк (улица Набережная, 9), ДК им. Орджоникидзе (улица Набережная, 1), онкологический диспансер (улица Герцена, 4), ТДК «Альфа-Центр» (улица Герцена, 6), театр оперы и балета (проспект Ленина,16), МУ «СШОР «Динамо» (улица Н.Шишки, 31), род.дом №3 (улица Н.Шишки, 30), противотуберкулезный диспансер (улица Бурденко, 2), ТЦ «Локомотив» (улица Вокзальная, 35), вокзал (улица Вокзальная, 39).

С 17 по 26 мая отключение затронет потребителей котельной «Западная» и котельной 71 квартала, а с 19 по 29 мая – потребителей котельной «Восточная» отключения пройдут с 19 мая по 29 мая.

Кто следующий на очереди – смотрите во вложении.

Похожие материалы

Фото  Динара ВОРОНЦОВА (архив)

 

Оставьте ВАШ КОММЕНТАРИЙ на новость через любимую социальную сеть:

Метки (Тэги): ВодоканалГрафик Отключения ВодыОтключение Горячей Воды

Get-MgDirectoryFederationConfiguration (Microsoft.Graph.Identity.DirectoryManagement) | Microsoft Узнайте

Твиттер LinkedIn Фейсбук Электронная почта

  • Артикул
Модуль:
Microsoft. Graph.Identity.DirectoryManagement

Настройте федерацию домена с организациями, поставщик удостоверений (IdP) которых поддерживает протокол SAML или WS-Fed.

Синтаксис

 Get-Mg  Каталог  Федерация  Конфигурация
   [-ExpandProperty ]
   [-Свойство ]
   [-Фильтр <Строка>]
   [-Поиск <Строка>]
   [-Пропустить ]
   [-Сортировать <Строка[]>]
   [-Верх ]
   [-PageSize ]
   [-Все]
   [-CountVariable ]
   [<Общие параметры>]    
 Get-Mg  Каталог  Федерация  Конфигурация
   -IdentityProviderBaseId 
   [-ExpandProperty ]
   [-Свойство ]
   []    
 Get-Mg  Каталог  Федерация  Конфигурация
   -InputObject 
   [-ExpandProperty ]
   [-Свойство ]
   [<Общие параметры>]    

Описание

Настройте федерацию домена с организациями, поставщик удостоверений (IdP) которых поддерживает протокол SAML или WS-Fed.

Примеры

Пример 1. Использование командлета Get-MgDirectoryFederationConfiguration

 Import-Module Microsoft.Graph.Identity.DirectoryManagement
Get-MgDirectoryFederationConfiguration -IdentityProviderBaseId $identityProviderBaseId -Filter "domains/any(x: x/id eq 'contoso.com')" 

В этом примере показано, как использовать командлет Get-MgDirectoryFederationConfiguration. Чтобы узнать о разрешениях для этого ресурса, см. справочник по разрешениям.

Пример 2. Использование командлета Get-MgDirectoryFederationConfiguration

 Import-Module Microsoft.Graph.Identity.DirectoryManagement
Get-MgDirectoryFederationConfiguration -IdentityProviderBaseId $identityProviderBaseId 

В этом примере показано, как использовать командлет Get-MgDirectoryFederationConfiguration. Чтобы узнать о разрешениях для этого ресурса, см. справочник по разрешениям.

Параметры

-Все

-CountVariable

-ExpandProperty

-Фильтр

-IdentityProviderBaseId

-InputObject

-PageSize

-Свойства

-Поиск

-Пропустить

-Sort

-Top

Входы

Microsoft. Graph.PowerShell.Models. IIdentityDirectoryManagementIdentity

Выходы

Microsoft.Graph.PowerShell.Models.IMicrosoftGraphIdentityProviderBase

Примечания

Псевдонимы

90 014 КОМПЛЕКСНЫЕ СВОЙСТВА ПАРАМЕТРА

Чтобы создать описанные ниже параметры, создайте хэш-таблицу, содержащую соответствующие свойства. Для получения информации о хеш-таблицах запустите Get-Help about_Hash_Tables.

INPUTOBJECT : Идентификационный параметр

  • [AdministrativeUnitId ] : Уникальный идентификатор administrativeUnit
  • [AllowedValueId ] : Уникальный идентификатор allowValue
  • [AttributeSetId ] : Уникальный идентификатор attributeSet
  • [CommandId ] : Уникальный идентификатор команды
  • [ContractId ] : Уникальный идентификатор контракта
  • [CustomSecurityAttributeDefinitionId ] : уникальный идентификатор customSecurityAttributeDefinition
  • [DeviceId ] : Уникальный идентификатор устройства
  • [DirectoryObjectId ] : Уникальный идентификатор directoryObject
  • [DirectoryRoleId ] : Уникальный идентификатор directoryRole
  • [DirectoryRoleTemplateId ] : уникальный идентификатор directoryRoleTemplate
  • [DirectorySettingId ] : Уникальный идентификатор directorySetting
  • [DirectorySettingTemplateId ] : Уникальный идентификатор directorySettingTemplate
  • [DomainDnsRecordId ] : Уникальный идентификатор domainDnsRecord
  • [DomainId ] : Уникальный идентификатор домена
  • [ExtensionId ] : Уникальный идентификатор расширения
  • [FeatureRolloutPolicyId ] : уникальный идентификатор featureRolloutPolicy
  • [IdentityProviderBaseId ] : Уникальный идентификатор identityProviderBase
  • [ImpactedResourceId ] : Уникальный идентификатор ImpactedResource
  • [InboundSharedUserProfileUserId ] : Уникальный идентификатор inboundSharedUserProfile
  • [InternalDomainFederationId ] : Уникальный идентификатор internalDomainFederation
  • [OnPremisesDirectorySynchronizationId ] : уникальный идентификатор onPremisesDirectorySynchronization
  • [OrgContactId ] : Уникальный идентификатор orgContact
  • [OrganizationId ] : Уникальный идентификатор организации
  • [OrganizationalBrandingLocalizationId ] : Уникальный идентификатор OrganizationalBrandingLocalization
  • [OutboundSharedUserProfileUserId ] : уникальный идентификатор outboundSharedUserProfile
  • [ProfileCardPropertyId ] : Уникальный идентификатор profileCardProperty
  • .
  • [RecommendationId ] : Уникальный идентификатор рекомендации
  • [ScopedRoleMembershipId ] : Уникальный идентификатор scopedRoleMembership
  • [SharedEmailDomainId ] : Уникальный идентификатор sharedEmailDomain
  • [SharedEmailDomainInvitationId ] : Уникальный идентификатор sharedEmailDomainInvitation
  • [SubscribedSkuId ] : Уникальный идентификатор subscribedSku
  • [TenantReferenceTenantId ] : уникальный идентификатор tenantReference
  • [UsageRightId ] : Уникальный идентификатор использованияRight
  • [UserId ] : Уникальный идентификатор пользователя

График функций

График функций позволяет учащемуся выбрать функцию и настроить две точки на четырехквадрантной сетке. Учащийся может настроить пунктирную линию асимптоты для экспоненциальных и логарифмических графиков. Автор элемента определяет количество ячеек в симметричной сетке, значения осей и заголовки, функциональные возможности, к которым точки могут привязываться, и функции, которые может выбирать учащийся.

Чтобы создать элемент диаграммы функций, нажмите «Создать элемент», а затем перейдите на вкладку «Элементы». В разделе «Ответные взаимодействия» щелкните знак «плюс» рядом с меткой «График функций» на левой панели.

Обратите внимание, что в настоящее время режим предварительного просмотра не будет отражать никакие введенные значения или выборки. Используйте кнопку предварительного просмотра TN8, чтобы увидеть точное представление элемента.


Сетка 

В этом разделе модуля Function Graph автор элемента определяет настройки сетки графика. Значения сетки и приращения шкалы настраиваются. Масштаб четырехквадрантной сетки по умолчанию варьируется от -5 до 5 по обоим x и и осей.

Общие

Эти настройки позволяют определить размер четырехквадрантной сетки. Вы можете определить количество ячеек в вертикальном столбце сетки (Вниз) и количество ячеек в горизонтальном ряду сетки (Поперек). Количество ячеек включает оба квадранта, которые имеют одну и ту же ось.

X — и  Y -Axis subsections

Приращение масштаба — это положительное целое или десятичное число, представляющее разницу в значениях между каждой вертикальной или горизонтальной линией сетки. Например, при шаге масштаба 5 создаются линии сетки с метками 0, 5, 10 и т. д. Значение шага масштаба по умолчанию для каждой оси равно 1. В этом поле должно быть указано значение для использования предварительного просмотра TN8.

Максимальное значение — наибольшее значение по каждой оси; самое правое значение по оси x и самое верхнее значение по оси y . Он генерируется автоматически с использованием значений, введенных в поле «Количество ячеек вниз и поперек» и «Приращение масштаба». Эти значения необходимо ввести для расчета и отображения наибольшего значения.


Правильные ответы

В этом разделе модуля Function Graph автор элемента вводит информацию о правильном ответе.

Правильная функция

В этом подразделе автор элемента выбирает одну правильную функцию. В зависимости от выбранной функции появятся или исчезнут дополнительные поля для ввода правильной информации об ответе.

Инструкции по уравнению

Инструкции по уравнению закрыты по умолчанию. Щелкните стрелку, чтобы увидеть примеры уравнений, а также правила правильного ввода уравнений.

Уравнение

Введите уравнение для правильной функции в формате, описанном в разделе инструкции по уравнению. Все точки, нанесенные учащимся, должны соответствовать определенному уравнению, чтобы ответ считался правильным. Для просмотра элемента в предварительном просмотре TN8 необходимо ввести уравнение.

Значения графика

Все значения графика следует вводить с тем же значением десятичного разряда, которое фиксирует предварительный просмотр TN8. Вам не нужно включать конечные нули. Лучший способ получить это — просмотреть элемент в предварительном просмотре TN8, разместить точки в правильных местах и ​​нажать «Получить счет». Скопируйте числа, которые вы видите в переменной RESPONSE (или RESPONSE_A, RESPONSE_B и т. д.), в соответствующее поле.

Значение асимптоты : Если правильная функция является экспоненциальной или логарифмической, автор элемента должен ввести значение асимптоты. Это значение y по оси асимптоты экспоненциальной функции является значением по оси x асимптоты логарифмической функции. Он может быть положительным или отрицательным, а также десятичным. Вам не нужно включать x= или y= перед значением, и интерфейс не позволит вам это сделать.

Среднее значение : Если правильной функцией является тангенс/котангенс или синус/косинус, автор элемента должен ввести среднее значение. Средняя линия – это среднее значение  y  значение максимума и значение y минимума функции. Это может быть положительное или отрицательное число, а также десятичное число. Вам не нужно включать x= или y= перед значением, и интерфейс не позволит вам это сделать.

Значение квартального периода : Если правильной функцией является тангенс/котангенс или синус/косинус, автор элемента должен ввести значение квартального периода. Квартальный период – это абсолютное значение разницы между значением 90 260 x 90 261 средней точки и 9 0260 x  значение соседнего максимума или x значение соседнего минимума. Это может быть положительное или отрицательное число, а также десятичное число. Вам не нужно включать x= или y= перед значением, и интерфейс не позволит вам это сделать.


Точка вершины. Если правильным значением функции является абсолютное или квадратичное значение, автор элемента должен ввести координаты x и y для точки вершины функции. Они могут быть положительными или отрицательными, а также десятичными.

Наименьшее общее кратное 12 и 30

Калькулятор «Наименьшее общее кратное»

Какое наименьшее общее кратное (НОК) у чисел 12 и 30?

Ответ: НОК чисел 12 и 30 это 60

(шестьдесят)

Нахождение наименьшего общего кратного для чисел 12 и 30 используя НОД этих чисел

Первый способ нахождения НОК для чисел 12 и 30 — через нахождение наибольшего общего делителя (НОД) этих чисел. Формула:

НОК = (Число1 × Число2) ÷ НОД

НОД чисел 12 и 30 равняется 6, следовательно

НОК = (12 × 30) ÷ 6

НОК = 360 ÷ 6

НОК = 60

Нахождение наименьшего общего кратного для чисел 12 и 30 используя перечисление кратных

Второй способ нахождения НОК для чисел 12 и 30 заключается в перечислении всех кратных для обоих чисел и выбор первого совпадающего:

Кратные числа 12: 12, 24, 36, 48, 60, 72, 84

Кратные числа 30: 30, 60, 90, 120

Следовательно, НОК для 12 и 30 равняется 60

Нахождение наименьшего общего кратного для чисел 12 и 30 используя разложение чисел на простые множители

Еще один способ нахождения НОК чисел 12 and 30 — это нахождение всех простых множителей для обоих чисел и перемножение самых больших экспоненциальных форм

Все простые множители числа 12: 2, 2, 3 (экспоненциальная форма: 22, 31)

Все простые множители числа 30: 2, 3, 5 (экспоненциальная форма: 21, 31, 51)

22 × 31 × 51 = 60

Похожие расчеты

Поделитесь текущим расчетом

Печать

https://calculat. io/ru/number/least-common-multiple-lcm-of/12—30

<a href=»https://calculat.io/ru/number/least-common-multiple-lcm-of/12—30″>Наименьшее общее кратное 12 и 30 — Calculatio</a>

О калькуляторе «Наименьшее общее кратное»

Данный калькулятор поможет найти Наименьшее общее кратное двух чисел. Например, он может помочь узнать какое наименьшее общее кратное (НОК) у чисел 12 и 30? Выберите первое число (например ’12’) и второе число (например ’30’). После чего нажмите кнопку ‘Посчитать’.

Наименьшее общее кратное (НОК) для двух чисел - это наименьшее натуральное число, которое делится на оба числа без остатка

Калькулятор «Наименьшее общее кратное»

Таблица Наименьших общих кратных

Число 1Число 2НОК
13030
23030
33030
43060
53030
63030
730210
830120
93090
103030
1130330
123060
1330390
1430210
153030
1630240
1730510
183090
1930570
203060
2130210
2230330
2330690
2430120
2530150
2630390
2730270
2830420
2930870
303030

событий, гонок и фестивалей | Открытый центр Нантахала

  • Все категории
  • Все местоположения
Women’s Paddlefest – ПРОДАНО

2 июня, 19:00 – 4 июня 2023 г. , 14:00

Nantahala Outdoor Centre

Присоединяйтесь к группе женщин-гребцов-единомышленниц на фестивале выходного дня, организованном вдохновляющим гребцом Анна Левеск! Этот фестиваль посвящен развитию сообщества путем общения, обмена, гребли и поддержки друг друга на реке. Поездки будут предлагаться по разным рекам и озерам, представляющим различные уровни квалификации, и открыты для всех […]

Фестиваль

Ретрит

Женское мероприятие

Subaru Summer Sounds

2 июня, 17:00 – 2 сентября 2023 г., 20:00

Nantahala Outdoor Centre

От местных фаворитов до новых региональных исполнителей, все живут на берегах реки Нантахала. Сплавляйтесь, обедайте, а затем играйте с нами под живую музыку каждую пятницу и субботу с 17:00 до 20:00 между Днем памяти и Днем труда! Смотрите расписание ниже!

Живая музыка

июнь 10

River Fam Jam

10 июня, 11:00 – 10 июня 2023 г. , 20:00

Нантахала, Северная Каролина

Мы планируем начать лето на ура! Приходите и наслаждайтесь живой музыкой, семейными мероприятиями, подарками, приключениями и многим другим прямо на красивой реке Нантахала. Основные моменты мероприятия: Местная деревня ремесленников с участием местных ремесленников и ремесленников Стенды партнеров сообщества Face Painter Живая музыка Riverside 13–16:00 Jacob’s Well Live Music Riverside 5–[

Подарки

Живая музыка

июнь 10

Летние ремесленные рынки

10 июня, 12:00 – 9 сентября 2023 г., 17:00

Открытый центр Нантахала

От региональных ремесленников до местных гидов-ремесленников, расположенных на берегу реки Нантахала. Приходите и знакомьтесь с этими и другими продавцами во вторую субботу каждого месяца с 12:00 до 17:00, с мая по сентябрь. Если вы заинтересованы в том, чтобы присоединиться к нам в качестве поставщика, пожалуйста, заполните свою информацию здесь: https://forms. gle/BgqQQpY4XSGqtH6f6

Событие поставщика

Not Your Average Ladies Night

5 июля, 18:30 – 5 июля 2023 г., 20:30

Открытый центр Nantahala

Приготовьтесь к ежегодному вечеру NOC Not Your Average Ladies Night! Одно из наших любимых событий года, где мы отмечаем женщин на природе! Встретьтесь с нами в магазине Outfitter’s Store, чтобы пообщаться, получить VIP-приватный шоппинг, выступить с докладом, поесть и выпить, а также получить призы. Скоро регистрация!

Фестиваль

Женское мероприятие

Август 19

River and Brews

Фестиваль ремесленных пивоваров в маленьком городке

19 августа, 13:00 – 19 августа 2023 г., 17:00 небольшие горные городки Западной Северной Каролины. От местных пивоварен с уникальным и вкусным наливом до творческих ремесленников и квалифицированных мастеров, эти люди помогают сделать наш дом таким же уникальным, как реки, которые здесь протекают. Пакеты на утренний рафтинг и пакеты билетов на дневные мероприятия […]

Фестиваль

Еда и напитки

Живая музыка

Tread Fest

8 сентября, 18:00 – 10 сентября 2023 г., 14:00

Открытый центр Nantahala

 Приготовьтесь к Tread Fest, главному фестивалю катания на горных велосипедах выходного дня, который скоро пройдет в НОК! Начните свой день с адреналиновой утренней гонки, клиник для улучшения ваших навыков, множества продавцов и обмена подержанного снаряжения по выгодным ценам. Оставайтесь с нами для более подробной информации! Идите вперед и забронируйте жилье на […]

Фестиваль

Сентябрь 22

Фестиваль благодарности гостям

22 сентября, 10:00 – 24 сентября 2023 г., 18:00

Открытый центр Нантахала

Любимый фестиваль благодарности NOC собирает любителей активного отдыха и приключений, а также гостей, чтобы завершить фестиваль выходного дня сезон! Наслаждайтесь розничными продажами, распродажами снаряжения и лодок, демоверсиями, релизами Nantahala, живой музыкой и многим другим!

Фестиваль

Розыгрыши подарков

Живая музыка

Розничная продажа

Мероприятие для поставщиков

Октябрь 13

Women’s Wilderness Weekend

13 октября, 8:00 — 15 октября 2023 г. , 15:00

Nantahala Outdoor Centre

Women’s Wilderness Weekend — это веселые и интерактивные выходные, посвященные изучению основных основ выживания и оказания первой помощи! Участники примут участие в дикой природе и проведут время в компании опытных учителей и женщин-единомышленниц. Выходные включают в себя как классное, так и практическое обучение, сертификацию по оказанию первой помощи в дикой природе, сердечно-легочную реанимацию и выживание в дикой природе […]

Ретрит

Женское мероприятие

ноябрь 10

Women’s Pedal Fest

10 ноября, 17:30 – 12 ноября 2023 г., 17:00

Нантахала, Северная Каролина

Приготовьтесь к первому женскому педальному фестивалю, чтобы получить образование, опыт и новые возможности в области катания на горных велосипедах национального леса Нантахала!

Женское мероприятие

Несколько дат событий

Women in the Outdoors

Присоединяйтесь к нам, чтобы отпраздновать День женщин на природе! Встречайтесь с женщинами-единомышленниками для веселья, товарищества и приключений на выходных этим летом и осенью! Женский каяк-ретрит — 5–7 мая и 8–10 сентября Присоединяйтесь к группе женщин-гребцов под руководством рок-звезд-инструкторов NOC на выходных […]

Несколько дат

West Fork Tuck River Releases

West Fork Tuck River

West Fork of the Tuck представляет собой бурную воду класса III-IV+ протяженностью 5,5 миль от плотины Торп до электростанции Торп. Посмотреть расписание выпусков!

Несколько дат

Спуски по реке Окои

Река Окои

Уайтуотер Окои с NOC — это лучший рафтинг в Теннесси. Просмотр графиков плотин, включая уровни озера и графики сброса.

Несколько дат

Попуски верхних каскадов

Каскады реки Нантахала

Несколько раз в год компания Duke Energy создает более сильные попуски воды в этом участке реки Нантахала, где каякеры любят плавать и привлекать зрителей. НОК предоставит трансфер до стоянки гребцов у пристани Каскадов.

Несколько дат

900:30 Выбросы через реку Таллула

Река Таллула

20:23 Запланированные выпуски из реки Таллула доставлены вам компанией American Whitewater. Nantahala Outdoor Center является гордым спонсором работы American Whitewater по защите и восстановлению рек по всей стране, особенно на юго-востоке. Nantahala Outdoor Center является гордым спонсором работы American Whitewater по защите и восстановлению рек по всей стране. Узнать […]

Несколько дат

Реки Чеоа

Река Чеоа

2023 Запланированные выпуски реки Чеоа доставлены вам компанией American Whitewater. Nantahala Outdoor Center является гордым спонсором American Whitewater и их усилий по сохранению бурной воды здесь, на юго-востоке. Узнайте больше о некоммерческой организации American Whitewater и о том, как стать ее членом, здесь. Участок реки Чеоа протяженностью 9,25 мили, […]

Право на подачу заявки на обучение в Canadian Experience Class (Express Entry)

Минимальные требования

Вы должны

  • соответствовать требуемому уровню владения языком, необходимому для вашей работы, для каждой языковой способности
    • письмо
    • чтение
    • прослушивание
    • говорящий
  • иметь как минимум 1 год квалифицированного опыта работы в Канаде за последние 3 года перед подачей заявления — вы можете выполнить это несколькими способами:
    • полный рабочий день на 1 месте работы: 30 часов в неделю в течение 12 месяцев = 1 год полной занятости (1560 часов)
    • равное количество при работе неполный рабочий день: например, 15 часов в неделю в течение 24 месяцев = 1 год полной занятости (1560 часов)
      • Вы можете работать столько раз, сколько вам нужно для выполнения этого требования.
    • полный рабочий день на более чем 1 месте работы: 30 часов в неделю в течение 12 месяцев на более чем 1 месте работы = 1 год полного рабочего дня (1560 часов)
  • приобрели опыт работы, работая в Канаде, имея статус временного резидента с разрешением на работу

Как мы рассчитываем опыт работы

Ваш квалифицированный опыт работы должен оплачиваться, включая заработную плату или комиссионные. Мы не учитываем волонтерскую работу или неоплачиваемые стажировки.

Работая неполный рабочий день, вы можете работать больше или меньше 15 часов в неделю, если в сумме это составляет 1560 часов. Вы можете работать более чем на одной работе с частичной занятостью, чтобы получить количество часов, необходимых для подачи заявки.

Мы не учитываем часы работы свыше 30 часов в неделю.

Вам 9 лет0055 не соответствует требованиям для участия в программе Canadian Experience Class, если

  • вы подаете заявление о предоставлении статуса беженца в Канаде
  • вы работаете без авторизации
  • ваш опыт работы получен без статуса временного резидента Канады

В соответствии с Канадской национальной классификацией профессий (NOC) квалифицированный опыт работы означает опыт работы, полученный в категориях TEER 0, 1, 2 или 3. Ваш опыт работы может относиться к одной или нескольким из этих категорий.

Вы должны показать, что выполняли обязанности, изложенные в ведущем заявлении описания профессии в NOC. Сюда входят все основные обязанности и большинство перечисленных основных обязанностей.

Самозанятость и опыт работы студентов

Самозанятость и опыт работы, полученные во время учебы на дневном отделении (даже если вы были на условиях совместной работы) не засчитывается в счет минимальных требований для этой программы.

Освобождение для врачей

Мы ввели временную государственную политику для иностранных врачей, которые

  • были приглашены подать заявление на постоянное проживание через Express Entry 25 апреля 2023 года или позже и подали заявление
  • иметь опыт работы в сфере предоставления государственных медицинских услуг в Канаде (например, оплачиваемые услуги) и
  • не имеют права, потому что они работают не по найму

Теперь мы учитываем этот опыт работы как канадский опыт работы. Когда вы создаете свой профиль Express Entry, чтобы ваш опыт работы мог учитываться как опыт работы в Канаде, не установите флажок «Работа на себя» под Опыт работы в Канаде .

Образование

Для участия в программе Canadian Experience Class не требуется никакого образования.

Если вы хотите повысить свой рейтинг в пуле Express Entry для иммиграционных целей , вы можете сделать это двумя способами.

  • Если вы учились в Канаде, вы можете получить баллы за сертификат, диплом или степень канадца:
    • среднее учреждение (средняя школа) или
    • высшее учебное заведение

    или

  • Если у вас есть иностранное образование, вы можете получить баллы за:
    • завершенный иностранный сертификат и
    •  отчет об оценке образовательного уровня (ECA) для иммиграционных целей от назначенной организации, подтверждающий, что ваше образование соответствует завершенному аттестату, диплому или степени, полученному в Канаде:
      • среднее учебное заведение (средняя школа) или
      • высшее учебное заведение

Владение языками

Вы должны:

  • соответствовать минимальному уровню владения языком:
    • Тест канадского языка 7 для рабочих мест TEER 0 или TEER 1 или
    • Тест канадского языка 5 для рабочих мест TEER 2 или TEER 3
  • пройти утвержденные языковые тесты на:
    • письмо
    • чтение
    • прослушивание
    • говорящий
  • опишите результаты теста в своем профиле Express Entry

Ваши языковые тесты действительны в течение 2 лет после даты результатов теста и должны быть действительны на день подачи заявления на постоянное проживание.

Доступность

Вы должны иметь право на въезд в Канаду.

Где вы можете жить в Канаде

Вы должны планировать жить за пределами провинции Квебек. Провинция Квебек сама отбирает квалифицированных рабочих. Если вы планируете жить в Квебеке, дополнительную информацию см. в разделе Квалифицированные работники, выбранные Квебеком.

Когда вы заполните свой профиль, мы спросим вас, где вы планируете жить в Канаде. Вам не нужно селиться в этой провинции или территории.

Если вы номинант от провинции, вы должны поселиться в провинции или на территории, которая вас выдвинула.

Следующие шаги

  • подготовьте документы
  • отправить свой профиль

Ваш стаж должен быть не менее:

  • Один год полной занятости
    • 30 часов в неделю в течение 12 месяцев = 1 год полной занятости (1560 часов), или
  • равное количество часов неполного рабочего времени, например:
    • 15 часов в неделю в течение 24 месяцев = 1 год полной занятости (1560 часов)
    • 30 часов в неделю в течение 12 месяцев на более чем 1 месте работы = 1 год полной занятости (1560 часов)

Если вы работаете неполный рабочий день, вы можете работать больше или меньше 15 часов в неделю, если в сумме это составляет 1560 часов.

Найти косинус угла между векторами ab и ac: Найти косинус угла между векторами AB И AC… — Математика

вычислить косинус угла между векторами онлайн

Вы искали вычислить косинус угла между векторами онлайн? На нашем сайте вы можете получить ответ на любой математический вопрос здесь. Подробное решение с описанием и пояснениями поможет вам разобраться даже с самой сложной задачей и калькулятор онлайн косинус угла между векторами, не исключение. Мы поможем вам подготовиться к домашним работам, контрольным, олимпиадам, а так же к поступлению в вуз. И какой бы пример, какой бы запрос по математике вы не ввели — у нас уже есть решение. Например, «вычислить косинус угла между векторами онлайн».

Применение различных математических задач, калькуляторов, уравнений и функций широко распространено в нашей жизни. Они используются во многих расчетах, строительстве сооружений и даже спорте. Математику человек использовал еще в древности и с тех пор их применение только возрастает. Однако сейчас наука не стоит на месте и мы можем наслаждаться плодами ее деятельности, такими, например, как онлайн-калькулятор, который может решить задачи, такие, как вычислить косинус угла между векторами онлайн,калькулятор онлайн косинус угла между векторами,калькулятор онлайн угол между векторами,калькулятор угол между векторами,косинус угла между векторами калькулятор онлайн,косинус угла между векторами онлайн,косинус угла между векторами онлайн калькулятор,найти косинус угла между векторами ab и ac онлайн калькулятор,найти косинус угла между векторами ав и ас,найти косинус угла между векторами ав и ас онлайн калькулятор,найти косинус угла между векторами онлайн,найти косинус угла между векторами онлайн калькулятор,найти онлайн угол между векторами,найти угол между векторами ав и ас,найти угол между векторами онлайн,найти угол между векторами онлайн калькулятор,онлайн калькулятор косинус угла между векторами,онлайн калькулятор найти косинус угла между векторами,онлайн калькулятор определить угол между векторами,онлайн калькулятор угол между векторами,онлайн найти угол между векторами,определить угол между векторами онлайн калькулятор,угол между векторами калькулятор,угол между векторами калькулятор онлайн,угол между векторами найти онлайн,угол между векторами онлайн,угол между векторами онлайн калькулятор. На этой странице вы найдёте калькулятор, который поможет решить любой вопрос, в том числе и вычислить косинус угла между векторами онлайн. Просто введите задачу в окошко и нажмите «решить» здесь (например, калькулятор онлайн угол между векторами).

Где можно решить любую задачу по математике, а так же вычислить косинус угла между векторами онлайн Онлайн?

Решить задачу вычислить косинус угла между векторами онлайн вы можете на нашем сайте https://pocketteacher.ru. Бесплатный онлайн решатель позволит решить онлайн задачу любой сложности за считанные секунды. Все, что вам необходимо сделать — это просто ввести свои данные в решателе. Так же вы можете посмотреть видео инструкцию и узнать, как правильно ввести вашу задачу на нашем сайте. А если у вас остались вопросы, то вы можете задать их в чате снизу слева на странице калькулятора.

2.3. Угол между векторами

Постановка задачи. Даны точки A(x1,y1,z1),B(x2,y2,z2) и C(x3,y3,z3).Найти косинус угла между векторами AB и AC.

План решения. Косинус угла между векторами AB и AC определяются формулой

.

  1. Чтобы вычислить длины векторов и и скалярное произведение , находим координаты векторов:

= {x2 – x1,y2 – y1, z2 – z1}, ={x3 – x1, y3 – y1, z3 – z1}.

2. По формулам для длины вектора и скалярного произведения векторов имеем

=

=

=(x2 — x1) (x3 – x1) + (y2 – y1) (y3 – y1) + (z2 – z1)( z3 – z1 ).

3.Вычисляем cos по формуле (1) и записываем ответ.

Пример. Даны точки A(-2,4,-6,), B(0,2,-4) и C(-6,8,-10). Найти косинус угла между и .

Решение.

1.Находим координаты векторов ={2,-2,2} и ={-4,4,-4}.

2.По формулам для длины вектора и скалярного произведения векторов имеем.

= = =

=2 (-4)+(-2) 4+2 (-4)=-24.

3.Вычисляем соs

Ответ. Косинус угла между векторами и равен — 1.

Задания для самостоятельной работы

Условия задач. Найти косинус угла между векторами и .

1. A(2,-2,3), B(1,-1,2) C(4,-4,5).

2. A(0,-2,6), B(-12,-2,-3,) C(-9,-2,-6).

3. A(2,3,-1), B(4,5,-2) C(3,1,1).

4. A(-1,2,-2,) B(3,4,-5) C(1,1,0).

5. A(-2,-2,0) B(1,-2,4) C(5,-2,1).

6. A(3,3,-1,) B(3,2,0) C(4,4,-1).

7. A(-1,-7,-4) B(2,-1,-1) C(4,3,1).

8. A(2,-2,6) B(0,0,4) C(6,-6,10).

9. A(0,1,0) B(3,1,4) C(4,1,3).

10. A(3,2,0) B(1,4,-1) C(4,0,2).

Ответы.1.cos =-1.2. cos =24/25.3. cos =-4/9.4. cos =0.5. cos = /2.6. cos =1/2.7. cos =-1.9. cos 24/25.10. cos =-8/9.

Постановка задачи. Вычислить площадь параллелограмма, построенного на векторах 1= + 2и = 1 + 2 , если известно, что | | = p0,| | = q0 и угол между векторами и равен .

План решения. Площадь параллелограмма, построенного на векторах и , равна модулю их векторного произведения:

S = |[ , ]|.

1.Вычисляем [ , ], используя свойства векторного произведения [ , ] = [ 1 + 2 , 1 + 2 ] = 1 1[ , ] + 1 2[ ] + 2 2[ ] + 2 2[ ] = ( 1 2 — 2 1) [ ].

2.Вычисляем модуль векторного произведения

|[ , ]| = | 1 2 — 2 1|| || |sin

(sin так как 0 ).

3.Находим площадь параллелограмма, используя формулу (1)

S = |[ , ]| = | 1 2 — 2 1|| || |sin

Пример.

Вычислить площадь параллелограмма, построенного на векторах = 3 + 2 и = 2 — , если известно, что | | = 4, | | =3 и угол между векторами и равен /4.

Решение.

1. Вычисляем [ , ], используя свойства векторного произведения [ , ] = [3 + 2 ,2 — ] = 6[ , ] – 3[ , ] + 4[ , ] – 2[ , ] = — 7[ , ].

2.Вычисляем модуль векторного произведения

|[ , ]| = 7[ , ]| = 7| || | sin

Ответ. Площадь параллелограмма равна 42 (ед. длины)2.

Задания для самостоятельной работы

Условия задач. Вычислить площадь параллелограмма, построенного на векторах и ( — угол между векторами и ).

1. = +3 , = 2 — | | = 2 , | | = 1 , = /6.

2. = 2 + , = — 3 | | = 2, | | = 2, = /4.

3. = — 2 , = + 3 | | = 1 , | | = 1, = /2.

4. = 3 — 5 , = + 2 | | = 2 , | | = 2, =5 /6.

5. = — , = 2 + 2 | | = 1 ,

| | = 6,

= 3 /3.

6. = + 2 , = 3 -2 | | = 3 , | | = 2,

= /3.

7. = 2 — 2 , = + | | = 2,

| | = 3,

= /2.

8. = + , = — 4 | | = 7,

| | = 4,

= /4.

9. = 4 — 4 , = + 3 | | = 2, | | = 1,

= /6.

10. = + , = 2 — | | = 2, | | = 3, = /3.

Ответы. 1. S = 7. 2. S = 14 . 3. S = 10. 4. S = 11. 5.S = 15 . 6. S = 24 . 7. S = 24. 8. S = 70 . 9. S = 16. 10. S = 9 .

Использование закона косинусов и формулы скалярного произведения векторов для нахождения угла между тремя точками – Muthukrishnan

Для любых 3 точек A, B и C на декартовой плоскости. Если нам нужно найти угол между этими точками, есть много способов сделать это. В этой статье я расскажу о двух часто используемых методах:

  1. Формула закона косинусов
  2. Вектор Точечная формула продукта

Закон косинусов

Для любого заданного треугольника ABC со сторонами AB, BC и AC угол, образованный прямыми AB и BC, определяется по формуле:

Вот как мы можем вывести эту формулу:

То же самое можно расширить и для других углов.

Теперь, если нам даны 3 точки на декартовой плоскости, мы можем найти расстояние между любыми двумя точками, используя формулу Евклидова расстояния:

Итак, чтобы найти угол между тремя точками A (x1,y1), B (x2,y2) и C (x3,y3), наша формула становится:

В Python мы можем представить приведенную выше формулу, используя код:

 импортировать numpy как np
импортировать математику
def angle_between_three_points (точка A, точка B, точка C):
    
    x1x2s = math.pow((точкаA[0] - точкаB[0]),2)
    x1x3s = math.pow((точкаA[0] - точкаC[0]),2)
    x2x3s = math.pow((точкаB[0] - точкаC[0]),2)
    
    y1y2s = math.pow((точкаA[1] - точкаB[1]),2)
    y1y3s = math.pow((точкаA[1] - точкаC[1]),2)
    y2y3s = math.pow((точкаB[1] - точкаC[1]),2)
    cosine_angle = np.arccos((x1x2s + y1y2s + x2x3s + y2y3s - x1x3s - y1y3s)/(2*math.sqrt(x1x2s + y1y2s)*math.sqrt(x2x3s + y2y3s)))
    вернуть np.степеней (косинус_угол)
A = np.массив ([2,4])
B = np. массив ([8,7])
C = np.массив ([9,1])
print("Угол между точками:", angle_between_three_points(A,B,C))
# дает 72,89
А = np.массив ([0,0])
B = np.массив ([0,4])
C = np.массив ([4,0])
print("Угол между точками:", angle_between_three_points(A,B,C))
# дает 45.0 

Векторное скалярное произведение

Три точки также могут быть представлены в виде вектора, как показано ниже.

где,

Скалярное произведение или скалярное произведение двух векторов представлено формулой:

, где || * || — величина вектора, а θ — угол между двумя векторами.

Из приведенной выше формулы мы можем представить угол, используя формулу:

В Python мы можем представить приведенную выше формулу, используя код:

 def angle_between_three_points (точка A, точка B, точка C):
    ВА = точка А - точка Б
    ВС = точка C - точка B
    пытаться:
        cosine_angle = np.dot (BA, BC) / (np.linalg.norm (BA) * np.linalg.norm (BC))
        угол = np.arccos(cosine_angle)
    кроме:
        распечатать("отл")
        поднять исключение («неверный косинус»)
    вернуть np. степени (угол) 

Выше приведен код из моего поста, где я говорил о нахождении коллинеарности трех точек на плоскости с помощью угла, образованного точками.

Каталожные номера:

  • http://mathworld.wolfram.com/DotProduct.html
  • https://en.wikipedia.org/wiki/Euclidean_distance
  • https://en.wikipedia.org/wiki/Law_of_cosines

Теги: Математика

Проверено 23 июня 2021 г., 1:35 (время сайта).

Доступно по адресу: 192.168.31.181/муту/?p=1106

MathScene — Векторы — Урок 4

MathScene — Векторы — Урок 4

2008 Расмус Эф и Джанн Сак

Урок 4

Скалярное произведение и перпендикулярные векторы


Как можно ли найти угол между двумя векторами? Правило косинуса выглядит многообещающе, поскольку мы С его помощью можно найти угол между двумя сторонами треугольника. (см. урок 3 на треугольники). Мы можем составить треугольник из двух векторов   и , и третий вектор − (видеть урок 1 по векторам 1). Посмотрите на диаграмму ниже.

В этом примере легко найти векторы, считая квадраты, но мы будет использовать общую форму для любого вектора:

Правило косинуса выглядит следующим образом:

в 2 = а 2 + б 2 − 2∙b∙a∙cos C

стороны треугольника имеют длины ||, || и | − |. Если обозначить треугольник так, что сторона a = ||, б = || и с = | − | и обозначьте угол C как v, мы получим:

| — | 2 = || 2 + || 2 — 2∙||∙ ||∙cos v

Теперь мы можем использовать формулу расстояния, чтобы найти длину векторов. Мы можем опустите квадратный корень, поскольку мы возводим в квадрат длины.

а — х б ) 2 + (y a − y b ) 2 = x a 2 + у а 2 + х б 2 + у б 2 — 2∙||∙ ||∙cos v  

Если мы упростим левую часть, умножив скобки, получится:

х а 2 − 2x a x b + x b 2 + у а 2 — 2г а г б + г б 2

Подставляя это в формулу, мы видим, что все члены в квадрате сокращаются, и мы остаются с уравнением

− 2x a x b − 2y a y b = — 2∙||∙ ||∙cos v  

Разделив на −2, мы получим следующую важную формулу:  

х а х б + у а у б = ||∙ ||∙cos v  

Каждая часть этого уравнения называется скалярным произведением двух векторов. В других слов:

скалярное произведение и «=» x a x b + y a y b  и
скалярное произведение и «=» ||||кос v

Символ скалярного произведения — это точка, похожая на символ, иногда используемый для умножение. Очень важно различать их. Точка написана между двумя векторами всегда скалярное произведение. На самом деле его часто называют Скалярное произведение. Используя это обозначение, мы можем написать:

∙ = х а х б + у а у б  и ∙ = ||∙ ||∙cos v  


Пример 1

Напишите скалярное произведение векторов и в следующую схему двумя разными способами.

Мы может считывать координаты векторов прямо с диаграммы. Мы также можем см. длину вектора но нам нужно вычислить длину вектора .

Использование координат для нахождения скалярного произведения:

∙ = 3∙4 + 3∙0 = 12

Координаты x перемножаются, y координаты перемножаются, а затем два результата складываются вместе .

Используя другое определение скалярного произведения с длинами векторов и косинусом, мы получить:

|| =

|| = 4

Теперь мы ответим на поставленный нами вопрос и посмотрим, как мы можем найти угол между двумя векторами.

Для этого мы используем обе формы скалярного произведения.

х а х б + у а у б = ||∙ ||∙cos v  

Разделив на |||| получаем следующую формулу:

Если мы используем формулу расстояния, чтобы найти длины, это будет:

 


Пример 2

Найдите углы между векторами

≈ 0,8575

в ≈ потому что −1 0,8575 ≈ 31

 


Пример 3

Найдите вектор такой же длины, как и перпендикулярно .

Мы называем этот вектор и напишите формулу скалярного произведения и .

х а х б + у а у б = ||∙ ||∙cos 90  

Но cos 90 = 0, поэтому мы получаем следующий результат:

.

Икс + 2г = 0

Мы также знаем, что || = х 2 + у 2 = 5.

Решая эти два уравнения вместе, мы получаем , x + 2y = 0 и x 2 + у 2 = 5.

х = -2у и х 2 = 5 — у 2

х 2 = 4г 2 = 5 — г 2

5 лет 2 = 5

у 2 = 1

у = 1

Если y = 1, то x = −2, а если y = −1, то x = 2,

Это означает, что у нас есть две возможности

На диаграмме мы видим эти два результата.


Обобщая приведенные выше результаты, мы видим, что мы можем найти два вектора, которые перпендикулярны друг другу и имеют одинаковую длину путем переключения вокруг x и координаты y и изменение знака по одной из координат.

Вектор перпендикулярно и

Это связано с тем, что угол между перпендикулярными векторами равен 90, а cos 90 = 0

Если вектор является перпендикулярно вектору затем их скалярное произведение равно нулю. х а х б + у а у б = 0

Верно и обратное этому правилу.

Если скалярное произведение двух векторов равно нулю, то они перпендикулярны друг друга.

 


Пример 4

Треугольник ABC имеет вершины A = (3, 4), B = (17, 12) и C = (5, 16). Найди углы треугольника.

Сначала мы находим векторы, представляющие стороны.

Нам больше не нужно выполнять какую-либо работу, так как векторы и являются явно перпендикулярны и равны по длине.

Решение уравнений методом горнера онлайн: Схема (метод) Горнера. Примеры. Разложение многочлена на множители

Схема (метод) Горнера. Примеры. Разложение многочлена на множители

Пример 1

Пример 2

Пример 3

Пример 4

РАЗЛОЖЕНИЕ МНОГОЧЛЕНА НА МНОЖИТЕЛИ ПО СХЕМЕ ГОРНЕРА

5x5 — 2x4 — 25x3 + 10x2 + 20x — 8

Для начала нужно методом подбора найти один корень. Обычно он является делителем свободного члена. В данном случае делителями числа -8 являются ±1, ±2, ±4, ±8. Начнем их подставлять по-очереди:

1: 5 — 2 — 25 + 10 + 20 — 8 = 0 ⇒ число 1 является корнем многочлена

Мы нашли 1 из корней многочлена. Корнем многочлена является 1, а значит исходный многочлен должен делиться на x — 1. Для того, чтобы выполнить деление многочленов, воспользуемся схемой Горнера:

5-2-251020-8
1

В верхней строке выставляются коэффициенты исходного многочлена. В первой ячейке второй строки ставится найденный нами корень 1. Во второй строке пишутся коэффициенты многочлена, который получится в результате деления. Они считаются так:

5-2-251020-8
15
Во вторую ячейку второй строки запишем число 5, просто перенеся его из соответствующей ячейки первой строки.
5-2-251020-8
153
1 ∙ 5 — 2 = 3
5-2-251020-8
153-22
1 ∙ 3 — 25 = -22
5-2-251020-8
153-22-12
1 ∙ (-22) + 10 = -12
5-2-251020-8
153-22-128
1 ∙ (-12) + 20 = 8
5-2-251020-8
153-22-1280
1 ∙ 8 — 8 = 0

Последнее число — это остаток от деления. Если он равен 0, значит мы все верно посчитали.

Таким образом мы исходный многочлен разложили на множители:

5x5 — 2x4 — 25x3 + 10x2 + 20x — 8 = (x — 1)(5x4 + 3x3 — 22x2 — 12x + 8)

Но это еще не конец. Можно попробовать разложить таким же способом многочлен 5x4 + 3x3 — 22x2 — 12x + 8.

Опять ищем корень среди делителей свободного члена. Делителями числа 8 являются ±1, ±2, ±4, ±8.

1: 5 + 3 — 22 — 12 + 8 = -18 ⇒ число 1 не является корнем многочлена

-1: 5 — 3 — 22 + 12 + 8 = 0 ⇒ число -1 является корнем многочлена

Напишем найденный корень в нашу схему Горнера и начнем заполнять пустые ячейки:

5-2-251020-8
153-22-1280
-15
Во вторую ячейку третьей строки запишем число 5, просто перенеся его из соответствующей ячейки второй строки.
5-2-251020-8
153-22-1280
-15-2
-1 ∙ 5 + 3 = -2
5-2-251020-8
153-22-1280
-15-2-20
-1 ∙ (-2) — 22 = -20
5-2-251020-8
153-22-1280
-15-2-208
-1 ∙ (-20) — 12 = 8
5-2-251020-8
153-22-1280
-15-2-2080
-1 ∙ 8 + 8 = 0

Таким образом мы исходный многочлен разложили на множители:

5x5 — 2x4 — 25x3 + 10x2 + 20x — 8 = (x — 1)(x + 1)(5x3 — 2x2 — 20x + 8)

Теперь найдем корень многочлена 5x3 — 2x2 — 20x + 8. Делителями числа 8 являются ±1, ±2, ±4, ±8.

1: 5 — 2 — 20 + 8 = -9 ⇒ число 1 не является корнем многочлена

-1: -5 — 2 + 20 + 8 = 29 ⇒ число -1 не является корнем многочлена

2: 5 ∙ 8 — 2 ∙ 4 — 20 ∙ 2 + 8 = 0 ⇒ число 2 является корнем многочлена

5-2-251020-8
153-22-1280
-15-2-2080
25
Во вторую ячейку четвертой строки запишем число 5, просто перенеся его из соответствующей ячейки третьей строки.
5-2-251020-8
153-22-1280
-15-2-2080
258
2 ∙ 5 — 8 = 8
5-2-251020-8
153-22-1280
-15-2-2080
258-4
2 ∙ 8 — 20 = -4
5-2-251020-8
153-22-1280
-15-2-2080
258-40
2 ∙ (-4) + 8 = 0

Таким образом мы исходный многочлен разложили на множители:

5x5 — 2x4 — 25x3 + 10x2 + 20x — 8 = (x — 1)(x + 1)(x — 2)(5x2 + 8x — 4)

Многочлен 5x2 + 8x — 4 тоже можно разложить на множители. Для этого можно решить квадратное уравнение через дискриминант, а можно поискать корень среди делителей числа -4. Так или иначе, мы придем к выводу, что корнем этого многочлена является число -2

5-2-251020-8
153-22-1280
-15-2-2080
258-40
-25
Во вторую ячейку четвертой строки запишем число 5, просто перенеся его из соответствующей ячейки третьей строки.
5-2-251020-8
153-22-1280
-15-2-2080
258-40
-25-2
-2 ∙ 5 + 8 = -2
5-2-251020-8
153-22-1280
-15-2-2080
258-40
-25-20
-2 ∙ (-2) — 4 = 0

Таким образом мы исходный многочлен разложили на линейные множители:

5x5 — 2x4 — 25x3 + 10x2 + 20x — 8 = (x — 1)(x + 1)(x — 2)(x + 2)(5x — 2)

А корнями многочлена являются:

x = ±1; ±2; 0. {n-2}+\ldots+\normblue{a_{n-1}}x+\normblue{a_n}\\ $$ $$ \begin{array} {c|c|c|c|c|c|c|c} & \normblue{a_0} & \normblue{a_1} & \normblue{a_2} & \normblue{a_3} & \ldots & \normblue{a_{n-1}} & \normblue{a_n} \\ \hline a & & & & & & & \end{array} $$

Вторая строка таблицы заполняется постепенно. Второй элемент этой строки (обозначим его $b_0$) равен $a_0$, т.е., по сути, мы просто переносим вниз число $a_0$:

$$ \begin{array} {c|c|c|c|c|c|c|c} & a_0 & a_1 & a_2 & a_3 & \ldots & a_{n-1} & a_n \\ \hline a & b_0=a_0 & & & & & & \end{array} $$

Следующий элемент второй строки, который мы обозначим как $b_1$, получается по такой формуле: $b_1=a\cdot{b_0}+a_1$:

$$ \begin{array} {c|c|c|c|c|c|c|c} & a_0 & a_1 & a_2 & a_3 & \ldots & a_{n-1} & a_n \\ \hline a & b_0 & b_1=a\cdot{b_0}+a_1 & & & & & \end{array} $$

Далее находим элемент $b_2$ по формуле $b_2=a\cdot{b_1}+a_2$:

$$ \begin{array} {c|c|c|c|c|c|c|c} & a_0 & a_1 & a_2 & a_3 & \ldots & a_{n-1} & a_n \\ \hline a & b_0 & b_1 & b_2=a\cdot{b_1}+a_2 & & & & \end{array} $$

Аналогично вычисляем и элемент $b_3=a\cdot{b_2}+a_3$:

$$ \begin{array} {c|c|c|c|c|c|c|c} & a_0 & a_1 & a_2 & a_3 & \ldots & a_{n-1} & a_n \\ \hline a & b_0 & b_1 & b_2 & b_3=a\cdot{b_2}+a_3 & & & \end{array} $$

Далее находим $b_4$, $b_5$ и так далее. 2}+\normblue{0}\cdot{x}+\normblue{(-11)}\\ $$

Так как мы делим на $x-\normred{1}$, то в первой ячейке второй строки запишем число $\normred{1}$. Таблица, с которой мы будем работать, имеет такой вид:

$$ \begin{array} {c|c|c|c|c|c} & \normblue{5} & \normblue{5} & \normblue{1} & \normblue{0} & \normblue{-11} \\ \hline \normred{1} & & & & & \end{array} $$

Начнём заполнять пустые ячейки во второй строке. Во вторую ячейку второй строки запишем число $5$, просто перенеся его вниз из второй ячейки первой строки:

$$ \begin{array} {c|c|c|c|c|c} & \normgreen{5} & 5 & 1 & 0 & -11 \\ \hline 1 & \normgreen{5} & & & & \end{array} $$

Следующую ячейку заполним по такому принципу: $\normred{1}\cdot\normgreen{5}+\normpurple{5}=\normblue{10}$:

$$ \begin{array} {c|c|c|c|c|c} & 5 & \normpurple{5} & 1 & 0 & -11 \\ \hline \normred{1} & \normgreen{5} & \normblue{10} & & & \end{array} $$

Аналогично заполним и четвертую ячейку второй строки: $\normred{1}\cdot\normgreen{10}+\normpurple{1}=\normblue{11}$:

$$ \begin{array} {c|c|c|c|c|c} & 5 & 5 & \normpurple{1} & 0 & -11 \\ \hline \normred{1} & 5 & \normgreen{10} & \normblue{11} & & \end{array} $$

Для пятой ячейки получим: $\normred{1}\cdot\normgreen{11}+\normpurple{0}=\normblue{11}$:

$$ \begin{array} {c|c|c|c|c|c} & 5 & 5 & 1 & \normpurple{0} & -11 \\ \hline \normred{1} & 5 & 10 & \normgreen{11} & \normblue{11} & \end{array} $$

И, наконец, для последней, шестой ячейки, имеем: $\normred{1}\cdot\normgreen{11}+\normpurple{(-11)}=\normblue{0}$:

$$ \begin{array} {c|c|c|c|c|c} & 5 & 5 & 1 & 0 & \normpurple{-11} \\ \hline \normred{1} & 5 & 10 & 11 & \normgreen{11} &\normblue{0} \end{array} $$

Числа, расположенные во второй строке (между единицей и нулём), есть коэффициенты многочлена, полученного после деления $P(x)$ на $x-1$. 6$) равен единице. В этом случае целочисленные корни многочлена нужно искать среди делителей свободного члена, т.е. среди делителей числа 45. Для заданного многочлена такими корнями могут быть числа $45; \; 15; \; 9; \; 5; \; 3; \; 1$ и $-45; \; -15; \; -9; \; -5; \; -3; \; -1$. Проверим, к примеру, число $1$:

Табл. №1

$$ \begin{array} {c|c|c|c|c|c|c|c} & 1 & 2 & -21 & -20 & 71 & 114 & 45 \\ \hline 1 & 1 & 3 & -18 & -38 & 33 & 147 & 192 \end{array} $$

Как видите, значение многочлена $P(x)$ при $x=1$ равно $192$ (последнее число в второй строке), а не $0$, посему единица не является корнем данного многочлена. Так как проверка для единицы окончилась неудачей, проверим значение $x=-1$. Новую таблицу составлять не будем, а продолжим использование табл. №1, дописав в нее новую (третью) строку. Вторую строку, в которой проверялось значение $1$, выделим красным цветом и в дальнейших рассуждениях использовать её не будем. 2-21x+45$. Проверим еще раз число $-1$:

$$ \begin{array} {c|c|c|c|c|c|c|c} & 1 & 2 & -21 & -20 & 71 & 114 & 45 \\ \hline \normred{1} & \normred{1} & \normred{3} & \normred{-18} & \normred{-38} & \normred{33} & \normred{147} & \normred{192}\\ \hline -1 & 1 & 1 & -22 & 2 & 69 & 45 & 0 \\ \hline -1 & 1 & 0 & -22 & 24 & 45 & 0 & \\ \hline -1 & 1 & -1 & -21 & 45 & 0 & & \\ \hline -1 & 1 & -2 & -19 & 64 & & & \end{array} $$

Проверка окончилась неудачей. Выделим шестую строку красным цветом и попробуем проверить иное число, например, число $3$:

$$ \begin{array} {c|c|c|c|c|c|c|c} & 1 & 2 & -21 & -20 & 71 & 114 & 45 \\ \hline \normred{1} & \normred{1} & \normred{3} & \normred{-18} & \normred{-38} & \normred{33} & \normred{147} & \normred{192}\\ \hline -1 & 1 & 1 & -22 & 2 & 69 & 45 & 0 \\ \hline -1 & 1 & 0 & -22 & 24 & 45 & 0 & \\ \hline -1 & 1 & -1 & -21 & 45 & 0 & & \\ \hline \normred{-1} & \normred{1} & \normred{-2} & \normred{-19} & \normred{64} & & & \\ \hline 3 & 1 & 2 & -15 & 0 & & & \end{array} $$

В остатке ноль, посему число $3$ – корень рассматриваемого многочлена. 2+2x-15\right) \end{equation} $$

Проверим ещё раз число $3$:

$$ \begin{array} {c|c|c|c|c|c|c|c} & 1 & 2 & -21 & -20 & 71 & 114 & 45 \\ \hline \normred{1} & \normred{1} & \normred{3} & \normred{-18} & \normred{-38} & \normred{33} & \normred{147} & \normred{192}\\ \hline -1 & 1 & 1 & -22 & 2 & 69 & 45 & 0 \\ \hline -1 & 1 & 0 & -22 & 24 & 45 & 0 & \\ \hline -1 & 1 & -1 & -21 & 45 & 0 & & \\ \hline \normred{-1} & \normred{1} & \normred{-2} & \normred{-19} & \normred{64} & & & \\ \hline 3 & 1 & 2 & -15 & 0 & & & \\ \hline 3 & 1 & 5 & 0 & & & & \end{array} $$

Полученный результат можно записать так (это продолжение равенства (6)):

$$ \begin{equation} P(x)=(x+1)^3(x-3)\left(x^2+2x-15\right) =(x+1)^3(x-3)(x-3)(x+5) =(x+1)^3(x-3)^2(x+5) \end{equation} $$

Из последней скобки видно, что число $-5$ также является корнем данного многочлена. 2(x+5)$$

Числа $-1$, $3$, $-5$ – корни данного многочлена. Причем, так как скобка $(x+1)$ в третьей степени, то $-1$ – корень третьего порядка; так как скобка $(x-3)$ во второй степени, то $3$ – корень второго порядка; так как скобка $(x+5)$ в первой степени, то $x=-5$ – корень первого порядка (простой корень).

Вообще, обычно оформление таких примеров состоит из таблицы, в которой перебираются возможные варианты корней, и ответа:

$$ \begin{array} {c|c|c|c|c|c|c|c} & 1 & 2 & -21 & -20 & 71 & 114 & 45 \\ \hline \normred{1} & \normred{1} & \normred{3} & \normred{-18} & \normred{-38} & \normred{33} & \normred{147} & \normred{192}\\ \hline -1 & 1 & 1 & -22 & 2 & 69 & 45 & 0 \\ \hline -1 & 1 & 0 & -22 & 24 & 45 & 0 & \\ \hline -1 & 1 & -1 & -21 & 45 & 0 & & \\ \hline \normred{-1} & \normred{1} & \normred{-2} & \normred{-19} & \normred{64} & & & \\ \hline 3 & 1 & 2 & -15 & 0 & & & \\ \hline 3 & 1 & 5 & 0 & & & & \end{array} $$

Из таблицы следует вывод, полученный нами ранее с подробным решением:

$$ P(x)=(x+1)^3(x-3)\left(x^2+2x-15\right)=(x+1)^3(x-3)^2(x+5)$$

Ответ: $-1$, $3$, $-5$. 2-10\right) $$

Конечно, данный метод подбора малоэффективен в общем случае, когда корни не являются целыми числами, но для целочисленных корней метод довольно-таки неплох.

Вернуться к списку тем

Задать вопрос на форуме

Записаться на занятия

Онлайн-занятия по высшей математике

Метод Хорнера — Ximera

Теоретически легко вычислить численное значение многочлен путем вычисления . Вам нужно вычислить большие мощности, чтобы сделать этот.

Существует алгоритм, сокращающий вычисление для произвольных многочленов и числа к более простым умножениям и добавлениям. Алгоритм также имеет особый побочный эффект, заключающийся в получении частного от деления на . Этот алгоритм известен как метод Хорнера . Мы объясняем это здесь с помощью пример.

Разделить на .

Вызов многочлена . Воспользуемся методом Хорнера для вычисления значения в точке , и мы получаем частное бесплатно. Вы можете, конечно, также рассчитать с вашим калькулятор, но тогда вам все равно придется использовать деление Горнера или Евклидово деление, если вам нужно частное.

Стартовая схема выглядит так

где коэффициенты находятся в верхней строке. Теперь мы опускаем первый коэффициент (т.е. ), умножьте его на слева и запишите результат во второй строке под второй коэффициент (т.е. под ):

Складываем и вместе, записываем результат внизу и умножаем на лево и снова напишите результат во второй строке под , а затем добавьте к этому :

Снова умножаем нижнее на левое, пишем произведение под, складываем и получить номер в правом нижнем углу:

Теперь схема завершена, и мы можем доказать, что самое правое число в дно всегда является числовым значением многочлена в точке , и что остальные числа в нижней строке всегда являются коэффициентами частного деленное на .

Таким образом, частное равно , а остаток равен , или

В качестве примера евклидова деления мы разделили на . Мы также можем выполнить это деление по методу Горнера:

Таким образом: .

Обратите внимание, что остаток, таким образом, также представляет собой числовое значение дивиденда для , потому что

Найдите все делители .

Обратите внимание, что это та же самая задача, что и «разложить этот многочлен на множители».

Кандидаты в делители формы , с целым числом, должны быть делителями . А Возможный способ решить это упражнение — проверить для всех целых делителей числа 24, являются ли их числовое значение равно нулю. Если вы начнете с самого маленького, вы сразу повезло, потому что , и поэтому является делителем . Теперь мы можем искать численные значения делителей и , но легче вычислить с помощью Хорнера что

и поэтому . Повторим вопрос для многочлена . Окончательный результат будет. Это очень четко и полно объяснено в следующем видео, в котором Хорнер правило также объясняется в начале.

В разделе о линейных делителях мы учитывали .

Поскольку является делителем .

Рассчитываем частное по методу Хорнера. 9к$

или

f(x) = a 0 + a 1 x + a 2 x 2 + . .. + a n x n

Где a k — действительные числа, представляющие полиномиальные коэффициенты, а
x k — полиномиальные переменные.

Говорят, что приведенный выше полином имеет степень n th , т. е. deg(f(x)) = n , где n представляет наивысший переменный показатель степени.

Правило Хорнера для полиномиального деления — это алгоритм, используемый для упрощения процесса вычисления многочлена f(x) при некотором значении x = x 0 делением полинома на мономы (многочлены 1-й степени). Каждый моном включает максимум один процесс умножения и один процесс сложения. Результат, полученный от одного монома, добавляется к результату, полученному от следующего монома, и так далее в режиме накопительного сложения. Этот процесс деления также известен как синтетическое деление.

Чтобы пояснить вышеизложенное, давайте перепишем полином в его расширенной форме;

f(x 0 ) = a 0 + a 1 x 0 + a 2 x 0 2 + . .. + a н х 0 н

Это также может быть записано как:

f(x 0 ) = a 0 + x 0 (a 1 + x 0 (a 2 + x 0 ( 3 + … + (а n-1 + a n x 0 )….)

Алгоритм, предложенный этим правилом, основан на оценке мономов, образованных выше, начиная с монома в самой внутренней скобке и продвигаясь к оценке мономов во внешней скобке.

Алгоритм выполняется по следующим шагам:

1. Положим k = n
2. Пусть b k = a k
3. Пусть b k — 1 = a k — 1 + b k x 0
4. Пусть k = k — 1
5. Если k ≥ 0 , то перейти к шагу 3
Иначе Конец

Этот алгоритм также можно представить графически, рассмотрев полином степени 5  , определяемый следующим образом:

f(x) = а 0 + а 1 х + а 2 х 2 + а 3 х 3 + а 4 х 4 + 5 x 5

который можно оценить в x = x 0 , переставив его как:

f(x 0 ) = a 0 + x 0 (a 1 + x 0 (a 2 + x 0 ( а 3 + х 0 4 + а 5 х 0 ))))

Другой способ представить результаты с использованием этого алгоритма — использовать таблицу, приведенную ниже:

К 5 4 3 2 1 0
б 5 = а 5 б 4 = а 4 + х 0 б 5 б 3 = а 3 + х 0 б 4 б 2 = а 2 + х 0 б 3 б 1 = а 1 + х 0 б 2 б 0 = а 0 + х 0 б 1

Пример: Вычисление многочлена f(x) = x 4 + 3x 3 + 5x 2 + 7x + 9 при x = 2

Решение:

Так как полином имеет степень 4 , то n = 4

К 4 3 2 1 0
Шаг б 4 = 1 б 3 = 3 + 2 * 1 б 2 = 5 + 2 * 5 б 1 = 7 + 2 * 15 б 0 = 9 + 2 * 37
Результат 1 5 15 37 83

Следовательно, f(2) = 83.

Почему мы должны это делать?

Обычно при вычислении полинома с определенным значением мы привыкли подставлять это значение в полином и выполнять вычисления. Для этого мы также можем использовать или разработать приложение для математических вычислений, что необходимо, когда мы имеем дело со сложными полиномами высокой степени.

То, как компьютер обрабатывает проблему, зависит главным образом от того, как вы, как программист, описываете ее компьютеру. Вы можете разработать приложение для решения полинома прямой подстановкой или синтетическим делением по правилу Горнера. Единственная разница между этими двумя подходами заключается в скорости, с которой компьютер решит задачу в любом случае.

Преимущество правила Горнера состоит в том, что оно уменьшает количество операций умножения. Зная, что время обработки компьютером одного процесса умножения в 5-20 раз превышает время обработки процесса сложения, вы можете сказать, что разработка вашего приложения для вычисления полинома с использованием правила Хорнера значительно улучшит время выполнения, затрачиваемое компьютером.

Степень окисления h3o: какие степени окисления в этом веществе? H3O — Спрашивалка

экзотическая химия — экстремальные степени окисления, соединения элемента с самим собой: dudenkov — LiveJournal

СТРАНИЦА ХИМИЧЕСКИХ РЕКОРДОВ №1
Мой острый интерес всегда вызывали и вызывают крайние проявления строения и свойств веществ. Своего рода химический экстремизм… Надеюсь, я не одинок в этом интересе и найдутся желающие пополнить свое образование или поделиться информацией. Некоторые из приводящихся ниже фактов общеизвестны из учебных курсов, но преобладают такие, которые отражают новейшие научные данные или оригинальные теоретические представления автора (последнее оговаривается). При необходимости всегда готов поделиться источниками этих фактов, но скорость не гарантирую (бумажный архив большой, а многое вношу сюда по памяти). Итак…
     НАИВЫСШАЯ ДОСТИГНУТАЯ СТЕПЕНЬ ОКИСЛЕНИЯ +VIII известна для ксенона, рутения и осмия в составе (XeO6)-4, XeO4, RuO4, OsO4, [OsO4(OH)2]-2, (OsO3N)-1, OsO3F2, (Os2O9)-2, (OsO3F3)-1 и некоторых других соединений. НАИВЫСШИЕ ВОЗМОЖНЫЕ СТЕПЕНИ ОКИСЛЕНИЯ +IX и +X на основании квантовохимических расчетов предсказывались для (IrO4)+1, IrH9 и (PtO4)+2. Я считаю, что предлагавшийся в литературе путь получения этих степеней окисления применением сильных окислителей в щелочных средах, просто абсурден. Если бы химики мыслили менее стереотипно, может быть, соли (IrO4)+1 (например, с SbF6-1, BF4-1, PtF6-1) давно уже были получены. Дело в том, что вследствие геометрически обусловленной ограниченности координационных чисел КЧ, в бинарных соединениях вида АБx при росте степени окисления А вдоль периодов таблицы Менделеева при фиксированном Б может наступить момент, когда из-за насыщения КЧ(А) предшествовавший РОСТ КИСЛОТНОСТИ (подробно описываемый в учебниках) «неожиданно» РЕЗКО СМЕНЯЕТСЯ ИНЕРТНОСТЬЮ (примеры — CF4, CCl4, SiCl4, SiBr4, SF6, SeF6 и относительно инертные RuO4, OsO4, TeF6, следующие за сильнокислыми BF3, BCl3, AlCl3, AlBr3, PF5, AsF5, Tc2O7, Re2O7, SbF5). Мало того, при дальнейшем росте степени окисления A ВОЗНИКАЮТ ОСНОВНЫЕ СВОЙСТВА! (Примеры: (PBr4)+1Br-1, (NF4)+1(HF2)-1, соли катионов (ClF6)+1, (BrF6)+1). При наличии высокоэнергетичных, но еще достижимых повышенных КЧ(А), возможна АМФОТЕРНОСТЬ, наиболее ярко проявляющаяся в ионной структуре (примеры — (NO2)+1(NO3)-1, (PCl4)+1(PCl6)-1). Подобную амфотерность можно ожидать и для гипотетического Ir2O9, но он (как и соли IrO5-1), может оказаться кинетически неустойчив из-за значительно меньших энергий связи, чем в (IrO4)+1. А электрохимический синтез последнего (например, в среде суперкислот) должен быть затруднен тем, что на аноде будет возникать неблагоприятная основная среда, а на катоде, притягательном для иридиевого сырья, возможно только выделение металла. Но другие пути получения Ir(+IX) возможны! У меня давно созрел замысел, пока только нет времени, сырья и оборудования. Не исключаю возможность серьезного сотрудничества с желающими, имеющими опыт работы с этим элементом (и, разумеется, лицензия на такую работу) и потребность быть соавторами хорошо цитируемых публикаций.
     НИЗШИЕ ДОСТОВЕРНО ДОСТИГНУТЫЕ СТЕПЕНИ ОКИСЛЕНИЯ -V для B, Ga, In, Tl (соединения BeAlB, Al3BC, Mg5Ga2, Mg5In2, Mg5Tl2, LiMg2Ga), -VI для Zn, Hg (соединения Ca3Zn, Mg3Hg, Ca3Hg, Sr3Hg, Ba3Hg). В ряде других известных интерметаллидов, для которых мне пока не удалось собрать достаточно полную информацию о составах и структурах, МОГУТ ПРИСУТСТВОВАТЬ ЕЩЕ БОЛЕЕ НИЗКИЕ СТЕПЕНИ ОКИСЛЕНИЯ -VII (для Cu, Ag и Au в соединениях приблизительного состава Si7Cu4, Al7Au3, Mg7Ag2), -VIII (для Ni, Pd и Pt в соединениях приблизительного состава Be4Ni, Be4Pd, Be4Pt, Si2Ni, Al8Pt3, Mg4Pd, Ln7Pt3, Ln7Pd3), -IX (для Co, Rh, Ir в соединениях приблизительного состава Be9Co2, Ln3Co, Ln3Rh, Ln3Ir, Si9Co4, Th9Co4) и даже -X (в соединениях приблизительного состава Be5Fe, Th5Fe2, Th5Ru2, Th5Rh3). Хотя представления о наличии металлических анионов в полярных интерметаллидах развита давно (автор Цинтль, Zintl) и ныне широко признаны, гипотезу о наличии одноатомных анионов подобного рода в соединениях перечисленного списка мне удалось найти в литературе только применительно к ртути и другим металлам 13 подгруппы. Я обратил внимание, что особенно наглядно доказывает наличие экстремальных, сверхнизких степеней окисления сравнение бинарных соединений магния с одноатомными анионами: Mgh3, MgF2, MgCl2, MgBr2, MgI2, MgO, MgS, MgSe, MgTe, MgPo, Mg3N2, Mg3P2, Mg3As2, Mg3Sb2, Mg3Bi2, Mg2C? (синтез при высоком давлении), Mg2Si, Mg2Ge, Mg2Sn, Mg2Pb, Mg5Ga2, Mg5In2, Mg5Tl2, Mg3Hg. Во всех этих соединениях анионы образуют плотноупакованную или очень близкую к ней решетку, а важнейшие свойства типа длин связей, ширин запрещенных зон изменяются плавно, что говорит и о плавном изменении типа связи от чисто ионного к ионно-металлическому. Переход же к металлическому типу связи обычно происходит резко и встречается при наименьшей разности электроотрицательностей, обычно только при плавлении.  А вот Mg5In2 при нагревании еще до плавления обратимо переходит в фазу типично металлического строения с широкой областью гомогенности, а Mg3Cd представляет такую фазу уже при обычных условиях, а при нагревании дает бесспорно металлический твердый раствор магния и кадмия.
    
     ЭЛЕМЕНТЫ-ПСЕВДОСОЕДИНЕНИЯ — это как бы «соединения элемента с собой», в котором часть атомов имеет положительную степень окисления, а остальная часть — отрицательную (или же часть атомов образуют анионы, а остальные — катионы). Из вузовского учебника неорганической химии Ахметова известна трактовка озона O3 как ОКСИДА КИСЛОРОДА (IV) OO2, что согласуется с его строением, подобным SO2, и распределением эффективных зарядов.
     Другим примером должен оказаться НИТРИД АЗОТА (V) N3N5, для которого я ожидаю ионное строение (N2N3)+1(NN2)-1 (азид азидодиазония (N5)+1(N3)-1, или в терминологии Ахметова — динитридонитрат (V) тринитридодинитрония (V)). Из числа его компонентов, азид ион давно и хорошо известен, а соли азидодиазония (предсказанные мной вместе со способом синтеза (из N2F2, (N3)-1 и AsF5) и свойствами в неопубликованной работе 1991 года, обсуждавшейся с друзьями) успешно получены именно этим способом к 1999 году другими авторами.  Проведенный мной квантовохимический расчет полуэмпирическим методом PM3, пригодным для описания вандерваальсовых сил, показал, что химическое связывание (N5)+1 и (N3)-1 в нейтральную молекулу N8 со строением диазидодиазина или азидопентазола хотя и выгодно, но барьер должен быть достаточно высоким, чтобы азид азидодиазония (pentanitrogen azide) мог быть выделен в индивидуальном состоянии. 
      Другими примерами являются три из четырех аллотропов элементарного бора, содержащие в полостях трехмерной сетки, состоящей из соединенных обычными двухцентровыми связями пространственно-ароматических икосаэдрических и родственных им кластеров, катионоподобные атомы бора B+3 или группы (B2)+4. То есть, эти полиморфы бора по существу представляют собой ПОЛИБОРИДЫ БОРА! Например, строение наиблее термостойкой формы бора — бета-ромбоэдрического бора — приблизительно таково: (B+3)19/9((B2)+4)2/3((B12)-2)4((B27)B(B28))2/3((B27)B(B27)-3)1/3, где B+3 и (B2)+4 легко замещаются на занимающие более крупные полости структуры разнообразные металлы, и даже прилегающие неметаллы Si+4 и Ge+4. Икосаэдрический ион B12h22-2 и его разнообразные замещенные, известные с 1962 года, обладают экстремально высокой химической и термической стойкостью, что позволяет говорить о борном аналоге органической химии. Например, соль Na2B12h22 устойчива в любых водных растворах (кроме азотной кислоты), абсолютно нетоксична (у нас нет ферментов, способных разрушать этот анион) и могла бы служить заменой поваренной соли, если бы не ее цена, а на ионообменной колонке можно получить сильную кислоту (h4O)2B12h22 в форме гигроскопичного кристаллогидрата. Соль Cs2B12h22 устойчива в вакууме до 800 градусов Цельсия, что гораздо выше температуры разложения бензола. Если у бензола 2 резонансные формы, то у аниона B12h22-2 их целых 240. Водород в этом анионе может замещаться на разнообразные группы — F, Cl, Br, I, OH, OR, OR2+1, Nh4+1, NR3+1, PR3+1, Ch4, SH, SCN, SeCN, S2BS-1, Se2BSe-1, HgOOCCF3, CN, NCR+1, OOCR, N(H)C(OH)R+1, CO+, C(OH)2+1, причем для части из этих групп (галогены, OH, Ch4, ртуть- и халькогенсодержащие) известно полное замещение. Родственная молекула B10Cl8(N2)2 содержит диазониевые заместители, стабильные вплоть до 180 градусов Цельсия. От аниона B12h22-2 производится семейство сверхслабокоординирующих анионов, в последние годы позволивших выделить в твердом состоянии многие экзотические анионы. Так что ионное строение бора в действительности скорее закономерность, чем экзотика.
     Могут ли у других элементов быть аллотропы-псевдосоединения? Это вопрос. Есть основания ожидать предполагать у серы возможность ОЗОНОПОДОБНОЙ ФОРМЫ — СУЛЬФИДА СЕРЫ (IV) SS2, поскольку она будет логическим завершением ряда из двух известных изоструктурных соединений — диоксида серы SO2 и ОКСОСУЛЬФИДА СЕРЫ (IV) SSO. Однако имеющаяся у газообразного SSO склонность к полимеризации при низкотемпературной конденсации, дает основание для достаточно драматичного прогноза о степени кинетической устойчивости SS2. Я предлагаю следующий возможный способ «поймать» SS2 — экстрагировать четыреххлористым углеродом или другими инертными растворителями серу из продуктов взаимодействия SSF2 (ТИОФТОРИДА СЕРЫ (IV)) с сульфидами активных металлов. Но шансов мало — SS2  сам должен быть весьма реакционноспособен, давая полисульфид …

Всеохватывающие соединения с водой | Бурение грунтовых зондов, установка энергетических колодцев

Всеохватывающие СОЕДИНЕНИЯ С ВОДОЙ

Здрасти!! Помогите пожалуйста отыскать ответ на таковой вопрос: «Процессы комплексообразования в гидросфере. Природные и синтетические комплексообразователи».

Заблаговременно спасибо.

Процессы комплексообразования в гидросфере
Природные и синтетические комплексообразователи

Здрасти, Мария.

Это целое отдельное исследование, информацию по которому можно отыскать в специальной литературе. При анализе данной трудности необходимо знать гидрохимию, высококачественный и количественный состав гидросферы, способность ионов металлов и томных металлов в составе гидросферы к гидролизу и гидролитической полимеризации, также лигандный состав самой гидросферы — наличие в ней органических гуминовых кислот и, как следует, формы существования в их комплексообразующих ионов металлов.

Всеохватывающие соединения — это частички (нейтральные молекулы либо ионы), которые образуются в итоге присоединения к данному иону (либо атому), именуемому комплексообразователем, нейтральных молекул либо других ионов, именуемых лигандами.

Лигандами могут быть частички, до образования всеохватывающего соединения представлявшие собой молекулы (h3O, CO, Nh4 и др.), анионы (OH, Cl, PO43 и др.), также катион водорода. Различают унидентатные либо монодентатные лиганды (связанные с центральным атомом через один из собственных атомов, другими словами, одной -связью), бидентатные (связанные с центральным атомом через два собственных атома, другими словами, 2-мя -связями), тридентатные и т. д.

По заряду всеохватывающие частички могут быть катионами, анионами, также нейтральными молекулами. Всеохватывающие соединения, включающие такие частички, могут относиться к разным классам хим веществ (кислотам, основаниям, солям). Примеры: (h4O)[AuCl4] – кислота, [Ag(Nh4)2]OH – основание, Nh5Cl и K3[Fe(CN)6] – соли.

Обычно комплексообразователь – атом элемента, образующего металл, но это может быть и атом кислорода, азота, серы, йода и других частей, образующих неметаллы. Степень окисления комплексообразователя может быть положительной, отрицательной либо равной нулю; при образовании всеохватывающего соединения из более обычных веществ она не изменяется.

Координационное число определяется количеством лигандов и находится в зависимости от электрического строения центрального атома, от его степени окисления, размеров центрального атома и лигандов, критерий образования всеохватывающего соединения, температуры и других причин. КЧ может принимать значения от 2 до 12. В большинстве случаев оно равно 6, несколько пореже – четырем.

Пример всеохватывающего соединения

 

Под действием создаваемого ионом электронного поля молекулы воды спецефическим образом ориентируются и потом притягиваются к иону обратно заряженным концом диполя. За счет такового притяжения в растворе появляется гидратированный ион. Если потом раствор будет концентрироваться из него станут выделяться кристаллы растворенного вещества, заключающие в собственном составе данный ион. Если при всем этом конкретно окружающие его в молекулы воды связаны с ним некрепко, то вода не войдет в состав кристалла. Если связь иона с молекулами воды довольно высокопрочна, то в состав кристалла он войдет с неким числом молекул связанной «кристаллизационной» воды. В итоге получится кристаллогидрат данного вещества, представляющий из себя всеохватывающее соединение. К примеру, фиолетовый кристаллогидрат СrСl3 ·6Н2 О является в реальности всеохватывающим соединением [Сr(ОН2 )6 ]Сl3 , в каком комплексообразующий ион (Сr3+) держит во внутренней сфере 6 молекул воды. Схожим же образом как всеохватывающие соединения следует рассматривать и многие другие кристаллогидраты солей.

Образование всеохватывающего соединения может происходить при содействии иона не только лишь с водой в аква среде, да и с другими нейтральными молекулами. К примеру, при действии аммиака на аква раствор СuСl2 появляется комплекс состава [Cu(Nh4 )4 ]Cl2 , диссоциирующий на ионы [Cu(Nh4 )4 ]2+и 2Сl–.

Комплексообразование не непременно должно протекать в аква растворе – всеохватывающие соединения нередко образуются и при содействии жестких веществ с газообразными. К примеру, безводный СаСl2 в атмосфере газообразного аммиака дает комплекс состава [Ca(Nh4 )8 ]Cl2 . При всем этом суть самого процесса остается при всем этом той же самой и заключается в присоединении нейтральных молекул к тому либо иному иону соли за счет возникающего меж ними обоюдного притяжения.

Употребляются два вида структурных формул всеохватывающих частиц: с указанием формального заряда центрального атома и лигандов, либо с указанием формального заряда всей всеохватывающей частички. Примеры:

Для свойства формы всеохватывающей частички употребляется представление о координационном многограннике (полиэдре).

 

Входящие в состав всеохватывающих соединений всеохватывающие частички достаточно многообразны. Потому для их систематизации употребляется несколько классификационных признаков: число центральных атомов, тип лиганда, координационное число и другие.

По числу центральных атомов всеохватывающие частички делятся на одноядерные и многоядерные. Центральные атомы многоядерных всеохватывающих частиц могут быть связаны меж собой или конкретно, или через лиганды. И в том, и в другом случае центральные атомы с лигандами образуют единую внутреннюю сферу всеохватывающего соединения:

 

По типу лигандов всеохватывающие частички делятся на

1) Аквакомплексы, другими словами всеохватывающие частички, в каких в качестве лигандов находятся молекулы воды. Более либо наименее устойчивы катионные аквакомплексы [M(h3O)n]m, анионные аквакомплексы неустойчивы. Все кристаллогидраты относятся к соединениям, содержащим аквакомплексы, к примеру:

Mg(ClO4)2.6h3O по сути [Mg(h3O)6](ClO4)2;
BeSO4.4h3O по сути [Be(h3O)4]SO4;
Zn(BrO3)2.6h3O по сути [Zn(h3O)6](BrO3)2;
CuSO4.5h3O по сути [Cu(h3O)4]SO4.h3O.

2) Гидроксокомплексы, другими словами всеохватывающие частички, в каких в качестве лигандов находятся гидроксильные группы, которые до вхождения в состав всеохватывающей частички были гидроксид-ионами, к примеру: [Zn(OH)4]2, [Cr(OH)6]3, [Pb(OH)3].

Гидроксокомплексы образуются из аквакомплексов, проявляющих характеристики катионных кислот:

[Zn(h3O)4]2 + 4OH = [Zn(OH)4]2 + 4h3O

3) Аммиакаты, другими словами всеохватывающие частички, в каких в качестве лигандов находятся группы Nh4 (до образования всеохватывающей частички – молекулы аммиака), к примеру: [Cu(Nh4)4]2, [Ag(Nh4)2], [Co(Nh4)6]3.

Аммиакаты также могут быть получены из аквакомплексов, к примеру:

[Cu(h3O)4]2 + 4Nh4 = [Cu(Nh4)4]2 + 4 h3O

Расцветка раствора в данном случае изменяется с голубой до ультрамариновой.

4) Ацидокомплексы, другими словами всеохватывающие частички, в каких в качестве лигандов находятся кислотные остатки как бескислородных, так и кислородсодержащих кислот (до образования всеохватывающей частички – анионы, к примеру: Cl, Br, I, CN, S2, NO2, S2O32, CO32, C2O42 и т. п.).

Примеры образования ацидокомплексов:

Hg2 + 4I = [HgI4]2
AgBr + 2S2O32 = [Ag(S2O3)2]3 + Br

Последняя реакция употребляется в фото для удаления с фотоматериалов непрореагировавшего бромида серебра.

(При проявлении фотопленки и фотобумаги незасвеченная часть бромида серебра, содержащегося в фотографической эмульсии, не восстанавливается проявителем. Для ее удаления и употребляют эту реакцию ( процесс носит заглавие «фиксирования», потому что неудаленный бромид серебра в предстоящем на свету равномерно разлагается, разрушая изображение)

5) Комплексы, в каких лигандами являются атомы водорода, делятся на две совсем различные группы: гидридные комплексы и комплексы, входящие в состав ониевых соединений.

При образовании гидридных комплексов – [Bh5], [Alh5], [Gah5] – центральный атом является акцептором электронов, а донором – гидридный ион. Степень окисления атомов водорода в этих комплексах равна –1.

В ониевых комплексах центральный атом является донором электронов, а акцептором – атом водорода в степени окисления +1. Примеры: h4O либо [Oh4] – ион оксония, Nh5 либо [Nh5] – ион аммония. Не считая того есть и замещенные производные таких ионов: [N(Ch4)4] – ион тетраметиламмония, [As(C6H5)4] – ион тетрафениларсония, [OH(C2H5)2] – ион диэтилоксония и т. п.

6) Карбонильные комплексы – комплексы, в каких в качестве лигандов находятся группы CO (до образования комплекса – молекулы монооксида углерода), к примеру: [Cr(CO)6], [Fe(CO)5], [Ni(CO)4] и др.

7) Анионгалогенатные комплексы – комплексы типа [I(I)2].

По типу лигандов выделяют и другие классы всеохватывающих частиц. Не считая того есть всеохватывающие частички с разными по типу лигандами; простой пример – аква-гидроксокомплекс [Zn(h3O)3(OH)].

Таким макаром, всеохватывающие соединения могут быть очень многообразны, так и химия всеохватывающих соединений.

К.х.н. О. В. Мосин

 

числа окисления

числа окисления

Номера окисления


Часто полезно следить за химическими реакциями, наблюдая за изменениями в окислении числа атомов в каждом соединении во время реакции. Окислительные числа также играют важную роль. важную роль в систематической номенклатуре химических соединений. По определению окисление Число атома — это заряд, который атом имел бы, если бы соединение состояло из ионы.

1. Степень окисления атома равна нулю в нейтральном веществе, содержащем атомы. только одного элемента. Таким образом, атомы в O 2 , O 3 , P 4 , S 8 , и металлический алюминий имеют степень окисления 0,

.

2. Степень окисления простых ионов равна заряду иона. окисление число натрия в ионе Na + равно +1, например, а степень окисления хлор в Cl ион равен -1.

3. Степень окисления водорода +1, когда он соединяется с неметаллом как в CH 4 , NH 3 , H 2 O и HCl.

4. Степень окисления водорода равна -1, когда он соединяется с металлом как в. LiH, NaH, CaH 2 и LiAlH 4 .

5. Металлы группы IA образуют соединения (такие как Li 3 N и Na 2 С) в котором атом металла имеет степень окисления +1.

6. Элементы группы IIA образуют соединения (такие как Mg 3 N 2 и CaCO 3 ), в котором атом металла имеет степень окисления +2.

7. Кислород обычно имеет степень окисления -2. Исключение составляют молекулы и многоатомные ионы, содержащие связи O-O, такие как O 2 , O 3 , H 2 O 2 , и О 2 2- ион.

8. Элементы группы VIIA часто образуют соединения (такие как AlF 3 , HCl и ZnBr 2 ), в котором неметалл имеет степень окисления -1.

9. Сумма степеней окисления нейтрального соединения равна нулю.

Н 2 О: 2(+1) + (-2) = 0

10. Сумма степеней окисления в многоатомном ионе равна заряду на ион. Степень окисления атома серы в SO 4 2- ионное сусло быть +6, например, потому что сумма степеней окисления атомов в этом ионе должна равно -2.

SO 4 2- : (+6) + 4(-2) = -2

11. Элементы в нижнем левом углу периодической таблицы. с большей вероятностью имеют положительные степени окисления, чем те, которые расположены в правом верхнем углу угол стола. Сера имеет положительную степень окисления в SO 2 , для Например, потому что он ниже кислорода в периодической таблице.

SO 2 : (+4) + 2(-2) = 0

Практическая задача 6:  

Назовите следующие ионные соединения.

(а) Fe(NO 3 ) 3

(б) SrCO 3

(в) Na 2 SO 3

(г) Ca(ClO) 2

Нажмите здесь, чтобы проверить свой ответ на практическое задание 6

 

Практическая задача 7:

Присвойте степени окисления атомов в следующих соединениях.

(а) Ал 2 О 3

(б) XeF 4

(в) K 2 Cr 2 O 7

Нажмите здесь, чтобы проверить свой ответ на практическое задание 7

 

Практическая задача 8:

Расположите следующие соединения в порядке возрастания степени окисления атома углерода.

(а) СО

(б) СО 2

(с) Н 2 СО

(г) CH 3 OH

(д) CH 4

Нажмите здесь, чтобы проверьте свой ответ на практическое задание 8


Степени окисления (степени окисления)

СТЕПЕНИ ОКИСЛЕНИЯ (ЧИСЛА ОКИСЛЕНИЯ)

 

На этой странице объясняется, что такое степени окисления (степени окисления), как их рассчитать и использовать.

Степени окисления легко определить и использовать, но довольно сложно определить, что они представляют собой каким-либо быстрым способом.

 

Объяснение того, что такое степени окисления (степени окисления)

Степени окисления упрощают весь процесс определения того, что окисляется и что восстанавливается в окислительно-восстановительных реакциях. Однако для целей этого введения было бы полезно, если бы вы знали о:


Примечание:   Если вы не уверены ни в одном из этих вопросов, возможно, вам стоит просмотреть страницы, посвященные определениям окислительно-восстановительного потенциала и полууравнениям для электронов. Вероятно, было бы лучше прочитать и вернуться к этим ссылкам, если вы чувствуете, что вам это нужно.


Мы рассмотрим несколько примеров из химии ванадия. Если вы ничего не знаете о ванадии, это не имеет ни малейшего значения.

Ванадий образует ряд различных ионов, например, V 2+ и В 3+ . Если вы думаете о том, как они могут быть получены из металлического ванадия, ион 2+ будет сформирован путем окисления металла путем удаления двух электронов:

В    В 2+  +  2e

Теперь говорят, что ванадий находится в степени окисления +2.

Удаление другого электрона дает ион V 3+ :

В 2+    В 3+  +  е

Ванадий теперь имеет степень окисления +3.

Удаление другого электрона дает более необычный ион, VO 2+ .

V 3+  +  H 2 O    VO 2+   +  2H +  +  e

Теперь ванадий имеет степень окисления +4. Обратите внимание, что степень окисления — это не просто подсчет заряда иона (это верно для первых двух случаев, но не для этого).

Положительная степень окисления подсчитывает общее количество электронов, которые необходимо удалить, начиная с элемента.

Также можно удалить пятый электрон, чтобы получить другой ион (легко спутать с предыдущим!). Степень окисления ванадия теперь +5.

VO 2+  + H 2 O    VO 2 +   +  2H +  +  e

Каждый раз, когда вы окисляете ванадий, удаляя из него еще один электрон, его степень окисления увеличивается на 1,9. 0005

Совершенно очевидно, что если вы снова начнете добавлять электроны, степень окисления упадет. В конечном итоге вы могли бы вернуться к элементу ванадия, который имел бы нулевую степень окисления.

Что, если бы вы продолжали добавлять электроны к элементу? На самом деле вы не можете сделать это с ванадием, но можете с таким элементом, как сера.

S  +  2e    S 2-

Сера имеет степень окисления -2.

Сводка

Степень окисления показывает общее количество электронов, которые были удалены из элемента (положительная степень окисления) или добавлены к элементу (отрицательная степень окисления), чтобы получить его текущее состояние.

Окисление включает повышение степени окисления

Восстановление включает уменьшение степени окисления

Распознавание этой простой закономерности — самая важная вещь в концепции степеней окисления. Если вы знаете, как изменяется степень окисления элемента во время реакции, вы можете мгновенно сказать, окисляется он или восстанавливается, без необходимости работать с полууравнениями электронов и переносами электронов.

 

Определение степеней окисления

Вы не вычисляете степени окисления, подсчитывая количество переданных электронов. Это заняло бы слишком много времени. Вместо этого вы изучаете несколько простых правил и решаете очень простые задачи!

  • Степень окисления несвязанного элемента равна нулю. Это очевидно, потому что он еще не окислился и не восстановился! Это относится к любой структуре элемента — будь то, например, Xe или Cl 2 или S 8 , или имеет ли он гигантскую структуру, такую ​​как углерод или кремний.

  • Сумма степеней окисления всех атомов или ионов в нейтральном соединении равна нулю.

  • Сумма степеней окисления всех атомов в ионе равна заряду иона.

  • Более электроотрицательный элемент в веществе получает отрицательную степень окисления. Менее электроотрицательному дается положительная степень окисления. Помните, что фтор является самым электроотрицательным элементом, вторым по электроотрицательности является кислород.

  • Некоторые элементы почти всегда имеют одинаковые степени окисления в своих соединениях:

элемент обычная степень окисления исключения
металлы группы 1 всегда +1 (см. примечание ниже)
Металлы группы 2 всегда +2  
Кислород обычно -2 кроме пероксидов и F 2 O (см. ниже)
Водород обычно +1 кроме гидридов металлов, где -1 (см. ниже)
Фтор всегда -1 
Хлор обычно -1 кроме в соединениях с O или F (см. ниже)

Примечание:   Мне было указано, что существует несколько малоизвестных соединений элементов натрия с цезием, где металл образует отрицательный ион, например, Na . Это дало бы степень окисления -1.

Вы можете игнорировать их, если изучаете химию на уровне A или его эквиваленте. Обобщение, что металлы группы 1 всегда имеют степень окисления +1, справедливо для всех соединений, которые вы, вероятно, встретите.

Если вас интересуют эти необычные соединения, выполните поиск в Интернете по запросу алкалиды .



Причины исключений

Водород в гидридах металлов

Гидриды металлов включают такие соединения, как гидрид натрия, NaH. В нем водород присутствует в виде иона гидрида, H . Степень окисления простого иона, такого как гидрид, равна заряду иона, в данном случае -1.

В качестве альтернативы можно представить, что сумма степеней окисления в нейтральном соединении равна нулю. Так как металлы 1 группы в своих соединениях всегда имеют степень окисления +1, то из этого следует, что водород должен иметь степень окисления -1 (+1 -1 = 0).

Кислород в пероксидах

Пероксиды включают пероксид водорода, H 2 O 2 . Это электрически нейтральное соединение, поэтому сумма степеней окисления водорода и кислорода должна быть равна нулю.

Поскольку каждый водород имеет степень окисления +1, каждый кислород должен иметь степень окисления -1, чтобы сбалансировать его.

Кислород в F 2 O

Проблема в том, что кислород не самый электроотрицательный элемент. Фтор более электроотрицательный и имеет степень окисления -1. В этом случае кислород имеет степень окисления +2.

Хлор в соединениях с фтором или кислородом

Существует так много различных степеней окисления, которые может иметь хлор в них, что безопаснее просто помнить, что хлор не имеет степени окисления -1 в них, и определять его фактическую степень окисления, когда вам нужно. это. Пример этого вы найдете ниже.

 

Внимание!

Не зацикливайтесь на этих исключениях. В большинстве случаев, с которыми вы столкнетесь, они неприменимы!
 

Примеры определения степеней окисления

Какова степень окисления хрома в Cr 2+ ?

Это просто! Для такого простого иона степень окисления — это заряд иона, другими словами: +2 (не забывайте знак +).

Какова степень окисления хрома в CrCl 3 ?

Это нейтральное соединение, поэтому сумма степеней окисления равна нулю. Хлор имеет степень окисления -1. Если степень окисления хрома н :

n + 3(-1) = 0

n = +3 (Опять же, не забудьте знак +!)

Какова степень окисления хрома в Cr(H 2 O) 6 3+ ?

Это ион, поэтому сумма степеней окисления равна заряду иона. Существует короткий путь для определения степеней окисления в таких сложных ионах, когда атом металла окружен электрически нейтральными молекулами, такими как вода или аммиак.

Сумма степеней окисления в присоединенной нейтральной молекуле должна быть равна нулю. Это означает, что вы можете игнорировать их, когда делаете сумму. Это будет по существу то же самое, что и неприсоединенный ион хрома, Cr 3+ . Степень окисления +3.

Какова степень окисления хрома в дихромат-ионе, Cr 2 O 7 2- ?

Степень окисления кислорода равна -2, а сумма степеней окисления равна заряду иона. Не забывайте, что присутствуют 2 атома хрома.

2n + 7(-2) = -2

н = +6


Предупреждение:   Поскольку это простые суммы, заманчиво попытаться вычислить их в уме. Если это имеет значение (например, на экзамене), запишите их, используя столько шагов, сколько вам нужно, чтобы не было шансов сделать ошибки по невнимательности. Ваши экзаменаторы не будут впечатлены вашей арифметикой в ​​уме — все, что им нужно, это правильный ответ!

Если вам нужны еще примеры для практики, вы найдете их в большинстве учебников, включая мою книгу расчетов по химии.



Какова степень окисления меди в CuSO 4 ?

К сожалению, не всегда возможно определить степени окисления, просто используя приведенные выше правила. Проблема в этом случае заключается в том, что соединение содержит два элемента (медь и сера), степень окисления которых может изменяться.

Единственный способ обойти это — знать простую химию! Есть два способа приблизиться к этому. (Могут быть и другие, но я не могу о них сейчас думать!)

  • Вы можете узнать это как ионное соединение, содержащее ионы меди и сульфат-ионы, SO 4 2-. Чтобы получить электрически нейтральное соединение, медь должна присутствовать в виде иона 2+. Следовательно, степень окисления +2.

  • Вы можете узнать формулу сульфата меди(II). «(II)» в названии говорит о том, что степень окисления равна 2 (см. ниже).

    Вы будете знать, что это +2, потому что вы знаете, что металлы образуют положительные ионы, а степень окисления будет просто зарядом иона.

 

Использование степеней окисления

В названиях соединений

Вы наверняка встречали такие названия, как сульфат железа(II) и хлорид железа(III). (II) и (III) представляют собой степени окисления железа в двух соединениях: +2 и +3 соответственно. Это говорит о том, что они содержат ионы Fe 2+ и Fe 3+ .

Это также может быть распространено на отрицательный ион. Сульфат железа(II) FeSO 4 . Существует также соединение FeSO 3 со старым названием сульфит железа(II). Современные названия отражают степени окисления серы в двух соединениях.

Сульфат-ион SO 4 2- . Степень окисления серы +6 (попробуй!). Этот ион правильнее называть ионом сульфата (VI).

Сульфит-ион SO 3 2- . Степень окисления серы равна +4 (и это тоже учтите!). Этот ион правильнее называть ионом сульфата (IV). 9Окончание 0003 или просто показывает, что сера находится в отрицательном ионе.

Таким образом, FeSO 4 правильно называется сульфатом железа (II) (VI), а FeSO 3 — сульфатом железа (II) (IV). Фактически, из-за легкой путаницы между этими названиями старые названия сульфат и сульфит обычно все еще используются во вводных курсах химии.


Примечание:   Даже это не полное имя! Следует также идентифицировать кислород в отрицательных ионах. FeSO 4 правильно называется тетраоксосульфатом (VI) железа (II). Все это немного выходит из-под контроля при повседневном использовании обычных ионов.


 

Использование степеней окисления для определения того, что было окислено, а что восстановлено

Это наиболее распространенное использование степеней окисления.

Помните:

Окисление включает повышение степени окисления

Восстановление включает уменьшение степени окисления

В каждом из следующих примеров мы должны решить, включает ли реакция окислительно-восстановительный процесс, и если да, то что было окислено, а что восстановлено.

Пример 1:

Это реакция между магнием и соляной кислотой или газообразным хлористым водородом:

Mg  +  2HCl    MgCl 2   + H 2

Изменились ли степени окисления чего-либо? Да, они есть — у вас есть два элемента, которые находятся в соединениях с одной стороны уравнения и в виде несвязанных элементов с другой. Проверьте все степени окисления, чтобы быть уверенным:.

Повысилась степень окисления магния — он окислился. Степень окисления водорода упала — он восстановился. Хлор находится в одинаковой степени окисления в обеих частях уравнения — он не окисляется и не восстанавливается.

Пример 2:

Реакция между гидроксидом натрия и соляной кислотой:

NaOH + HCl NaCl + H 2 O

Проверка всех степеней окисления:

Ничего не изменилось. Это не окислительно-восстановительная реакция.

Пример 3:

Это подлый! Реакция между хлором и холодным разбавленным раствором гидроксида натрия:

2NaOH + Cl 2 NaCl + NaClO + H 2 O

Очевидно, хлор изменил степень окисления, потому что он оказался в соединениях, начиная с исходного элемента. Проверка всех степеней окисления показывает:

Хлор — это единственная вещь, которая изменила степень окисления. Он окислился или восстановился? Да! Оба! Один атом был восстановлен, потому что его степень окисления упала. Второй окислился.

Это хороший пример реакции диспропорционирования . Реакция диспропорционирования – это реакция, при которой одно вещество одновременно окисляется и восстанавливается.

 

Использование степеней окисления для идентификации окислителя и восстановителя

Это небольшое дополнение к последнему разделу. Если вы знаете, что окислялось, а что восстанавливалось, то вы можете легко определить, что такое окислитель и восстановитель.

Пример 1

Это реакция между ионами хрома (III) и металлическим цинком:

2Cr 3+  + Zn    2Cr 2+  +  Zn 2+

Хром перешел из степени окисления +3 в +2 и, таким образом, был восстановлен. Цинк перешел от нулевой степени окисления в элементе к +2. Он был окислен.

Так что же делает сокращение? Это цинк — цинк отдает электроны ионам хрома (III). Таким образом, цинк является восстановителем.

Точно так же вы можете решить, что окислителем должны быть ионы хрома (III), потому что они забирают электроны у цинка.

Пример 2

Это уравнение реакции между ионами манганата(VII) и ионами железа(II) в кислой среде. Это проработано ниже на странице.

MnO 4  +  8H +  +  5Fe 2+     Mn 2+  +  4H 2 O  +  5Fe 3+

При быстром рассмотрении видно, что ионы железа(II) окислились до ионов железа(III). Каждый из них потерял электрон, а их степень окисления увеличилась с +2 до +3.

Водород все еще находится в степени окисления +1 до и после реакции, но ионы манганата (VII) явно изменились. Если посчитать степень окисления марганца, то она упала с +7 до +2 — восстановление.

Итак, ионы железа(II) окислились, а ионы манганата(VII) восстановились.

Что восстановило ионы манганата(VII) — однозначно это ионы железа(II). Железо — единственное, что имеет измененную степень окисления. Таким образом, ионы железа (II) являются восстановителем.

Точно так же ионы манганата(VII) должны быть окислителями.

 

Использование степеней окисления для расчета пропорций реакции

Это иногда бывает полезно, когда вам нужно определить реакционные пропорции для использования в реакциях титрования, когда у вас недостаточно информации для расчета полного ионного уравнения.

Помните, что каждый раз, когда степень окисления изменяется на одну единицу, передается один электрон. Если степень окисления одного вещества в реакции падает на 2, это означает, что оно приобрело 2 электрона.

Что-то еще в реакции должно терять эти электроны. Любое понижение степени окисления одного вещества должно сопровождаться таким же повышением степени окисления чем-то другим.

 

Этот пример основан на информации из старого вопроса уровня AQA A’.

Ионы, содержащие церий в степени окисления +4, являются окислителями. (Они сложнее, чем просто Ce 4+ .) Они могут окислять ионы, содержащие молибден, от степени окисления +2 до +6 (от Mo 2+ до MoO 4 2-). В процессе церий восстанавливается до степени окисления +3 (Ce 3+ ). Каковы пропорции реакции?

Степень окисления молибдена увеличивается на 4. Это означает, что степень окисления церия должна упасть на 4, чтобы компенсировать это.

Но степень окисления церия в каждом из его ионов падает только с +4 до +3 — падение на 1. Таким образом, очевидно, что на каждый ион молибдена должно приходиться 4 иона церия.

Реагирующие пропорции: 4 церийсодержащих иона на 1 ион молибдена.

 

Или, если взять более распространенный пример, включающий ионы железа (II) и ионы манганата (VII). . .

Раствор манганата калия(VII), KMnO 4 , подкисленный разбавленной серной кислотой, окисляет ионы железа(II) до ионов железа(III). При этом ионы манганата(VII) восстанавливаются до ионов марганца(II). Используя степени окисления, составьте уравнение реакции.

Степень окисления марганца в манганат-ионе (VII) равна +7. Название говорит вам об этом, но попробуйте еще раз, просто для практики!

При переходе к ионам марганца (II) степень окисления марганца упала на 5. Каждый реагирующий ион железа (II) увеличивает свою степень окисления на 1. Это означает, что должно быть пять ионов железа (II), реагирующих на каждый ион манганата(VII).

Таким образом, левая часть уравнения будет:

MnO 4 + 5Fe 2+ + ?

Правая часть будет:

Mn 2+ + 5Fe 3+ + ?

После этого вам придется гадать, как сбалансировать оставшиеся атомы и заряды.

Как находить определитель матрицы 4 порядка: Найти определитель матрицы четвертого порядка

как считать по столбцу, как вычислить от треугольной матрицы

Определитель матрицы 4 порядка

Определение

Матрица представляет собой прямоугольную таблицу скаляров, то есть элементов некоторого поля, которая состоит из определенного числа столбцов и определенного числа строк.

Существует разные типы матриц. При рассмотрении данной темы важно уточнить понятия некоторых из них:

  • в том случае, когда матрица обладает единственным элементом, она является совпадающей со своим единственным скаляром;
  • квадратная матрица представляет собой такую матрицу, которая имеет равное число столбцов и строк.

Алгебраические действия с матрицами характеризуются определенным алгоритмом и порядком. Подобные операции отличаются от манипуляций с простыми числами. Кроме алгебраических действий, над матрицами выполняют и другие операции. К примеру, транспортирование матрицы.

Осторожно! Если преподаватель обнаружит плагиат в работе, не избежать крупных проблем (вплоть до отчисления). Если нет возможности написать самому, закажите тут.

В задачах часто встречаются примеры нахождения определителя матриц разных порядков. Матрицы первого, второго, третьего, четвертого и других порядков относят к квадратным матрицам.

Определение

Определитель или детерминант матрицы является определенным числом, которое можно поставить в соответствие некой квадратной матрице.

В том случае, когда элементы матрицы имеют вид действительных чисел, то и определитель является действительным числом. Определитель обозначают detA или |A|. Определитель первого порядка соответствует скаляру рассматриваемой матрицы.

Определители второго и третьего порядка определяются в соответствии со стандартным алгоритмом, то есть с помощью известных формул. Для того чтобы вычислить определитель больше третьего порядка, требуется ознакомиться с понятием минора матрицы (М).

Определение

Минор матрицы третьего порядка является определителем второго порядка, который получен из заданной матрицы третьего порядка путем вычеркивания i-ой степени и j-го столбца.

Примечание

Изучая материалы по теме матричного определителя, можно встретить термин «детерминант». Фактически данные понятия идентичны. Однако детерминант обладает множеством значений в других научных областях, что объясняет использование его русского перевода в математике.

Свойства определителя:

  • при перестановке местами двух строк или столбцов знак определителя меняется;
  • при умножении строки или столбца на число, весь определить также умножится на число;
  • в том случае, когда одну строку сколько угодно раз прибавляют или вычитают из другой, определитель не меняется;
  • при наличии одинаковых, пропорциональных или заполненных нулями двух строк определителя весь определитель равен нулю;
  • все перечисленные свойства справедливы в случае столбцов;
  • транспортирование матрицы не сопровождается изменением определителя;
  • определитель произведения матриц равен произведению определителей. {N (a_{1},a_{2},…a_{n})}}*a_{1a_{1}}a_{2a_{2}}…a_{na_{n}}\)

    Каждое слагаемое является произведением n-элементов, которые взяли по одному из каждой строки и каждого столбца, умноженное на (-1) в степени Np, определяемое, как количество инверсий.

    Геометрический смысл понятия заключается в том, что определитель представляет собой объем параллелепипеда, получаемый в том случае, если рассмотреть строки, как вектора, образующие ребра геометрической фигуры.

    При этом число строк или столбцов соответствует количеству векторов. Таким образом, в случае матрицы А = (2*2), получается двухмерный параллелограмм, а детерминант является площадью рассматриваемой фигуры. Если А = (3*3), то геометрическая фигура будет иметь вид трехмерного параллелепипеда, а определитель – являться его объемом.

    Правильная расстановка индексов в матрице

    Определение

    Индексы являются координатами элемента в системе.

    Каждый элемент обладает парой индексов:

    • первый – определяет строку;
    • второй – указывает столбец.

    Так как порядок представляет собой число строк или столбцов в квадратной матрице, то он определяется с помощью m-индекса нижней строки или n-индекса крайнего правого столбца. Данный метод используют при очень больших таблицах, когда считать строки или столбы неудобно.

    Матрица представляет собой таблицу, заполненную числами. Одной из ее ключевых характеристик является размерность, то есть число строк и столбцов, из которых она состоит. Как правило, говорят, что какая-то матрица A имеет размер \(\left[ m\times n \right]\), если в ней имеется m строк и n столбцов. Запись имеет следующий вид:

    \(A=\left[ m\times n \right]\)

    или  \(A=\left( {{a}_{ij}} \right),\quad 1\le i\le m;\quad 1\le j\le n.\)

    Существуют и другие обозначения для матрицы. В любом случае, при разных \(\left[ m\times n \right]\) и \({{a}_{ij}}\) возникает вопрос расстановки индексов. В этом случае целесообразно обратиться к обычной системе координат:

    Источник: berdov. com

    Данная система имеет начало координат (точка \( O=\left( 0;0 \right)\)) оси x и y, а каждая точка на плоскости однозначно определяется по координатам:

    \(A=\left( 1;2 \right)\)

    \(B=\left( 3;1 \right)\) и так далее.

    Следует поставить рассматриваемую систему рядом с матрицей, таким образом, чтобы начало координат совпадало с левым верхним углом, что существенно облегчит задачу определения индексов. Оси должны быть направлены так, чтобы охватить всю систему. При этом необходимо повернуть систему координат. Правильный вариант расположения представлен на рисунке:

    Источник: berdov.com

    Получается, что любая клетка матрицы обладает однозначными координатами х и у. К примеру, запись \({{a}_{24}}\) означает, что мы обращаемся к элементу с координатами x=2  и y=4. Размеры матрицы также однозначно заданы двумя числами:

    Источник: berdov.com

    Общая схема вычисления определителей

    Определение

    Теорема Лапласа о разложении определителя: пусть в матрице размера выбрано k строк или столбцов, причем . {i+j}}\cdot {{M}_{ij}}}\)

    Таким образом удалось вывести формулу для разложения определителя по строке. Аналогичный метод можно использовать для того, чтобы разложить определитель по столбцам.

    Данное следствие позволяет сделать несколько выводов:

    • методика подходит для строк и столбцов;
    • число слагаемых в разложении в любом случае составляет n;
    • вместо одного определителя \(\left[ n\times n \right]\) необходимо вычислить несколько определителей размера на единицу меньше \(\left[ \left( n-1 \right)\times \left( n-1 \right) \right].\)

    Практическое применение алгоритма можно продемонстрировать, решая задачу по нахождению определителя матрицы:

    \(\left| \begin{matrix} 1 &amp; 2 &amp; 3 \\ 4 &amp; 5 &amp; 6 \\ 7 &amp; 8 &amp; 9 \\\end{matrix} \right|\)

    В первую очередь необходимо разложить данный определитель по первой строке:

    \(\begin{align} \left| A \right|=1\cdot {{\left( -1 \right)}^{1+1}}\cdot \left| \begin{matrix} 5 &amp; 6 \\ 8 &amp; 9 \\\end{matrix} \right|+ &amp; \\ 2\cdot {{\left( -1 \right)}^{1+2}}\cdot \left| \begin{matrix} 4 &amp; 6 \\ 7 &amp; 9 \\\end{matrix} \right|+ &amp; \\ 3\cdot {{\left( -1 \right)}^{1+3}}\cdot \left| \begin{matrix} 4 &amp; 5 \\ 7 &amp; 8 \\\end{matrix} \right|= &amp; \\\end{align}\)

    Отсюда следует:

    \(\begin{align} &amp;=1\cdot \left( 45-48 \right)-2\cdot \left( 36-42 \right)+3\cdot \left( 32-35 \right)= \\ &amp;=1\cdot \left( -3 \right)-2\cdot \left( -6 \right)+3\cdot \left( -3 \right)=0. {4+4}}\cdot \left| \begin{matrix} 0 &amp; 1 &amp; 1 \\ 1 &amp; 0 &amp; 1 \\ 1 &amp; 1 &amp; 0 \\\end{matrix} \right| &amp; \\\end{align}\)

    От пары слагаемых можно избавиться. В результате останется пара определителей 3х3:

    \(\begin{align} &amp; \left| \begin{matrix} 0 &amp; 1 &amp; 1 \\ 1 &amp; 1 &amp; 0 \\ 1 &amp; 1 &amp; 1 \\\end{matrix} \right|=0+0+1-1-1-0=-1; \\ &amp; \left| \begin{matrix} 0 &amp; 1 &amp; 1 \\ 1 &amp; 0 &amp; 1 \\ 1 &amp; 1 &amp; 1 \\\end{matrix} \right|=0+1+1-0-0-1=1. \\\end{align}\)

    \(\left| \begin{matrix} 0 &amp; 1 &amp; 1 &amp; 0 \\ 1 &amp; 0 &amp; 1 &amp; 1 \\ 1 &amp; 1 &amp; 0 &amp; 1 \\ 1 &amp; 1 &amp; 1 &amp; 0 \\\end{matrix} \right|=1\cdot \left( -1 \right)+\left( -1 \right)\cdot 1=-2\)

    В результате вычислений получен ответ: -2

    Метод понижения порядка

    Упростить расчеты при вычислении определителей можно, используя их свойства. Например, требуется вычислить определитель:

    \(\begin{vmatrix}6&3&8&-4\\5&6&4&2\\0&3&4&2\\4&1&-4&6\end{vmatrix}\)

    Следует вынести из третьего столбца множитель 4:

    \(\begin{vmatrix}6&amp;3&amp;8&amp;-4\\5&amp;6&amp;4&amp;2\\0&amp;3&amp;4&amp;2\\4&amp;1&amp;-4&amp;6\end{vmatrix}=4\cdot\begin{vmatrix}6&amp;3&amp;2&amp;-4\\5&amp;6&amp;1&amp;2\\0&amp;3&amp;1&amp;2\\4&amp;1&amp;-1&amp;6\end{vmatrix}\)

    Далее следует вынести из четвертого столбца множитель 2:

    \(4\cdot\begin{vmatrix}6&amp;3&amp;2&amp;-4\\5&amp;6&amp;1&amp;2\\0&amp;3&amp;1&amp;2\\4&amp;1&amp;-1&amp;6\end{vmatrix}=4\cdot2\cdot\begin{vmatrix}6&amp;3&amp;2&amp;-2\\5&amp;6&amp;1&amp;1\\0&amp;3&amp;1&amp;1\\4&amp;1&amp;-1&amp;3\end{vmatrix}=8\cdot\begin{vmatrix}6&amp;3&amp;2&amp;-2\\5&amp;6&amp;1&amp;1\\0&amp;3&amp;1&amp;1\\4&amp;1&amp;-1&amp;3\end{vmatrix}\)

    Затем можно прибавить к строке №1 строку №2, умноженную на (-2):

    \(8\cdot\begin{vmatrix}6&amp;3&amp;2&amp;-2\\5&amp;6&amp;1&amp;1\\0&amp;3&amp;1&amp;1\\4&amp;1&amp;-1&amp;3\end{vmatrix}=8\cdot\begin{vmatrix}-4&amp;-9&amp;0&amp;-4\\5&amp;6&amp;1&amp;1\\0&amp;3&amp;1&amp;1\\4&amp;1&amp;-1&amp;3\end{vmatrix}\)

    Следующим шагом будет сложение строки №3 и строки №2, умноженной на (-1):

    \(8\cdot\begin{vmatrix}-4&amp;-9&amp;0&amp;-4\\5&amp;6&amp;1&amp;1\\0&amp;3&amp;1&amp;1\\4&amp;1&amp;-1&amp;3\end{vmatrix}=8\cdot\begin{vmatrix}-4&amp;-9&amp;0&amp;-4\\5&amp;6&amp;1&amp;1\\-5&amp;-3&amp;0&amp;0\\4&amp;1&amp;-1&amp;3\end{vmatrix}\)

    Далее нужно прибавить к четвертой строке вторую, умноженную на 1:

    \(8\cdot\begin{vmatrix}-4&amp;-9&amp;0&amp;-4\\5&amp;6&amp;1&amp;1\\-5&amp;-3&amp;0&amp;0\\4&amp;1&amp;-1&amp;3\end{vmatrix}=8\cdot\begin{vmatrix}-4&amp;-9&amp;0&amp;-4\\5&amp;6&amp;1&amp;1\\-5&amp;-3&amp;0&amp;0\\9&amp;7&amp;0&amp;4\end{vmatrix}\)

    Можно разложить определитель по третьему столбцу:

    \(8\cdot\begin{vmatrix}-4&amp;-9&amp;0&amp;-4\\5&amp;6&amp;1&amp;1\\-5&amp;-3&amp;0&amp;0\\9&amp;7&amp;0&amp;4\end{vmatrix}=8\cdot1\cdot(-1)^{2+3}\begin{vmatrix}-4&amp;-9&amp;-4\\-5&amp;-3&amp;0\\9&amp;7&amp;4\end{vmatrix}=8\cdot(-1)^{5}\begin{vmatrix}-4&amp;-9&amp;-4\\-5&amp;-3&amp;0\\9&amp;7&amp;4\end{vmatrix}=-8\begin{vmatrix}-4&amp;-9&amp;-4\\-5&amp;-3&amp;0\\9&amp;7&amp;4\end{vmatrix}\)

    Если прибавить к первой строке третью, умноженную на 1, получим:

    \(-8\begin{vmatrix}-4&amp;-9&amp;-4\\-5&amp;-3&amp;0\\9&amp;7&amp;4\end{vmatrix}=-8\begin{vmatrix}5&amp;-2&amp;0\\-5&amp;-3&amp;0\\9&amp;7&amp;4\end{vmatrix}\)

    Затем необходимо разложить определитель по третьему столбцу и вычислить его:

    \(-8\begin{vmatrix}5&amp;-2&amp;0\\-5&amp;-3&amp;0\\9&amp;7&amp;4\end{vmatrix}=-8\cdot4\cdot(-1)^{3+3}\begin{vmatrix}5&amp;-2\\-5&amp;-3\end{vmatrix}=-32\cdot(-1)^{6}\begin{vmatrix}5&amp;-2\\-5&amp;-3\end{vmatrix}=-32\begin{vmatrix}5&amp;-2\\-5&amp;-3\end{vmatrix}\)

    Далее следует прибавить к строке №2 строку №1, умноженную на 1:

    \(-32\begin{vmatrix}5&amp;-2\\-5&amp;-3\end{vmatrix}=-32\begin{vmatrix}5&amp;-2\\0&amp;-5\end{vmatrix}\)

    При разложении определителя по столбцу №1 и замене определителя 1-го порядка единственным его элементом получим:

    \(-32\begin{vmatrix}5&amp;-2\\0&amp;-5\end{vmatrix}=-32\cdot5\cdot(-1)^{1+1}\cdot(-5)=-32\cdot5\cdot1\cdot(-5)=800\)

    Приведение к треугольному виду

    Методика заключается в приведении определителя к треугольнику. После этого остается вычислить произведение элементов, расположенных на главной диагонали. С помощью данного способа вычислим определитель:

    \(\begin{vmatrix}4&amp;-2&amp;0&amp;5\\3&amp;2&amp;-2&amp;1\\-2&amp;1&amp;3&amp;-1\\2&amp;3&amp;-6&amp;-3\end{vmatrix}\)

    В первую очередь следует поменять местами первую и третью строки:

    \(\begin{vmatrix}4&amp;-2&amp;0&amp;5\\3&amp;2&amp;-2&amp;1\\-2&amp;1&amp;3&amp;-1\\2&amp;3&amp;-6&amp;-3\end{vmatrix}=-\begin{vmatrix}-2&amp;1&amp;3&amp;-1\\3&amp;2&amp;-2&amp;1\\4&amp;-2&amp;0&amp;5\\2&amp;3&amp;-6&amp;-3\end{vmatrix}\)

    Далее первую строку, умноженную на 1, можно прибавить к четвертой:

    \(-\begin{vmatrix}-2&amp;1&amp;3&amp;-1\\3&amp;2&amp;-2&amp;1\\4&amp;-2&amp;0&amp;5\\2&amp;3&amp;-6&amp;-3\end{vmatrix}=-\begin{vmatrix}-2&amp;1&amp;3&amp;-1\\3&amp;2&amp;-2&amp;1\\4&amp;-2&amp;0&amp;5\\0&amp;4&amp;-3&amp;-4\end{vmatrix}\)

    Затем нужно к строке №3 прибавить строку №1, умноженную на 2:

    \(-\begin{vmatrix}-2&amp;1&amp;3&amp;-1\\3&amp;2&amp;-2&amp;1\\4&amp;-2&amp;0&amp;5\\0&amp;4&amp;-3&amp;-4\end{vmatrix}=-\begin{vmatrix}-2&amp;1&amp;3&amp;-1\\3&amp;2&amp;-2&amp;1\\0&amp;0&amp;6&amp;3\\0&amp;4&amp;-3&amp;-4\end{vmatrix}\)

    Следующий шаг – умножение второй строки на 2:

    \(\begin{vmatrix}-2&amp;1&amp;3&amp;-1\\3&amp;2&amp;-2&amp;1\\0&amp;0&amp;6&amp;3\\0&amp;4&amp;-3&amp;-4\end{vmatrix}=-\frac{1}{2}\begin{vmatrix}-2&amp;1&amp;3&amp;-1\\6&amp;4&amp;-4&amp;2\\0&amp;0&amp;6&amp;3\\0&amp;4&amp;-3&amp;-4\end{vmatrix}\)

    Сложим вторую и первую строку, умноженную на 3:

    \(-\frac{1}{2}\begin{vmatrix}-2&amp;1&amp;3&amp;-1\\6&amp;4&amp;-4&amp;2\\0&amp;0&amp;6&amp;3\\0&amp;4&amp;-3&amp;-4\end{vmatrix}=-\frac{1}{2}\begin{vmatrix}-2&amp;1&amp;3&amp;-1\\0&amp;7&amp;5&amp;-1\\0&amp;0&amp;6&amp;3\\0&amp;4&amp;-3&amp;-4\end{vmatrix}\)

    После умножения строки №4 на 7 получим:

    \(-\frac{1}{2}\begin{vmatrix}-2&amp;1&amp;3&amp;-1\\0&amp;7&amp;5&amp;-1\\0&amp;0&amp;6&amp;3\\0&amp;4&amp;-3&amp;-4\end{vmatrix}=-\frac{1}{2}\cdot\frac{1}{7}\begin{vmatrix}-2&amp;1&amp;3&amp;-1\\0&amp;7&amp;5&amp;-1\\0&amp;0&amp;6&amp;3\\0&amp;28&amp;-21&amp;-28\end{vmatrix}\)

    Затем следует прибавить к четвертой строке вторую строку, умноженную на (-4):

    \(-\frac{1}{2}\cdot\frac{1}{7}\begin{vmatrix}-2&amp;1&amp;3&amp;-1\\0&amp;7&amp;5&amp;-1\\0&amp;0&amp;6&amp;3\\0&amp;28&amp;-21&amp;-28\end{vmatrix}=-\frac{1}{2}\cdot\frac{1}{7}\begin{vmatrix}-2&amp;1&amp;3&amp;-1\\0&amp;7&amp;5&amp;-1\\0&amp;0&amp;6&amp;3\\0&amp;0&amp;-41&amp;-24\end{vmatrix}\)

    В результате смены мест столбцов №3 и №4 получим:

    \(-\frac{1}{2}\cdot\frac{1}{7}\begin{vmatrix}-2&amp;1&amp;3&amp;-1\\0&amp;7&amp;5&amp;-1\\0&amp;0&amp;6&amp;3\\0&amp;0&amp;-41&amp;-24\end{vmatrix}=\frac{1}{2}\cdot\frac{1}{7}\begin{vmatrix}-2&amp;1&amp;-1&amp;3\\0&amp;7&amp;-1&amp;5\\0&amp;0&amp;3&amp;6\\0&amp;0&amp;-24&amp;-41\end{vmatrix}\)

    После того, как третья строка, умноженная на 8, будет прибавлена к четвертой строке, получится вычислить определитель:

    \(\frac{1}{2}\cdot\frac{1}{7}\begin{vmatrix}-2&amp;1&amp;-1&amp;3\\0&amp;7&amp;-1&amp;5\\0&amp;0&amp;3&amp;6\\0&amp;0&amp;-24&amp;-41\end{vmatrix}=\frac{1}{2}\cdot\frac{1}{7}\begin{vmatrix}-2&amp;1&amp;-1&amp;3\\0&amp;7&amp;-1&amp;5\\0&amp;0&amp;3&amp;6\\0&amp;0&amp;0&amp;7\end{vmatrix}=\frac{1}{2}\cdot\frac{1}{7}\cdot(-2)\cdot7\cdot3\cdot7=-21\)

    Решение высшей математики онлайн

    ‹— Назад

    С понятием определителя мы уже сталкивались при изучении векторного произведения в разделе 10. Там были введены определители матриц второго и третьего порядка. В этом разделе мы дадим определение определителя квадратной матрицы любого порядка. Это определение будет рекуррентным, то есть чтобы установить, что такое определитель матрицы порядка , нужно уже знать, что такое определитель матрицы порядка . Такое рекуррентное определение и было использовано для введения определителя матрицы третьего порядка . Отметим также, что определитель существует только у квадратных матриц.

    Определитель квадратной матрицы будем обозначать или .

            Определение 14.6   Определителем квадратной матрицы второго порядка называется число . Определителем квадратной матрицы порядка , , называется число

    где  — определитель матрицы порядка , полученной из матрицы вычеркиванием первой строки и столбца с номером .         

    Легко проверить, что это определение для определителей второго и третьего порядка совпадает с данным ранее в разделе 10.

    Для наглядности запишем, как можно вычислить определитель матрицы четвертого порядка:


            Замечание 14.7   Реальное вычисление определителей для матриц выше третьего порядка на основе определения используется в исключительных случаях. Как правило, вычисление ведется по другим алгоритмам, которые будут рассмотрены позже и которые требуют меньше вычислительной работы.         

            Замечание 14.8   В определении 14.6 было бы точнее сказать, что определитель есть функция, определенная на множестве квадратных матриц порядка и принимающая значения в множестве чисел.         

            Замечание 14.9   В литературе вместо термина «определитель» используется также термин «детерминант», имеющий тот же самый смысл. От слова «детерминант» и появилось обозначение .         

    Рассмотрим некоторые свойства определителей, которые сформулируем в виде предложений.

            Предложение 14.6   При транспонировании матрицы определитель не меняется, то есть .     

            Предложение 14.7   Определитель произведения квадратных матриц равен произведению определителей сомножителей, то есть .     

            Предложение 14.8   Если в матрице поменять местами две строки, то ее определитель сменит знак.     

    Ввиду ограниченности курса доказательства этих трех свойств мы опускаем. Читатель может найти их в учебниках по линейной алгебре [3], [5] или же может без особых сложностей проверить их на матрицах второго и третьего порядков.

            Предложение 14.9   Если матрица имеет две одинаковые строки, то ее определитель равен нулю.

            Доказательство.     Поменяем местами две одинаковые строки. В силу предложения 14.8 определитель сменит знак. С другой стороны, так как строки были одинаковыми, то матрица не изменилась и, следовательно, не изменился и ее определитель. Получим, что , откуда следует, что .     

    В дальнейшем нам потребуется складывать строки и умножать строку на число. Эти действия над строками (столбцами) мы будем выполнять так же, как действия над матрицами-строками (матрицами-столбцами), то есть поэлементно. Результатом будет служить строка (столбец), как правило, не совпадающая со строками исходной матрицы. При наличии операций сложения строк (столбцов) и умножения их на число мы можем говорить и о линейных комбинациях строк (столбцов), то есть суммах с числовыми коэффициентами.

            Предложение 14.10   Если строку матрицы умножить на число , то ее определитель умножится на это число.

            Доказательство.     Пусть — исходная матрица,  — матрица, полученная из умножением первой строки на число :

    Тогда

    где — определитель матрицы, полученной из матрицы или, что то же самое, из матрицы вычеркиванием первой строки и -ого столбца.

    Вынесем множитель за знак суммы и получим

    Пусть теперь матрица получается из матрицы умножением -ой строки на число . Поменяем местами первую и -ую строки в матрице и то же самое проделаем в матрице . Получим две новых матрицы и . По предложению 14.8

    (14.10)

    Очевидно, что матрица получается из матрицы умножением первой строки на число . Как только что было доказано, . Таким образом, из второго равенства (14.10) находим , отсюда с помощью первого равенства (14.10) получаем .     

            Предложение 14.11   Если матрица содержит нулевую строку, то ее определитель равен нулю.

            Доказательство.     Нулевую строку можно рассматривать как строку из единиц, умноженную на число ноль. По предложению 14.10 определитель такой матрицы равен нулю, умноженному на определитель матрицы, содержащей строку из единиц. Результат такого умножения всегда будет ноль.     

            Предложение 14.12   Если одна из строк матрицы равна другой, умноженной на число (строки пропорциональны), то определитель матрицы равен нулю.

            Доказательство.    По предложению 14.10 определитель исходной матрицы равен числу , умноженному на определитель матрицы, у которой есть две одинаковые строки. По предложению 14.9 определитель последней матрицы равен нулю. Поэтому и определитель исходной матрицы равен нулю.     

            Предложение 14.13   Пусть в матрице -ая строка имеет вид . Тогда , где матрица получается из матрицы заменой -ой строки на строку , а матрица  — заменой -ой строки на строку .

            Доказательство.     Пусть первая строка матрицы имеет вид . Тогда


    Для случая утверждение доказано.

    Пусть . Обозначим через , , матрицы , , и , в которых поменяли местами первую и -ую строки. По только что доказанному (для ) утверждению . По предложению 14.8 , , . Следовательно, . Умножив обе части последнего равенства на , получим требуемое утверждение.     

            Предложение 14.14   Если к одной из строк матрицы добавить другую, умноженную на число, то определитель матрицы не изменится.

            Доказательство.     Пусть к -ой строке матрицы прибавлена -ая строка, умноженная на число . Новую матрицу обозначим . В матрице элементы -ой строки имеют вид . По предложению 14.13 , где  — матрица, полученная из матрицы заменой -ой строки на -ую строку, умноженную на число . По предложению 14.12 , то есть .     

            Предложение 14.15   Если одна из строк матрицы является линейной комбинацией других ее строк, то определитель матрицы равен нулю.

            Доказательство.     По предложению 14.13 определитель исходной матрицы равен сумме определителей матриц, в каждой из которых есть пропорциональные строки. По предложению 14.12 все эти определители равны нулю. Следовательно, и определитель исходной матрицы тоже равен нулю.     

            Определение 14.7   Алгебраическим дополнением к элементу матрицы называется число, равное , где  — определитель матрицы, полученной из матрицы вычеркиванием -ой строки и -ого столбца.         

    Алгебраическое дополнение к элементу матрицы обозначается .

            Пример 14.4   Пусть . Тогда

            

            Замечание 14.10   Используя алгебраические дополнения, определение 14.6 определителя можно записать так:

            

            Предложение 14.16   Разложение определителя по произвольной строке. Для определителя матрицы справедлива формула

            Доказательство.      Если , положим . Пусть . Тогда -ую строку поменяем местами со строкой с номером . Определитель сменит знак. Затем строку с номером поменяем местами со строкой с номером . Определитель снова сменит знак. Процесс перестановки строк будем продолжать до тех пор, пока -ая строка матрицы не станет первой строкой новой матрицы, которую мы обозначим . Отметим, что в матрице , начиная со второй строки, стоят строки матрицы , причем порядок их следования не изменился.

    При переходе от матрицы к матрице определитель сменит знак раз (проверьте для случая ). Таким образом

    (14.11)

    Это соотношение верно и при . По определению 14.6 определителя,

    где  — определитель матрицы, полученной из матрицы вычеркиванием первой строки и -ого столбца. Первая строка матрицы совпадает с -ой строкой матрицы , поэтому . Результат вычеркивания в матрице первой строки и -ого столбца будет таким же, как при вычеркивании в матрице -ой строки и -ого столбца. Поэтому , где  — определитель матрицы, полученной при вычеркивании в матрице -ой строки и -ого столбца. Следовательно,

    В силу равенства (14.11) получим

    По определению 14.7 алгебраического дополнения получим . Тогда из предыдущего равенства вытекает

    что и требовалось доказать.     

            Пример 14.5   Вычислите .

    Решение. Воспользуемся разложением по третьей строке, так выгоднее, поскольку в третьей строке два числа из трех — нули. Получим


            

            Предложение 14.17   Для квадратной матрицы порядка при выполнено соотношение

    (14.12)

            Доказательство.      Пусть  — матрица, полученная из матрицы , в которой -ая строка заменена -ой строкой этой же матрицы, а сама -ая строка осталась без изменения. Таким образом, в матрице есть две одинаковые строки и в силу  предложения 14.9 .

    С другой стороны, используя разложение определителя по -ой строке (предложение 14.16), получим

    где  — алгебраическое дополнение к элементу . Так как все строки матрицы , кроме -ой, совпадают со строками матрицы , то . Так как по построению матрицы , то

    Так как , то равенство (14.12) доказано.     

            Предложение 14.18   Все свойства определителя, сформулированные для строк ( предложения 14.8-14.17), справедливы и для столбцов, в частности, справедливо разложение определителя по -ому столбцу

    (14.13)

    и равенство

    при .

            Доказательство.     В силу  предложения 14.6 определитель не меняется при транспонировании матрицы, а ее столбцы становятся строками транспонированной матрицы, для которой доказываемые свойства имеют место.     

            Предложение 14.19   Определитель треугольной матрицы равен произведению элементов ее главной диагонали.

            Доказательство.     Воспользуемся индукцией по порядку матрицы. Для :

    утверждение верно. Предположим, что доказываемое утверждение верно для матриц порядка . Покажем, что оно верно для матрицы порядка .

    Если — верхняя треугольная матрица, то используем разложение по первому столбцу (равенство (14.13) при ):


    Справа стоит определитель треугольной марицы порядка . По предположению индукции этот определитель равен . Поэтому .

    Если  — нижняя треугольная матрицы, то нужно воспользоваться разложением по первой строке. В остальном рассуждения аналогичны.

    Итак, утверждение верно для матрицы порядка . Предложение доказано.     

            Следствие 14.1   Определитель единичной матрицы равен единице, .     

    Перечисленные выше свойства позволяют находить определители матриц достаточно высоких порядков при сравнительно небольшом объеме вычислений. Алгоритм вычислений следующий.

    Алгоритм создания нулей в столбце.

    Пусть требуется вычислить определитель матрицы порядка . Если , то поменяем местами первую строку и любую другую, в которой первый элемент не нуль. В результате определитель , будет равен определителю новой матрицы с противоположным знаком. Если же первый элемент каждой строки равен нулю, то матрица имеет нулевой столбец и по  предложениям 14.11, 14.18 ее определитель равен нулю.

    Итак, считаем, что уже в исходной матрице . Первую строку оставляем без изменений. Прибавим ко второй строке первую строку, умноженную на число . Тогда первый элемент второй строки будет равен

    Остальные элементы новой второй строки обозначим , . Определитель новой матрицы по  предложению 14.14 равен .

    Первую строку умножим на число и прибавим к третьей. Первый элемент новой третьей строки будет равен

    Остальные элементы новой третьей строки обозначим , . Определитель новой матрицы по  предложению 14.14 равен .

    Процесс получения нулей вместо первых элементов строк продолжим дальше. Наконец, первую строку умножим на число и прибавим к последней строке. В результате получается матрица, обозначим ее , которая имеет вид

    причем . Для вычисления определителя матрицы используем разложение по первому столбцу

    Так как , то

    В правой части стоит определитель матрицы порядка . К нему применим тот же алгоритм, и вычисление определителя матрицы сведется к вычислению определителя матрицы порядка . Процесс повторяем до тех пор, пока не дойдем до определителя второго порядка, который вычисляется по определению.     

    Если матрица не обладает какими-то специфическими свойствами, то заметно уменьшить объем вычислений по сравнению с предложенным алгоритмом не удается. Еще одна хорошая сторона этого алгоритма — по нему легко составить программу для компьютера для вычисления определителей матриц больших порядков. В стандартных программах вычисления определителей используется этот алгоритм с не принципиальными изменениями, связанными с минимизацией влияния ошибок округления и погрешностей входных данных при вычислениях компьютера.

            Пример 14.6   Вычислите определитель матрицы

    .

    Решение. Первую строку оставляем без изменения. Ко второй строке прибавляем первую, умноженную на число :

    Определитель не меняется. К третьей строке прибавляем первую, умноженную на число :

    Определитель не меняется. К четвертой строке прибавляем первую, умноженную на число :

    Определитель не меняется. В результате получаем

    По тому же алгоритму считаем определитель матрицы порядка 3, стоящий справа. Первую строку оставляем без изменений, ко второй строке прибавляем первую, умноженную на число :

    К третьей строке прибавляем первую, умноженную на число :

    В результате получаем


    Ответ. .         

            Замечание 14.11   Внимательный читатель, наверное, отметил, что хотя при вычислениях использовались дроби, результат оказался целым числом. Действительно, используя свойства определителей и то, что исходные числа — целые, операций с дробями можно было бы избежать. Но в инженерной практике числа крайне редко бывают целыми. Поэтому, как правило, элементы определителя будут десятичными дробями и применять какие-то ухищрения для упрощения вычислений нецелесообразно.         

    Математика, вышка, высшая математика, математика онлайн, вышка онлайн, онлайн математика, онлайн решение математики, ход решения, процес решения, решение, задачи, задачи по математике, математические задачи, решение математики онлайн, решение математики online, online решение математики, решение высшей математики, решение высшей математики онлайн, матрицы, решение матриц онлайн, векторная алгебра онлайн, решение векторов онлайн, система линейных уравнений, метод Крамера, метод Гаусса, метод обратной матрицы, уравнения, системы уравнений, производные, пределы, интегралы, функция, неопределенный интеграл, определенный интеграл, решение интегралов, вычисление интегралов, решение производных, интегралы онлайн, производные онлайн, пределы онлайн, предел функции, предел последовательности, высшие производные, производная неявной функции

    Определитель матрицы 4×4 – Как вычислить определитель матрицы 4×4?

    Определитель матрицы 4×4 — это уникальное число, которое также рассчитывается по определенной формуле. Если порядок матрицы равен n x n, то это квадратная матрица. Итак, здесь 4×4 — это квадратная матрица, состоящая из четырех строк и четырех столбцов. Если A — квадратная матрица, то определитель матрицы A представлен как |A|.

     

    Найдите определитель матрицы 4×4? мы будем использовать нормальный метод, есть формула определителя матрицы 4 × 4, которую мы обычно используем для нахождения определителя матрицы 3 × 3.

     

    Определение определителя

    Самый простой способ — сформулировать определитель, учитывая верхний ряд элементов и соответствующие миноры. Возьмите первый элемент верхней строки и затем умножьте его на минор, после чего вычтите произведение второго элемента на минор. Продолжайте поочередно складывать и вычитать произведение каждого элемента верхней строки с заданным минором, пока не будут учтены все элементы верхней строки.

     

    Детерминанты также играют очень важную роль при нахождении обратной матрицы, а также при решении систем линейных уравнений. В следующей части мы также предполагаем, что у нас есть квадратная матрица (m равно n). Определитель матрицы A будет стоять через det(A) или |A|. Сначала вводится определитель матрицы 2×2 и 3×3, затем ставится случай n×n.

    Что такое Матрица?

    Прежде чем изучать операции над матрицей, давайте обсудим, что такое матрица. Матрица может быть определена как прямоугольный массив чисел или символов, которые обычно располагаются в строках и столбцах. Порядок матрицы также может быть определен как количество строк и количество. столбцов. Элементы также являются числами в матрице, и каждое из чисел называется элементом. Множественное число слова матрица известно как матрицы.

    Размер матрицы называется матрицей «n на m» и записывается как m×n, где n — количество строк, а m — количество столбцов. Например, у нас есть матрица 3×2, потому что количество строк здесь равно 3, а количество столбцов здесь равно 2.

    [Изображение скоро будет загружено]

    Размеры матрицы также может быть определено как количество строк и столбцов матрицы в указанном порядке. Поскольку приведенная выше матрица А имеет 2 строки и 3 столбца, она известна как матрица 2 × 3.

     

    Ярлык для определения ранга матрицы

    Общее количество линейно независимых векторов в матрице равно общему количеству ненулевых строк в ее строке, присутствующей в ступенчатой ​​матрице. Итак, чтобы узнать ранг матрицы, мы должны в общем случае преобразовать матрицу в ее эшелонированную форму строк, а затем подсчитать общее количество ненулевых строк.

     

    Символ определителя

    Символ определителя представляет собой две вертикальные линии с обеих сторон.

    Пример:

    |А| обозначает определитель матрицы A

    (тот же символ, что и абсолютное значение)

    Для матрицы 2×2

    Для матрицы 2×2 (2 строки, 2 столбца):

    A = \[\begin {bmatrix} a & b \\ c & d \end{bmatrix}\]

    Определитель записывается как:

    |A| = ad − bc

    Очень легко запомнить, когда вы думаете о кресте:

    Определитель матрицы 2×2

    Если A — произвольная матрица 2×2 A, элементы задаются следующим образом:

    A = \[\begin{bmatrix} a_{11} & a_{12} \ \ a_{21} & a_{22} \end{bmatrix}\]

    , то определитель матрицы a и этой матрицы представляется следующим образом:

    det(A) = lAl = \[\begin{vmatrix} a_ {11} & a_{12} \\ a_{21} & a_{22} \end{vmatrix}\] = a\[_{11}\]a\[_{22}\] — a\[_ {21}\]a\[_{12}\]

    Для матрицы 3×3

    Для матрицы 3×3 (3 строки, 3 столбца):

    A = \[\begin{bmatrix} a & b & c \\ d & e & f \\ g & h & i \end{bmatrix}\]

    Определитель записывается как:

    |A| будет равно a(ei − fh) − b(di − fg)  + c(dh − eg)

    Это может показаться сложным, но есть известная закономерность:

    [Изображение скоро будет загружено]

    To вычислить определитель матрицы 3×3 вот точки:

    • Умножить a на определитель матрицы 2×2, которая не находится ни в строке, ни в столбце a.

    • Также для b, и для c тоже

    • Сложите их, но также помните о минусе перед b

    Что касается формулы (помните, что вертикальные черточки || определитель»):

    lAl = a . \[\begin{vmatrix} e & f \\ h & i \end{vmatrix}\] — b . \[\begin{vmatrix} d & f \\ g & i \end{vmatrix}\] + c . \[\begin{vmatrix} d & e \\ g & h \end{vmatrix}\]

    Как вычислить определитель матрицы 4×4?

    Прежде чем мы попытаемся найти определитель матрицы 4×4, давайте сначала проверим несколько условий, приведенных ниже.

    • Если присутствует какое-либо условие, где определитель матрицы 4×4 может быть равен 0 (например, полная строка или полный столбец равен 0)

    • возможный.

    • Если элементы матрицы одинаковы, но затем переупорядочены в любом из столбцов или строк.

    В любом из трех случаев, приведенных выше, используются соответствующие методы вычисления определителей 3 x 3. Мы меняем строку или столбец, чтобы заполнить их 0, за исключением одного элемента. Определитель матрицы 4 × 4 будет эквивалентен произведению этого элемента и его кофактора. В этой ситуации кофактор представляет собой детерминант 3 × 3, который оценивается по своей конкретной формуле.

    Как рассчитать определитель 4X4? Посчитайте с нашим экспертом

    Мы продолжаем готовить уроки по математике. Надеюсь, это будет полезно для вас, и вам не нужно будет спрашивать наших экспертов что-то вроде «Сделай мне домашнее задание по математике, пожалуйста! Я застрял..» Это вторая часть нашего руководства, объясняющая, как вычислять определители. Нас просят вычислить определитель следующей матрицы 4 × 4:

    A=\begin{pmatrix}-1 & {1} & 4 & 2 \\2 & -1 & 2 & 5 \\1  &2  & 3& 4\\3& 4& -1 & 2\end{pmatrix} 9{4+1}a_{41}M_{41}\\&=a_{11}M_{11}-a_{21}M_{21}+a_{31}M_{31}-a_{41}M_{ 41}=-1\cdot M_{11}-2 \cdot M_{21}+1\cdot M_{31}-3 \cdot M_{41} \end{aligned}

    Вот видеоверсия этого руководства:

    Чтобы найти значение \Delta , нам нужно вычислить миноры M_{11}, M_{21}, M_{31}, M_{41}. Это определители порядка 3×3:

    Вычислим их таким же образом, но выбрав первую строку. Это означает, что мы устанавливаем i=1, в то время как j меняется от 1 до 3. Обратите внимание, что на этот раз верхний предел равен 3 вместо 4 для исходного определителя, потому что это определители размера 3×3. Как правило, мы можем выбрать любую строку или столбец:

    \begin{align} M_{11}&=\begin{vmatrix}-1&2 &5 \\2& 3 &4 \\4&-1 &2\end{vmatrix}=-1 \cdot\begin{vmatrix}3&4 \\ -1&2 \end{vmatrix}-2\cdot\begin{vmatrix}2&4 \\4&2 \end{vmatrix}+5\begin{vmatrix}2&3 \\4&-1\end{vmatrix}\\&=-1\ cdot (6+4)-2\cdot (4-16)+5\cdot(-2-12)=-10+24-70=-56\end{aligned}

    Аналогично вычисляем остальные три минора:

    \begin{выровнено} M_{21}&=\begin{vmatrix}1&4&2 \\2& 3 &4 \\4&-1 &2\end{vmatrix}=1 \cdot\begin{vmatrix}3&4 \\-1&2 \ end{vmatrix}-4\cdot\begin{vmatrix}2&4 \\4&2 \end{vmatrix}+2\begin{vmatrix}2&3 \\4&-1\end{vmatrix}\\&=1\cdot (6+ 4)-4\cdot (4-16)+2\cdot (-2-12)=10+48-28=30\end{выровнено}

    \begin{align} M_{31}&=\begin{vmatrix}1&4&2 \\-1& 2 &5 \\4&-1 &2\end{vmatrix}=1 \cdot\begin{vmatrix}2&5 \\-1&2 \end{vmatrix}-4\cdot\begin{vmatrix}-1&5 \\4&2 \end{vmatrix}+2\begin{vmatrix}-1&2 \\4&-1\end{vmatrix}\\&=1\cdot (4+5)-4\cdot (-2-20)+2\cdot (1-8)=9+88-14=83\end{выровнено}

    M_{41}=\begin{vmatrix}1&4&2 \\-1& 2 &5 \\2&3 &4\end{vmatrix}=1\cdot (8-15)-4\cdot (-4-10)+2\cdot (-3-4)=-7+56- 14=35

    Наконец, у нас есть все необходимые данные для нахождения нужного определителя. Подставим значения в выражение для \det A:

    \begin{aligned} \Delta& =-1\cdot M_{11}-2 \cdot M_{21}+1\cdot M_{31}-3 \cdot M_{41}\\&=-1\cdot (-56)-2\cdot 30+1\cdot 83-3\cdot 35=56-60+83-105=-26\end{выровнено}

    Таким образом , ответ:

    \det A =-26

    Подводим итоги. Если вас просят вычислить определитель какой-либо матрицы, прежде всего убедитесь, что вы имеете дело с квадратной матрицей, т.е. количество строк и количество столбцов совпадают. Если это так, то можно продолжить и применить общую формулу для вычисления определителей, которая выглядит следующим образом: 9{i+j}a_{ij}M_{ij}

    Здесь $n$ — размер вашей квадратной матрицы. Затем вы выбираете некоторую строку или столбец и выполняете расширение, получая, таким образом, множество меньших определителей для вычисления. Для удобства выберите строку или столбец с наименьшими значениями или лучше с нулями, если это возможно. Такой подход сэкономит ваше время и позволит быстрее выполнить домашнее задание по математике.

© 2015 - 2019 Муниципальное казённое общеобразовательное учреждение «Таловская средняя школа»

Карта сайта